Серия Гран-при 2016/17. Результаты по вопросам




1. Вопрос 4.22: Согласно мусульманским поверьям, кисточка на феске предназначена для того же, что и ОНИ по представлениям другой религии. Назовите ИХ.

Ответ: Дре́ды

Зачёт: Дредло́ки; ло́ки

Комментарий: Существовало поверье, будто носителя за такую кисточку могли поднять на небо. Другая религия - растафариа́нство.

Источник: Н. Стоун. Краткая история Турции. http://flib.nwalkr.tk/b/382524/read

Автор: Расул Бабазаде (Баку)

Средняя оценка: 2.25

Комментарии (1)


2. Вопрос 7.44: Во время учёбы в техникуме будущая звезда советского баскетбола Ува́йс Ахта́ев занимался другим командным видом спорта. Рассказывают, что Ахтаев тратил намного меньше сил, чем остальные игроки, поскольку ДЕЛАЛ ЭТО. Ответьте тремя словами: что именно делал?

Ответ: ходил по дну.

Зачёт: стоял на дне, бегал по дну.

Комментарий: его рост составлял 236 сантиметров. Занимался Ахтаев водным поло.

Источник: https://www.championat.com/basketball/article-264124-istorija-centrovogo-almatinskogo-burevestnika-uvajsa-ahtaeva.html

Автор: Сергей Шорин (Краснодар)

Средняя оценка: 2.1

Комментарии


3. Вопрос 4.6: Натуралист Джейн Гу́долл в своих англоязычных научных работах о типах поведения шимпанзе одной из первых стала употреблять слова, ранее употреблявшиеся для описания человеческих поступков, а описывая отдельных шимпанзе, Гу́долл стала пользоваться ИМИ, несмотря на протесты редакторов журналов. Назовите любое из НИХ.

Ответ: he.

Зачёт: she; он; она.

Комментарий: ОНИ - это местоимения «он» и «она». Джейн Гу́долл рассказывала о шимпанзе, как будто они были «личностями», и использовала личные местоимения «он» и «она» вместо общепринятого «оно» - it. Редактор известного британского научного журнала Nature [нэ́йче] заменил личные местоимения на «оно» в её первой рукописи. Гудолл, в свою очередь, зачеркнула все «оно» и восстановила местоимения, позже сказав, что окончательный вариант, по крайней мере, «выявит тот факт, что шимпанзе бывают разного пола».

Источник: В. Морелл. Эти удивительные животные. http://flib.nwalkr.tk/b/449564/read

Автор: Расу́л Бабазаде́ (Баку)

Средняя оценка: 2.08

Комментарии


4. Вопрос 2.19: Организаторам первой советской антарктической экспедиции не удалось избежать непредвиденных ситуаций. Так, в марте некоторые участники стали замерзать. Причиной была ОНА. Назовите ЕЕ.

Ответ: линька.

Комментарий: Лайки начали линять в связи с наступлением весны в родном Северном полушарии, а в Антарктиде в это время зима как раз наступала.

Источник: передача «Советская Антарктида», канал ОРТ, эфир от 16.02.06г.

Автор: Денис Рыбачук (Брест)

Средняя оценка: 2.05

Комментарии


5. Вопрос 2.25: В вопросе мы сделали небольшие изменения. По словам олимпийского чемпиона Виктора Ждано́вича, первые уроки ему дал дома отец. Каким видом спорта занимался Жданович? Ответ: фехтование на рапирах. Зачёт: фехтование.

Комментарий: Жданович говорил, что первым тренером был Дюма-отец. "С его трилогии о приключениях д'Артаньяна началась моя любовь к этому виду спорта".

Источник: http://www.sport-express.ru/newspaper/2002-08-26/14_1/

Автор: Олег Кожедуб, Алексей Полевой (Гомель)

Средняя оценка: 2

Комментарии


6. Вопрос 2.38: ИКС - это замена. Власти Берлина решили, что у Гамбургского ИКСА будет иной путь развития. Судьбу какого европейского объекта он повторил?

Ответ: вокзала д’Орсэ́. Зачёт: по “д’Орсэ”.

Комментарий: теперь Гамбургский вокзал в Берлине - музей современного искусства.

Источник: https://ru.wikipedia.org/wiki/Гамбургский_вокзал_(Берлин)

Автор: Алексей Полевой (Гомель)

Средняя оценка: 2

Комментарии


7. Вопрос 3.4: Назовите город, покровителем которого является, как ни удивительно, святой РОмул.

Ответ: Сан-Ремо.

Зачёт: Санремо.

Комментарий: можно подумать, что название города связано с именем Рема, но это не так. В пещере на месте современного Сан-Ремо скончался святой Ромул Генуэзский. Сначала городок стали называть Сан-Ромоло, потом Сан-Ромо, а в итоге — Сан-Ремо.

Источник: https://ru.wikipedia.org/wiki/Сан-Ремо

Автор: Григорий Алхазов (Кишинёв)

Средняя оценка: 1.94

Комментарии


8. Вопрос 4.26: Яков Зельдо́вич – автор десятков книг и научных статей. Сти́вен Хо́кинг, однажды встретив Зельдо́вича, был сильно удивлен. Какую французскую фамилию он при этом упоминал?

Ответ: Бурбаки́.

Комментарий: На Западе считали что под именем «Зельдович» работает группа авторов, подобно группе французских математиков, объединенных под псевдонимом «Бурбаки́».

Источник:
    1. А. Миллер. Империя звезд, или Белые карлики и черные дыры. http://flibusta.is/b/453010/read 2.http://www.ega-math.narod.ru/Reid/Zeld.htm

Автор: Расу́л Бабазаде́ (Баку)

Средняя оценка: 1.94

Комментарии


9. Вопрос 2.42: Тихохо́дки обладают выдающимися способностями: выдерживают пребывание в открытом космосе, сверхнизкие температуры, мощное ионизирующее излучение, долговременное обезвоживание. Екатерина Мищенко советует одной компании обратить внимание на тихоходку, и тогда может появиться ОН. Назовите его словом с дефисом.

Ответ: человек-тихоходка.

Комментарий: Очевидно, что человек-тихоходка будет практически неуязвим и может стать новым героем комиксов Marvel.

Источник: http://indicator.livejournal.com/8686.html?media

Автор: Алексей Полевой (Гомель)

Средняя оценка: 1.9

Комментарии


10. Вопрос 4.35: Согласно одной мрачной шутке, ничто так не формирует внутренний стержень человека, как ЭТО. Назовите монарха, одно из прозвищ которого происходит, в частности, от слова ЭТО.

Ответ: Влад Це́пеш.

Зачёт: Влад Те́пеш, Дра́кула, Влад III, Влад Дра́кула, Граф Дра́кула.

Комментарий: Речь идет о казни в виде сажания на кол. Прозвище Влада Це́пеша – колосажатель.

Источник:
    1. http://www.inpearls.ru/comments/320645
    2. dic.academic.ru/dic.nsf/ruwiki/845516

Автор: Расул Бабазаде (Баку)

Средняя оценка: 1.9

Комментарии


11. Вопрос 3.3: Дуплет. Два вопроса по 30 секунд обсуждения, ответы сдаются на одном бланке.
    1. Рецензент называет матёрого политика Фрэнка Андервуда из американского сериала “Карточный домик” ИМ. Назовите ЕГО двумя словами.
    2. Самое известное ЕЁ изображение находится во Дворце консерваторов и имеет высоту 75 сантиметров и длину 114 сантиметров. Назовите ЕЁ двумя словами.

Ответ:
    1. Капитолийский волк; 2. Капитолийская волчица.

Зачёт: точные ответы.

Комментарий: в первом случае имеется в виду вашингтонский Капитолий. Римский ПалАццо-деи-КонсерватОри является одним из главных Капитолийских музеев.

Источник:
    1. https://www.facebook.com/andrey.bodrey/posts/1143971362282944
    2. https://en.wikipedia.org/wiki/Capitoline_Wolf

Автор: Григорий Алхазов (Кишинёв)

Средняя оценка: 1.86

Комментарии


12. Вопрос 2.17: Мы не зачтем ответ, в котором будет буква "к". По одной из версий, находившиеся в Европе солдаты из США стали разбавлять ... Что?

Ответ: эспрессо. Незачёт: экспрессо.

Комментарий: Таким образом, появилось название кофе “американо”, рецепт приготовления которого состоит в разбавлении водой крепкого эспрессо. Распространенная и раздражающая многих ошибка - называть одну из разновидностей кофе словом “экспрессо”.

Источник: https://en.wikipedia.org/wiki/Caff%C3%A8_Americano

Автор: Алексей Полевой (Гомель)

Средняя оценка: 1.82

Комментарии


13. Вопрос 7.32: Обложка одного альбома группы Dream Theater [дрим ти́этер], вышедшего осенью, ныне является коллекционной редкостью. На этой обложке, помимо прочего, изображено ярко-красное яблоко, на котором пылают ОНИ. Назовите ИХ.

Ответ: башни-близнецы.

Зачёт: точный ответ.

Комментарий: Альбом Live Scenes From New York [лайв синс фром нью йорк] вышел 11 сентября 2001 года, за несколько часов до атаки на Всемирный Торговый Центр. Сразу же после этих событий обложка, на которой было изображено пылающее Большое Яблоко с расположенными на нём силуэтами, олицетворяющими город, была изъята из продажи.

Источник: https://en.wikipedia.org/wiki/Live_Scenes_from_New_York

Автор: Евгений Долженко́в (Санкт-Петербург)

Средняя оценка: 1.82

Комментарии


14. Вопрос 4.14:

Раздаточный материал

    Засела смерть в сердце моем. Эти таблетки — ее семена, а виски — свежая вода для них. И лишь когда я буду под землей, жизнь сможет прорасти.

    Перед вами часть [ПРОПУСК]-спектакля по “Ромео и Джульетте”, который длился на протяжении пяти недель. Какое слово с удвоенной согласной мы пропустили?

Ответ: Тви́ттер.

Комментарий: Твиттер-спектакль Королевской шекспировской компании по "Ромео и Джульетте" длился 5 недель: актеры вели бло́ги героев и болтали с читателями. Отрывок из пьесы на раздатке состоит из 140 символов.

Источник: http://www.kommersant.ru/doc/2937743

Автор: Расул Бабазаде (Баку)

Средняя оценка: 1.77

Комментарии


15. Вопрос 4.11: Перед вами опубликованная в “Арт Джо́рнал” часть схемы устройства, предназначенного для создания ИХ. ОНИ неоднократно упоминаются в статье о голландце, сумевшим заработать около 30 миллионов долларов. Назовите ИХ двумя словами.

Ответ: Поддельные картины.

Зачёт: Подделки картин; фальсификации картин; фальшивые полотна; поддельные полотна; фальшивые картины; фальсификации полотен; подделки полотен.

Комментарий: Устройство предназначалось для искусственного состаривания современных картин, который вешались над ним. Хан Ван Ме́герен заработал свои миллионы на подделках картин мастеров, которые у него получались очень достоверными.

Источник:
    1. Ф. Хук. Завтрак у Sotheby's http://flibusta.is/b/452909/read#anotelink44
    2. http://cn.flibusta.is/i/9/452909/i_053.jpg
    3. https://ru.wikipedia.org/wiki/Ван_Мегерен,_Хан Авторы: Расу́л Бабазаде́, Алексе́й Ула́нов (оба - Баку)

Средняя оценка: 1.75

Комментарии


16. Вопрос 4.15: В одной миниатюре описывается нестандартная консультация со страховым агентом. Агент утверждает, что АЛЬФА его клиента слишком коротка. Назовите АЛЬФУ двумя словами.

Ответ: Линия жизни.

Комментарий: Агент во время консультации занимался хирома́нтией.

Источник: http://acomics.ru/~channelate/336

Автор: Расул Бабазаде (Баку)

Средняя оценка: 1.75

Комментарии (1)


17. Вопрос 4.17: По одной из версий, клуб «Бо́рнмут» получил своё прозвище благодаря красным футболкам. Согласно другой версии, раньше на месте домашнего стадиона клуба располагался ОН. В известном произведении ЕГО собирался уничтожить... Кто?

Ответ: (Ермолай) Лопа́хин.

Комментарий: Прозвище клуба – «вишенки». Согласно одной из версий, на месте домашнего стадиона клуба располагался вишневый сад.

Источник:
    1. http://fapl.ru/teams/41/
    2. http://www.sports.ru/tribuna/blogs/fc/525651.html

Автор: Расул Бабазаде (Баку)

Средняя оценка: 1.75

Комментарии


18. Вопрос 3.9: Распространённость узла как единицы измерения связана с удобством его применения в навигационных расчётах — например, 60 узлов соответствуют скорости ПРОПУСК. Святослав Сахарнов пишет, что косвенной причиной гибели “Титаника” было обещание, что лайнер прибудет в Нью-Йорк ПРОПУСК. Заполните пропуск тремя словами.

Ответ: минута в минуту.

Зачёт: точный ответ.

Комментарий: двигаясь вдоль меридиана, на скорости в 60 узлов судно за одну минуту проходит одну угловую минуту широты. Капитан “Титаника” не хотел выбиваться из графика и не снизил скорость, несмотря на предупреждения об айсбергах по курсу.

Источник:
    1. https://ru.wikipedia.org/wiki/Узел_(единица_измерения)
    2. http://mreadz.com/read-252373/p9

Автор: Максим Коцюруба (Кишинёв)

Средняя оценка: 1.66

Комментарии


19. Вопрос 5.5: Иллюстратор Гюста́в Доре́, как правило очень точный в исторических деталях, нарисовал ЕГО с эспаньолкой, нехарактерной для XVIII [восемнадцатого] века. Дело в том, что эспаньолку вернул в моду Наполеон III, так и не исполнивший своих предвыборных обещаний. Назовите ЕГО.

Ответ: [Карл Фридрих Иероним барон фон] Мюнхгаузен.

Зачёт: [Карл Фридрих Иероним барон фон] Мюнхаузен.

Комментарий: в XVIII веке эспаньолок не носили, так что вряд ли реальный барон выглядел так, как на хрестоматийных иллюстрациях Доре́. Эспаньолку носил Наполеон III, что даёт повод считать упомянутые иллюстрации карикатурой на французского императора.

Источник: https://ru.wikipedia.org/wiki/Наполеон_III

Автор: Александр Мудрый (Черновцы)

Средняя оценка: 1.64

Комментарии


20. Вопрос 4.23: Группа Die Antwoord [Ди А́нтворд] выпустила свой первый студийный альбом «SOS» в 2009 году, выложив его в свободный доступ для скачивания. Напишите название альбома так, как оно было задумано группой.

Ответ: $O$. [Ведущему: знак доллара — о — знак доллара.]

Комментарий: Музыканты фактически отказались от гонорара за альбом, что и подчеркнули, вынеся на обложку знаки доллара.

Источник: https://ru.wikipedia.org/wiki/Die_Antwoord

Автор: Эльну́р Гасымзаде́ (Баку)

Средняя оценка: 1.61

Комментарии


21. Вопрос 1.3: Рассказывают, что на коронации Эдгара Миролюбивого в 973 году восемь других правителей Британии решили стать его вассалами. Чтобы продемонстрировать своё желание работать вместе под командованием единого монарха, эти вассалы во время торжественной церемонии даже на время взяли в руки… Что?

Ответ: вёсла.

Комментарий: Вассалы на время стали гребцами. Впрочем, в первоначальных хрониках вассалов было не 8, а 6, да и вообще, сама история вызывает определённые сомнения.

Источник:
    1. https://senchus.wordpress.com/2008/09/13/nine-men-in-a-boat/
    2. https://en.wikipedia.org/wiki/Edgar_the_Peaceful

Автор: Анвар Мухаметкалиев (Алматы)

Средняя оценка: 1.55

Комментарии (1)


22. Вопрос 2.36: Дуплет. Два вопроса по 30 секунд обсуждения, ответы сдаются на одном бланке.
    1. В английском романе мастерская художника-мизантропа располагается в бывшей конюшне с больши́ми воротами. Для каких существ, по мнению художника, предназначена дверь поменьше?
    2. У традиционных жилищ бута́нцев низкая дверь. Кто, по мнению бутанцев, не умеет сгибаться?

Ответ: йе́ху, йе́ти.

Зачёт: еху; ети, снежный человек, бигфу́т.

Комментарий: в романе Олдоса Ха́ксли “Шутовско́й хоровод” есть явные отсылки к произведениям Джонатана Свифта. Людей персонаж не очень уважает и считает глуповатыми йеху, а гуигнгнмы-лошади для него - венец эволюции. На Тибете снежного человека опасаются и стараются от него защититься.

Источник:
    1. Олдос Хаксли «Шутовской хоровод»
    2. http://www.bbc.com/russian/society/2015/11/151102_yeti_why_end_of_legends

Автор: Денис Рыбачук (Брест) Алексей Полевой (Гомель)

Средняя оценка: 1.53

Комментарии


23. Вопрос 5.41: Нарисовав карту, американские инженеры Уильям Янг и Дуглас Ринг увидели сходство. Напишите англоязычный термин, появившийся в результате этого.

Ответ: cell phone [сэл фо́ун].

Зачёт: cellular phone [сэ́лулар фоун].

Комментарий: инженеры компании «Белл» нарисовали карту зон покрытия вышек беспроводной связи. Эта карта напомнила им клетки живого организма. В результате появился термин cell phone [сэл фо́ун], что буквально значит «клеточный телефон».

Источник: http://gizmodo.com/5840939/why-do-we-use-the-term-cellular-phone-instead-of-mobile-phone

Автор: Александр Кудрявцев (Николаев)

Средняя оценка: 1.53

Комментарии (2)


24. Вопрос 4.39: На обложке книги «Тайны советской кухни» серп и молот изображены ВТОРОЙ на ПЕРВОМ. Назовите ПЕРВЫЙ и добавьте в ответ ВТОРУЮ.

Ответ: Борщ, сметана.

Зачёт: в любом порядке.

Комментарий: Серп и молот изображены сметаной на борще. Борщ - это первое блюдо :)

Источник: http://flibusta.is/b/455858

Автор: Расул Бабазаде (Баку)

Средняя оценка: 1.52

Комментарии


25. Вопрос 4.3: В книге «Эволюция разума» ЭТО названо одной из причин, по которой ритмичная музыка является частью культуры всех человеческих цивилизаций. Назовите ЭТО точно.

Ответ: Сердцебиение матери.

Зачёт: Стук сердца матери, пульс матери, сердечные сокращения матери.

Комментарий: Как пишет Курцвейл, к третьему триместру плод набирается определенного опыта и его новая кора обучается. Он слышит звуки, особенно сердцебиение матери, и это одна из причин, объясняющих ритмичность музыки во всех человеческих культурах.

Источник: Р. Курцвейл. Эволюция разума. http://coollib.com/b/357478

Автор: Расу́л Бабазаде́ (Баку)

Средняя оценка: 1.49

Комментарии (1)


26. Вопрос 4.10: Этот француз сравнивал себя со скрипичной декой, улавливающей и откликающейся на эмоциональные вибрации произведения искусства. О ком идет речь?

Ответ: о Стенда́ле.

Зачёт: о Мари́-Анри́ Бейле.

Комментарий: «Синдром Стендаля» характеризуется крайне эмоциональной реакцией на восприятие произведения искусства.

Источник: Ф. Хук. Завтрак у Sotheby's http://flibusta.is/b/452909/read#anotelink44

Автор: Расу́л Бабазаде́ (Баку)

Средняя оценка: 1.49

Комментарии


27. Вопрос 4.18: Одна из разновидностей посуды, которой пользовались средневековые правители, отличалась специальным рисунком, менявшим цвет при контакте с НИМ. С каким существом этимологически связано название ЭТОГО?

Ответ: Мышь

Комментарий: Речь идет о мышьяке́. Таким образом, посуда служила своеобразным индикатором яда.

Источник:
    1. Н. Стоун. Краткая история Турции. http://bookz.ru/authors/norman-stoun/kratkaa-_641.html
    2. http://vasmer.lexicography.online/м/мышьяк

Автор: Расул Бабазаде (Баку)

Средняя оценка: 1.49

Комментарии


28. Вопрос 4.21: Пол Э́рдеш — автор огромного количества научных публикаций. Рассказывают, что Э́рдеш любил цитировать следующий афоризм: “Математик - это автомат по переработке ЭТОГО в теоремы”. Назовите ЭТО несклоняемым словом.

Ответ: Кофе

Комментарий: Чтобы умудриться написать такое огромное количество научных работ, ученому приходилось работать в том числе и по ночам. Для того чтобы максимизировать свою математическую результативность, Э́рдеш подпитывал мозг кофе.

Источник: С. Сингх. Симпсоны и их математические секреты. http://flibusta.is/b/444952/read

Автор: Расу́л Бабазаде́ (Баку)

Средняя оценка: 1.49

Комментарии


29. Вопрос 4.24: Согласно теории, выдвинутой в 1990 году, ЭТО возникает, когда определенное число фрагментов одной своеобразной голограммы совпадает с фрагментами другой, создавая путаницу. Автору книги «История мозга» эта теория представляется знакомой и напоминает объяснение, выдвинутое в 1969 г. психиатром Левита́ном. Назовите ЭТО.

Ответ: Дежавю́.

Зачёт: в любом написании (например, дежа вю, déjà vu).

Комментарий: В 1990 г. для объяснения этого явления доктора́ Герман Сно и Дон Ли́нчен предложили голографическую теорию дежавю́. Ученые утверждают, что восприятие и воспоминания подобны голограммам. Изучая эти теории, автор упомянутой книги также испытал дежавю́.

Источник: С. Джуан. История мозга. 1640 фактов http://flibusta.is/b/454367/read#t9

Автор: Расу́л Бабазаде́ (Баку)

Средняя оценка: 1.49

Комментарии


30. Вопрос 4.31: Работы аль-Бируни́ долго не были известны в Европе. Именно поэтому, как предполагает Сти́вен Ва́йнберг, у Бируни́ нет ЕГО. А какое ОНО у врача, родившегося в 980 году?

Ответ: Авице́нна.

Комментарий: ОНО — латинизированное имя. В отличие от Аверро́эса (Ибн Рушда) или Авице́нны (Ибн Сины), у Бируни́ его нет, что Ва́йнберг объясняет его малой известностью.

Источник:
    1. Вайнберг С. Объясняя мир http://flibusta.is/b/430757/read
    2. https://ru.wikipedia.org/wiki/Ибн_Сина

Автор: Расул Бабазаде́ (Баку)

Средняя оценка: 1.49

Комментарии


31. Вопрос 4.43: В середине двадцатого века учёные предположили, что, зафиксировав ИХ, детекторы американских спутников могут ошибочно инициировать Третью мировую войну. На самом деле ОНИ произошли гораздо раньше. Назовите ИХ точно.

Ответ: Вспышки сверхновых звезд

Зачёт: Вспышки/взрывы сверхновых, вспышки новых звезд, взрывы новых звезд.

Комментарий: Ученые поняли, что детекторы американских спутников, которые с разведывательной целью наблюдают за территорией Советского Союза, могут зафиксировать вспышки света от сверхновых и, приняв их за взрывы, инициировать третью мировую войну. При этом неважно, что сверхновые находятся на расстоянии сотен тысяч триллионов километров от Земли, а вспышки произошли более 100 тысяч лет назад.

Источник: А. Миллер Империя звезд, или Белые карлики и черные дыры. http://flibusta.is/b/453010/read

Автор: Расул Бабазаде (Баку)

Средняя оценка: 1.49

Комментарии


32. Вопрос 4.44: Согласно одной из гипотез, РНК образовалась раньше ДНК во многом потому, что природа нуждалась в зачаточных клетках, способных копировать самих себя. Сэм Кин, описывая данный процесс, упоминает работу 1948 года. Назовите ее автора.

Ответ: [Маури́с] Э́шер

Комментарий: РНК может сама себя копировать, как на литографии Э́шера «Рисующие руки», где две руки рисуют друг друга.

Источник: С. Кин. Исчезающая ложка. http://flibusta.is/b/429981/read

Автор: Расул Бабазаде (Баку)

Средняя оценка: 1.49

Комментарии


33. Вопрос 5.13: В XIX веке Бе́нджамин Гулд «оторвал» у Скорпиона лапу и «прикрепил» к НИМ. Назовите ИХ.

Ответ: Весы.

Зачёт: точный ответ.

Комментарий: астроном Бе́нджамин Гулд обратил внимание, что звезда Бра́хиум (что значит «Лапа»), относившаяся к созвездию Скорпиона, лежит очень далеко от ярких звёзд этого созвездия и довольно близко к звёздам из созвездия Весов. Гулд передал эту звезду Весам. С тех пор это сигма Весов.

Источник: https://ru.wikipedia.org/wiki/Сигма_Весов

Автор: Александр Кудрявцев (Николаев).

Средняя оценка: 1.44

Комментарии


34. Вопрос 6.20:

Раздаточный материал

    Есть где-то мир, исполненный чудес. Там лысый ** водит внуков в лес, Пузатый *** пускает грог по кругу, А господа *** и ** Из пистолетов лупят друг по другу.

    Заполните пропуски в стихотворении Григория Остера четырьмя фамилиями.

Ответ: Пушкин, Лермонтов, Мартынов, Дантес

Источник: http://citati-ru.livejournal.com/1142406.html

Автор: Юлия Кальменс (Чикаго)

Средняя оценка: 1.43

Комментарии


35. Вопрос 4.4: Рассказы о Дже́ке Керуа́ке и Ни́ле Кэ́ссиди современники сравнивали с приключениями персонажей другого известного произведения. Назовите город, в котором скончался Керуа́к.

Ответ: Сент-Пи́терсберг.

Зачёт: Санкт-Петербу́рг.

Комментарий: Многие сравнивали Керуа́ка и Кэ́ссиди с повзрослевшими Томом Сойером и Ге́кльберри Фи́нном. По совпадению, Керуа́к последние годы жизни провел в американском городе Сент-Пи́терсберг, правда, в штате Флорида. В одноименном вымышленном городе, находящемся в штате Миссу́ри, разворачиваются действия романа Марка Твена.

Источник: http://www.aif.ru/culture/person/tihiy_golos_pokoleniya_istoriya_dzheka_keruaka

Автор: Расу́л Бабазаде́ (Баку)

Средняя оценка: 1.43

Комментарии


36. Вопрос 3.33: Отметив ЕЁ, скаут сказал, что футболист работает с мячом подобно дьяволу. Известный актёр в недавнем интервью признался, что ОНА до сих пор появляется во время съемок. Назовите ЕЁ.

Ответ: хромота.

Зачёт: хромоногость.

Комментарий: согласно поверьям, хромота является отличительной чертой дьявола, поскольку он повредил ногу, упав с небес. В первом случае речь идет о бразильском футболисте Гарринче, во втором — о Хью Лори, исполнившем роль доктора Хауса.

Источник:
    1. http://www.sports.ru/tribuna/blogs/football_all_around/493101.html
    2. https://goo.gl/MK8SS9

Автор: Григорий Алхазов (Кишинёв)

Средняя оценка: 1.41

Комментарии


37. Вопрос 4.40: В 1918 году этот писатель получил квартиру в Большом Кремлевском дворце, его книги издавались миллионными тиражами, а нарком культуры Лунача́рский называл его великим. Назовите этого писателя.

Ответ: Демья́н Бедный.

Зачёт: (Ефим) Придво́ров.

Комментарий: Подобного успеха писатель смог добиться благодаря хорошему отношению к нему власти, будучи по сути придворным литератором, и даже живя в Кремле. В то время Демьян Бедный был далеко не беден.

Источник: https://ru.wikipedia.org/wiki/Демьян_Бедный

Автор: Алексей Уланов (Баку)

Средняя оценка: 1.39

Комментарии


38. Вопрос 5.45: Одной из достопримечательностей города О́да являются часы высотой более пяти метров. Эти часы заводят раз в год, что привлекает множество посетителей. Какое слово мы заменили в тексте этого вопроса?

Ответ: переворачивают.

Зачёт: точный ответ.

Комментарий: это песочные часы, одни из самых больших в мире. Песок весом в одну тонну подобран таким образом, чтобы он сыпался ровно один год.

Источник: http://glavnoe.ua/news/n243814

Автор: Александр Мудрый (Черновцы)

Средняя оценка: 1.39

Комментарии


39. Вопрос 4.19: В статье о безрогих коровах журналист пишет, в том числе, и о редактировании ЕГО. Назовите ЕГО словом греческого происхождения.

Ответ: Генома

Зачёт: в любой форме.

Комментарий: Исследователи предложили метод получения безрогих коров при помощи так называемого редактирования генома. После данной манипуляции родились здоровые телята, не имеющие рогов. Как полагают авторы, он может найти применение в животноводстве, так как только в США сейчас у 80 % поголовья крупного рогатого скота ежегодно подрезают рога, чтобы животные не ранили друг друга и людей.

Источник: http://www.polit.ru/news/2016/05/11/ps_no_horns/

Автор: Расу́л Бабазаде́ (Баку)

Средняя оценка: 1.38

Комментарии


40. Вопрос 1.6: Политик Ке́ннет Кларк весьма харизматичен и даже полушутя считает, что однажды своим громким криком смог убедить ЕГО, стоявшего недалеко. В англоязычной Википедии указано, что ЕГО имя в переводе с арабского означает “возможность добиться успеха”. Напишите ЕГО фамилию.

Ответ: Бахра́мов.

Комментарий: Возможность добиться самого большого успеха в истории футбольной сборной англичанам предоставил советский боковой судья То́фик Бахрамов. Будущий британский министр Кеннет Кларк был тогда ещё рядовым болельщиком, стоявшим на трибуне недалеко от судьи.

Источник:
    1. https://www.theguardian.com/football/2001/jun/04/sport.comment
    2. https://en.wikipedia.org/wiki/Kenneth_Clarke#cite_note-49
    3. https://en.wikipedia.org/wiki/Tawfik
    4. http://www.ft.com/cms/s/0/6460f310-36cf-11da-bedc-00000e2511c8.html#axzz4JrvRppig

Автор: Павел Солахян (Ереван)

Средняя оценка: 1.36

Комментарии


41. Вопрос 4.41: Умбе́рто Э́ко пишет, что Холодная война стала первым случаем, когда мир осознал, что современная война не имеет ничего общего с классическими конфликтами и что сейчас любая ОНА так или иначе оказывается ТАКОЙ. Какие два слова, начинающиеся на одну и ту же букву, мы заменили словами ТАКАЯ ОНА?

Ответ: Пи́ррова победа.

Комментарий: Умберто Эко считает, что современная война не имеет ничего общего с классическими конфликтами, где в конце концов с одной стороны оказывались проигравшие, а с другой — победители.

Источник: https://esquire.ru/umberto-eco-15

Автор: Расул Бабазаде (Баку)

Средняя оценка: 1.34

Комментарии


42. Вопрос 5.22: Во время НЕЁ смертельную опасность представляют порывы ветра, который колышет полотно непредсказуемым образом. Назовите ЕЁ.

Ответ: коррида.

Зачёт: точный ответ.

Комментарий: как известно, бык во время корриды реагирует не на цвет мулеты, а на её движения. Поэтому сильный ветер на арене – большая опасность для матадора.

Источник: https://goo.gl/4eZ67i

Автор: Александр Мудрый (Черновцы)

Средняя оценка: 1.33

Комментарии


43. Вопрос 5.24: [Ведущему: максимально чётко прочитать последнее предложение вопроса, чтобы игроки поняли, что ИКС – одушевлённое, а ИГРЕК – неодушевлённое] Франсуа Турт в конце XVIII века заменил прямую трость изогнутой, а волосы расположил в виде ленты. За его мастерство Турта прозвали ИКСОМ ИГРЕКА. В советском детективном романе ИКС пользовался ИГРЕКОМ Турта, хотя, конечно, это анахронизм. Назовите ИКСА и ИГРЕК словами, начинающимися на одну букву.

Ответ: Страдивари, смычок.

Зачёт: в любом порядке.

Комментарий: роман – «Визит к Минотавру» братьев Вайнеров. Один из его героев – Антонио Страдивари. Франсуа Турт родился через 10 лет после смерти Страдивари.

Источник:
    1. http://dic.academic.ru/dic.nsf/enc_music/7706/Турт
    2. https://ru.wikipedia.org/wiki/Страдивари,_Антонио
    3. http://lib.ru/RUSS_DETEKTIW/WAJNERY/minotawr.txt

Автор: Игорь Коршовский (Тернополь)

Средняя оценка: 1.33

Комментарии


44. Вопрос 7.9: Согласно верованиям жителей Калаха́ри, ОНА – это лестница, по которой спускается дождь. Назовите ЕЁ двумя словами, начинающимися на парные согласные.

Ответ: шея жирафа.

Зачёт: точный ответ.

Комментарий: такая лестница довольно длинная, так что неудивительно, что дождя в пустыне приходится ждать долго.

Источник: https://www.youtube.com/watch?v=IVzZC2kRo08 Автор Александр Сидоренков (Смоленск)

Средняя оценка: 1.33

Комментарии


45. Вопрос 3.17:

Раздаточный материал

    Laissant son kimono bleu à trois gamins, elle partit matinalement.

    В 2012 году французскую мнемоническую фразу “Она ушла рано утром, оставив свое синее кимоно троим детям” удлинили на одно слово. Какое событие обозначает в этой фразе запятая?

Ответ: распад СССР.

Зачёт: распад/развал СССР/[Советского] Союза.

Комментарий: это фраза для запоминания руководителей СССР и Российской Федерации. До запятой — Ленин, Сталин, Хрущев, Брежнев, Андропов, Черненко, Горбачев. После запятой — Ельцин, Путин, Медведев. В 2012 году, после второго пришествия Путина к власти, окончание фразы пришлось поменять.

Источник:
    1. рассказ преподавателя французского языка
    2. https://fr.wikibooks.org/wiki/Liste_de_mnémoniques

Автор: Марина Соловьева (Кишинёв)

Средняя оценка: 1.33

Комментарии (3)


46. Вопрос 1.33: Фрэнк рассказывал, что был кем-то вроде инспектора по безопасности: следил, чтобы все работники надевали каски и рисовал пути эвакуации по пустыне. Тем не менее, он гордо называл себя “дядей”. Напишите фамилию Фрэнка.

Ответ: О́ппенгеймер.

Комментарий: Родной брат “отца атомной бомбы” Роберта Оппенгеймера Фрэнк тоже был физиком и участвовал в ядерных испытаниях. Брат “отца” может формально считаться “дядей”.

Источник:
    1. http://goo.gl/kYByqu
    2. https://ru.wikipedia.org/wiki/Оппенгеймер,_Фрэнк

Автор: Павел Солахян (Ереван)

Средняя оценка: 1.31

Комментарии


47. Вопрос 3.29: Ральф РАнгник дважды проделывал со своими клубами путь от региональной лиги до первой Бундеслиги. Шутят, что Рангник даже на НЕЙ всегда выбирает самый нижний уровень. Назовите ЕЁ двумя словами, которые начинаются на одну и ту же букву.

Ответ: подземная парковка.

Зачёт: точный ответ.

Комментарий: Рангник ставит свою машину на самый нижний уровень подземной парковки, потому что любит подниматься с самых низов. Сначала он проделал этот путь с “Хоффенхаймом”, потом — с клубом “РБ Лейпциг” (его спонсором, кстати, является компания Red Bull).

Источник: http://www.sports.ru/tribuna/blogs/gorkynavkus/1082999.html

Автор: Григорий Алхазов (Кишинёв)

Средняя оценка: 1.28

Комментарии (2)


48. Вопрос 5.3: Одна из НИХ – переодеться бедным школьным учителем и в какой-нибудь захолустной церкви попросить разрешения сыграть на церковном органе. Другая ОНА появилась в Лейпциге. Назовите ЕЁ двумя словами.

Ответ: шутка Баха.

Зачёт: «Шутка» Баха.

Комментарий: музыка Баха была настолько великолепной и мощной, что прихожане в испуге разбегались, решив, что в церковь зашёл переодетый дьявол. Скерцо из «Сюиты для оркестра № 2 Си минор» более известно под названием «Шутка» Баха.

Источник: http://www.kulturologia.ru/blogs/180116/28071/ https://en.wikipedia.org/wiki/Orchestral_suites_(Bach)

Автор: Александр Мудрый (Черновцы)

Средняя оценка: 1.27

Комментарии


49. Вопрос 5.25: В Канаде ИМИ называют волонтёров, которые помогают пенсионерам или людям с ограниченными возможностями с уборкой придомовой территории. 4 декабря 2016 года игрок футбольной команды из Виско́нсина Рэ́ндалл Кобб сделал ЕГО после тачдауна. Назовите ЕГО двумя словами.

Ответ: снежный ангел.

Зачёт: точный ответ.

Комментарий: волонтёры помогают расчищать снег у домов людей, которые сами это сделать не в состоянии. Лёжа на заснеженном поле, Кобб стал делать характерные движения руками и ногами из стороны в сторону, создавая фигуру ангела.

Источник:
    1. http://vancouver.ca/streets-transportation/snow-angel.aspx
    2. http://www.calgary.ca/CSPS/CNS/Pages/Seniors/Snow-Angels/Snow-Angels.aspx
    3. https://www.youtube.com/watch?v=GxRgW7e5CXs

Автор: Александр Мудрый (Черновцы)

Средняя оценка: 1.27

Комментарии


50. Вопрос 5.30: ОН входил в состав популярного во второй половине XIX века «Вина инков». Александр Вертинский в книге воспоминаний «Дорогой длинною» рассказал, в том числе, и о НЁМ. Назовите ЕГО.

Ответ: кокаин.

Зачёт: кокаинум :)

Комментарий: во второй половине позапрошлого века кокаин ещё не считался наркотиком и добавлялся во многие продукты. В автобиографии Вертинский рассказал и о своей кокаиновой зависимости.

Источник: https://ru.wikipedia.org/wiki/История_кокаина

Автор: Александр Мудрый (Черновцы)

Средняя оценка: 1.27

Комментарии


51. Вопрос 5.34: В книге Елены Кно́рре рассказывается о самых тяжёлых элементах таблицы Менделеева. В английском переводе названия этой книги упоминается несуществующая страна. Назовите эту страну.

Ответ: Трансурания.

Зачёт: точный ответ.

Комментарий: трансурановые элементы – радиоактивные элементы, расположенные в таблице Менделеева за ураном. В оригинале книга Елены Кно́рре называется «Путешествие в мир трансуранов». Латинская приставка «транс-» означает «за».

Источник:
    1. http://spisok-literaturi.ru/books/puteshestvie-v-mir-transuranov_26244208.html
    2. https://books.google.com.ua/books/about/A_Visit_to_Transurania.html?id=o_6hAAAACAAJ&redir_esc=y
    3. http://tolkslovar.ru/t4198.html

Автор: Игорь Коршовский (Тернополь)

Средняя оценка: 1.27

Комментарии


52. Вопрос 4.5: Дэ́ниел Брук пишет, что Шанхай был создан как часть передового западного мира, посреди отсталого Китая. Затем Брук называет Шанхай ИМ. Назовите ЕГО устойчивым выражением их трех слов.

Ответ: Окно в Европу.

Комментарий: Проводится аналогия с Санкт-Петербургом, хотя и не с тем, о котором был предыдущий вопрос.

Источник: http://www.bfm.ru/news/279387

Автор: Расу́л Бабазаде́ (Баку)

Средняя оценка: 1.27

Комментарии


53. Вопрос 3.16: Под микроскопом видно, что “рабочие” элементы новой АЛЬФЫ грубые и шероховатые. Через несколько месяцев они становятся гладкими, и АЛЬФА приходит в негодность. Согласно соннику, новая АЛЬФА, увиденная во сне, предвещает прогресс в любовных отношениях. Назовите АЛЬФУ двумя словами.

Ответ: зубная щетка.

Зачёт: в любом числе.

Комментарий: можно подумать, что щетинки новой зубной щетки должны выглядеть аккуратно, но их специально делают шероховатыми, поскольку менее гладкие щетинки лучше чистят зубы. Нередко появление новой зубной щетки в квартире является индикатором перехода любовных отношений на новый уровень.

Источник:
    1. https://www.youtube.com/watch?v=cwN983PnJoA
    2. http://enigma-project.ru/sonnik/zubnaya-shhetka

Автор: Андрей Галушка (Кишинёв)

Средняя оценка: 1.26

Комментарии


54. Вопрос 7.7: По мнению автора вопроса, трещина в стволе пистолета и трещина в кобуре – это ИКС. Один из персонажей картины под названием «ИКС» – священник. Назовите ИКС.

Ответ: неравный брак.

Зачёт: точный ответ.

Комментарий: брак в стволе и кобуре пистолета неравнозначен по последствиям.

Источник:
    1. ЛОАВ
    2. http://ru.wikipedia.org/wiki/Неравный_Брак(картина) Автор Александр Сидоренков (Смоленск)

Средняя оценка: 1.24

Комментарии


55. Вопрос 7.45: В одной статье предлагается, чтобы претенденты на НЕЁ демонстрировали выписку с банковского счёта по крайней мере на сто миллионов долларов. Назовите ЕЁ точно.

Ответ: Нобелевская премия по экономике.

Зачёт: точный ответ.

Комментарий: в качестве доказательства экспериментального подтверждения своих теорий.

Источник: http://www.financial-math.org/blog/2016/07/where-are-the-billionaire-financial-academics/

Автор: Александр Коробейников (Саратов)

Средняя оценка: 1.24

Комментарии


56. Вопрос 4.28: Художественный критик Дюранти́ говорил, что в живописи, как и в НЁМ, чтобы добиться высот, необходимы умение и хватка. Назовите ЕГО словом, произошедшим от топонима.

Ответ: Альпинизм.

Источник: А. Данчев. Сезанн. Жизнь. http://flib.nwalkr.tk/b/464722/read

Автор: Расул Бабазаде (Баку)

Средняя оценка: 1.22

Комментарии


57. Вопрос 5.19: Николай Пинегин, рассказывая об экспедиции Георгия Седова, пишет, что лежавший перед путешественниками пролив весь оказался запечатанным. В предыдущем предложении мы заменили одно слово. Напишите это слово.

Ответ: запакованным.

Зачёт: запакован.

Комментарий: во время своей экспедиции к Северному полюсу Георгию Седову не раз преграждал путь паковый лёд – многолетний полярный морской лёд, просуществовавший более двух годовых циклов нарастания и таяния.

Источник:
    1. http://az.lib.ru/p/pinegin_n_w/text_0010.shtml
    2. https://ru.wikipedia.org/wiki/Паковый_лёд

Автор: Александр Мудрый (Черновцы)

Средняя оценка: 1.21

Комментарии (3)


58. Вопрос 3.26: В 1989 году ОНА покинула родину и переехала в Америку, обосновалась в Оклахоме и тренирует детей. Примечательно, что в этом же штате компактно проживают ОНИ. ОНИ короче НЕЁ на одну букву. Назовите ИХ и ЕЁ.

Ответ: команчи, КоманЕчи.

Зачёт: в любом порядке.

Комментарий: румынская гимнастка Надя Команечи, пятикратная олимпийская чемпионка, на играх в Монреале ставшая первой гимнасткой, получившей высший балл на международных соревнованиях. Незадолго до румынской революции она бежала из страны и сейчас живет в одном штате с индейцами из племени команчей.

Источник:
    1. https://ru.wikipedia.org/wiki/Команечи,_Надя
    2. https://ru.wikipedia.org/wiki/Команчи

Автор: Григорий Алхазов (Кишинёв)

Средняя оценка: 1.19

Комментарии


59. Вопрос 4.27: Герой экранизированного романа «Марсианин», получив перелом ребер, решает не сдерживать своей боли, поскольку… Закончите мысль героя по-русски семью словами.

Ответ: В космосе никто не услышит твой крик.

Зачёт: В космосе никто не услышит твоего крика.

Комментарий: Данная реплика является и тегла́йном к знаменитому фильму Ри́дли Ско́тта «Чужой». Ридли Скотт был и режиссёром «Марсианина», так что, вероятнее всего, это сознательная аллюзия.

Источник:
    1. Э. Вейер. Марсианин. https://www.litres.ru/endi-veyer/marsianin-2/chitat-onlayn/
    2. https://www.kinopoisk.ru/film/386/

Автор: Тейму́р Баба́ев (Цюрих)

Средняя оценка: 1.18

Комментарии


60. Вопрос 5.8: В одной поучительной истории рассказывается, как трое мужчин приготовили изысканное блюдо. Отделив небольшую часть, мужчины угостили нищего. Нищий незаметно угощение спрятал и съел его тогда, когда несколько часов спустя снова увидел этих мужчин. Назовите это блюдо.

Ответ: фугу.

Зачёт: точный ответ.

Комментарий: мужчины не были уверены, что правильно приготовили опасное блюдо. Они угостили им нищего, чтобы посмотреть, останется ли тот в живых. Нищий сделал вид, что съел его, и мужчины радостно побежали домой, чтобы отведать деликатес.

Источник: https://en.wikipedia.org/wiki/Fugu

Автор: Александр Кудрявцев (Николаев)

Средняя оценка: 1.18

Комментарии


61. Вопрос 5.29: Назовите фамилию Ивана, для которого около тридцати лет назад, был создан крупнокалиберный пистолет Подбырина.

Ответ: Данко.

Зачёт: точный ответ.

Комментарий: «лучший в мире советский пистолет системы Подбырина калибра 9,2 мм». Его создали специально для фильма «Красная жара» в оружейной мастерской Тима Лафранса. В этом фильме Арно́льд Шварцене́ггер сыграл советского милиционера Ивана Данко. На раздаточном материале памятник Данко – персонажу рассказа Горького, который спас свой народ, вырвав из груди собственное сердце.

Источник:
    1. https://ru.wikipedia.org/wiki/Красная_жара_(фильм,_1988)
    2. https://ru.wikipedia.org/wiki/Данко

Автор: Александр Кудрявцев (Николаев)

Средняя оценка: 1.18

Комментарии


62. Вопрос 5.43: На старинной карикатуре под названием «Сатана вдохновляет Лютера» дьявол держит в руках ЕЁ в форме головы Мартина Лютера, в ухо которому вставлена трубка. Назовите ЕЁ.

Ответ: волынка.

Зачёт: точный ответ.

Комментарий: дьявол наполняет голову Лютера ересью, а он распространяет ересь повсюду.

Источник: http://www.hellenicaworld.com/Art/Literature/JamesParton/en/CaricatureAndOtherComicArt.html

Автор: Александр Кудрявцев (Николаев)

Средняя оценка: 1.18

Комментарии


63. Вопрос 7.8: Владимир Щербинин в последний момент отказался принять постриг и, по собственным словам, из ИГРЕКА стал ГРЕКОМ. Какие слова мы заменили ИГРЕКОМ и ГРЕКОМ?

Ответ: по́слушник; ослу́шник.

Зачёт: в любом порядке.

Комментарий: ослушался авторитетов и передумал.

Источник: Щербинин В. Сердце сокрушенно. М., 2016. С. 215.

Автор: Александр Коробейников (Саратов)

Средняя оценка: 1.18

Комментарии


64. Вопрос 7.18: За какой фильм получила «Оскар» актриса, которой принадлежит рекорд длительности прохода на сцену за наградой?

Ответ: «Унесённые ветром».

Зачёт: точный ответ.

Комментарий: «Оскар» за лучшую женскую роль второго плана получила негритянка Хэтти Макдэниел, которой пришлось пробираться из дальнего угла сегрегированного зала.

Источник:
    1. http://www.kommersant.ru/doc/3213498#id=2
    2. https://en.wikipedia.org/wiki/Hattie_McDaniel

Автор: Александр Коробейников (Саратов)

Средняя оценка: 1.18

Комментарии


65. Вопрос 3.7: [Ведущему: сделать паузу на месте многоточия.] Предел ШОкли-КвИссера не позволяет сделать современные солнечные батареи достаточно эффективными для широкого применения в авиации. Возможно, создателю самолета “СОлар Импульс” Бертрану ПиккАру, сыну Жака и внуку Огюста Пиккара, удастся решить эту проблему благодаря появлению солнечных батарей… Ответьте абсолютно точно, какие два слова мы пропустили в конце предыдущего предложения?

Ответ: третьего поколения.

Зачёт: в любом падеже.

Комментарий: этот предел смогут преодолеть так называемые солнечные батареи третьего поколения. Бертран Пиккар — представитель третьего поколения династии исследователей и путешественников: его дед Огюст изобрел стратостат и батискаф, а отец Жак спускался в Марианскую впадину.

Источник:
    1. ttp://synergyfiles.com/2015/04/solar-impulse-an-aircraft-changing-aviation-history/
    2. https://en.wikipedia.org/wiki/Third-generation_photovoltaic_cell
    3. https://ru.wikipedia.org/wiki/Пиккар,_Бертран

Автор: Максим Коцюруба (Кишинёв)

Средняя оценка: 1.17

Комментарии


66. Вопрос 3.8: В каждом из последних нескольких сезонов Премьер-лиги “Сандерленд” подолгу находился на дне турнирной таблицы, но к концу турнира выпутывался из безвыходной ситуации. Назовите человека, с которым за это сравнивают “Сандерленд”.

Ответ: [Гарри] Гудини.

Зачёт: по фамилии; Эрик Вайс.

Комментарий: великий эскапист тоже много раз шел ко дну, но выпутывался и спасался.

Источник: https://goo.gl/92ruqY

Автор: команда WTH (Кишинёв)

Средняя оценка: 1.14

Комментарии


67. Вопрос 2.2: Хоккеист Го́рди Хоу рос во времена Великой депрессии. Однажды мама принесла домой мешок с вещами, которые разделила между детьми. Ответьте двумя словами, что вскоре купил у сестры Горди?

Ответ: второй конек.

Зачёт: левый конек, правый конек.

Комментарий: Мама очень буквально подошла к делению вещей "поровну". Кто знает, как сложилась бы жизнь Хоу, если бы в свое время сестра не согласилась продать второй конек из пары.

Источник: http://sport.tut.by/news/hockey/500103.html

Автор: Алексей Полевой (Гомель)

Средняя оценка: 1.12

Комментарии


68. Вопрос 2.39: В историческом фильме заключенный просит ЕГО сшить сюртук и изготовить цилиндр. Назовите его фамилию.

Ответ: Бертильо́н.

Комментарий: Альфонс Бертильон слышит эти слова во время обмера заключённого, который сейчас известен как бертильона́ж.

Источник: д/ф «История криминалистики», 2014, 1 серия http://tfile.co/forum/viewtopic.php?t=758246

Автор: Денис Рыбачук (Брест)

Средняя оценка: 1.12

Комментарии


69. Вопрос 5.44: Невежественный шотландец из романа Вальтера Скотта после битвы 1745 года ничего не получил при дележе трофеев. Он злорадствует, что доставшиеся соратнику ОНИ околели в тот же вечер, не подозревая, что ИХ легко можно «воскресить». Назовите ИХ.

Ответ: часы.

Зачёт: карманные часы.

Комментарий: невежественный шотландец считал часы живым существом. Часы околели из-за того, что закончился завод.

Источник: В. Скотт. Уэверли, или шестьдесят лет назад. – М., «Правда», 1990, с. 335.

Автор: Игорь Коршовский (Тернополь)

Средняя оценка: 1.09

Комментарии


70. Вопрос 7.34: Небогатые персонажи Герцена мечтают, как в одном месте получат какой-нибудь подарок, в другом пообедают – вот ИКСЫ и окажутся вместе. Назовите ИКСЫ одним словом.

Ответ: концы.

Зачёт: точный ответ.

Комментарий: «Свести концы с концами» – синоним преодоления трудных времен.

Источник:
    1. https://ru.wiktionary.org/wiki/сводить_концы_с_концами
    2. А. Герцен. Былое и думы http://rulibrary.ru/gercen/byloe_i_dumy/163

Автор: Александр Сидоренков (Смоленск)

Средняя оценка: 1.09

Комментарии


71. Вопрос 6.16: Автор статьи на луркморе приходит к выводу, что единственным достоинством электронно-лучевых мониторов было то, что они служили котам ТАКОЙ лежанкой. Какие два прилагательных мы заменили на ТАКАЯ?

Ответ: Теплая ламповая.

Комментарий: Любители старой техники испытывают ностальгические чувства к теплому ламповому звуку. Редакторы турнира предпочитают теплое ламповое ЧГК.

Источник: http://lurkmore.to/Тёплый_ламповый_звук

Автор: Владимир Грамагин (Нью-Йорк)

Средняя оценка: 1.08

Комментарии


72. Вопрос 6.21: Британский музей использует для защиты некоторых экспонатов недавно разработанное вещество, влияющее на ориентацию ЕЕ. Назовите ЕЕ коротким словом.

Ответ: Моль.

Комментарий: Часть мужских особей моли обрабатывается специальной пудрой, изобретенной компанией Exosect, затем их выпускают в места, облюбованные молью. Они привлекают к себе остальных самцов, оставляя самок без потомства. Трудно сказать, меняет ли эта пудра сексуальную ориентацию самцов моли, но по крайней мере она дезориентирует их в пространстве. Таким способом Британский Музей решает проблему уничтожения бесценных экспонатов молью.

Источник: http://artdaily.com/news/79401/London-s-Natural-History-Museum-deploys--gay--moth-to-stop-the-damaging-cloth-eating-insects#.WEixBuYrK70

Автор: Мара Могилевская (Чикаго)

Средняя оценка: 1.08

Комментарии


73. Вопрос 2.3: Скандинавы строили свои жилища из брикетов торфа. Сара Па́ркик называет эти брикеты природным… Чем?

Ответ: LEGO.

Зачёт: лего.

Комментарий: Брикеты были прямоугольными, легко совмещались и плотно прилегали друг к другу. "Родина" LEGO - Дания, скандинавская страна.

Источник: Неизвестные викинги. Под парусами драккара / The Vikings Uncovered, 2016, 2 серия, http://tfile.co/forum/viewtopic.php?t=846494

Автор: Денис Рыбачук (Брест)

Средняя оценка: 1.06

Комментарии


74. Вопрос 5.2: В заметке под названием «Утраченное благородство» рассказывается о том, как ВТОРОЙ был, так сказать, лишён «неприкосновенности». Назовите ВТОРОЙ.

Ответ: гелий.

Зачёт: точный ответ.

Комментарий: благородные газы, к которым относится и гелий, долгое время считались абсолютно инертными. Однако химики не сдавались и одного за другим лишали их «химической неприкосновенности». Одним из последних недавно сдался гелий, который удалось соединить с натрием в электридный кристалл Na2He [натрий два гелий].

Источник: http://golbis.com/pin/utrachennoe-blagorodstvo/

Автор: Александр Мудрый (Черновцы)

Средняя оценка: 1.06

Комментарии


75. Вопрос 5.16: У жителей одного городка, переехавших в новые дома, начали зеленеть волосы. Вскоре выяснилось, что виноваты бракованные ОНИ. Что предшествует ИМ в известном выражении?

Ответ: огонь и вода.

Зачёт: вода; огонь.

Комментарий: ОНИ – медные трубы. Соединения меди дают зелёный цвет.

Источник: http://www.ig-nobel.ru/a2012him.php

Автор: Александр Кудрявцев (Николаев)

Средняя оценка: 1.06

Комментарии


76. Вопрос 7.4: «АЛЬФА в пупке» – английская идиома, которая обозначает девушку с разворота. Из других АЛЬФ однажды сделали цепь длиной в 15 километров. Назовите АЛЬФУ.

Ответ: скрепка.

Зачёт: точный ответ.

Комментарий: журнальный разворот обычно вырывают и вешают на стену. Листы разворота соединяются скрепкой, которая оказывается примерно посередине изображения девушки – иногда действительно на пупке.

Источник:
    1. http://www.ar15.com/forums/t_1_5/1512819_A_Great_American__Male__Tragedy.html
    2. https://ru.wikipedia.org/wiki/Скрепка

Автор: Александра Киланова (Санкт-Петербург)

Средняя оценка: 1.06

Комментарии


77. Вопрос 7.25: Древнеримский историк в шутку приписал ИМ слова, обращённые к императору: «Если ты победил, мы пропали!» Назовите словом греческого происхождения то, чего ОНИ опасались.

Ответ: гекатомба.

Зачёт: точный ответ.

Комментарий: ОНИ — это быки, которых по случаю победы активно приносили в жертву.

Источник:
    1. Аман А.-Г. Повседневная жизнь первых христиан. 95-197 http://flib.nwalkr.tk/b/231028/read
    2. https://ru.wikipedia.org/wiki/Гекатомба

Автор: Александр Коробейников (Саратов)

Средняя оценка: 1.06

Комментарии


78. Вопрос 7.28: ЕГО изначальный вариант символизировал чуму, чтобы военные держались подальше. Назовите ЕГО двумя словами.

Ответ: Весёлый Роджер.

Зачёт: пиратский флаг, флаг пиратов.

Комментарий: изначально флаг был чёрный с двумя белыми диагональными линиями.

Источник: https://ru.wikipedia.org/wiki/Весёлый_Роджер

Автор: Сергей Шорин (Краснодар)

Средняя оценка: 1.06

Комментарии (8)


79. Вопрос 1.17: Рассказывают, что мать Ричарда Хо́ллингсхеда была очень крупной женщиной и чувствовала себя некомфортно в обычных креслах. После нескольких лет экспериментов в собственном дворе Холлингсхед открыл первый в истории ИКС. Назовите ИКС точно.

Ответ: автокинотеатр

Зачёт: drive-in, драйв-ин, автомобильный кинотеатр.

Комментарий: Мать Холлингсхеда не могла ходить в обычные кинотеатры. В своём дворе Холлингсхед отрабатывал различные положения экрана для удобного просмотра с любой точки.

Источник:
    1. https://en.wikipedia.org/wiki/Richard_Hollingshead
    2. http://www.umich.edu/~drivein/theater.html

Автор: Анвар Мухаметкалиев (Алматы)

Средняя оценка: 1.05

Комментарии (3)


80. Вопрос 4.12: В одной научно-популярной книге ЕГО сравнили с машиной времени, позволяющей ученым заглянуть в прошлое. Назовите ЕГО словом греческого происхождения.

Ответ: Телескоп.

Комментарий: От звезд и галактик нас отделяют такие огромные расстояния, что мы наблюдаем все космические объекты в их прошлом.

Источник: К. Саган. Голубая точка. Космическое будущее человечества http://flib.nwalkr.tk/b/454689/read

Автор: Расу́л Бабазаде́ (Баку)

Средняя оценка: 1.05

Комментарии


81. Вопрос 3.41: Белинда Купер пишет, что в условиях ГДР не было большого смысла ДЕЛАТЬ ЭТО, поскольку даже в случае успеха возможности были крайне ограничены. Как следствие, уровень готовности к таким ситуациям был очень низок, поэтому в начале девяностых ДЕЛАТЬ ЭТО на востоке Германии стали гораздо чаще. Назовите тех, кто прославился, СДЕЛАВ ЭТО от десяти до пятнадцати раз с 1932 по 1934 год.

Ответ: Бонни [Паркер] и Клайд [Барроу].

Зачёт: по именам, без неверных уточнений.

Комментарий: марка ГДР была внутренней неконвертируемой валютой, ее ввоз и вывоз были запрещены. В самой ГДР было попросту негде потратить награбленное, поэтому банки грабили крайне редко. После объединения страны восточногерманские банки стали оперировать марками ФРГ и сразу привлекли грабителей, поскольку были очень слабо защищены. За Клайдом Барроу и Бонни Паркер закрепилась слава грабителей банков, хотя на самом деле их банда предпочитала грабить небольшие магазины и заправки.

Источник:
    1. https://www.ncjrs.gov/policing/fall239.htm
    2. http://www.biography.com/news/bonnie-and-clyde-9-facts-lifetime-movie-video

Автор: Максим Мозуль (Мюнхен)

Средняя оценка: 1.04

Комментарии


82. Вопрос 4.34: Уолт Дисне́й говорил, что Диснейле́нд будет расти, пока в мире существует воображение. Развивая свою мысль, он приписывает Диснейле́нду свойство, роднящее его с объектом, пожертвования на который стали собирать в 1882 году. Каким?

Ответ: Собор Святого Семейства.

Зачёт: Сагра́да Фами́лия.

Комментарий: Уолт Дисне́й считал, что Диснейленд никогда не будет достроен. Никак не могут достроить Собор Святого Семейства в Барсело́не.

Источник: http://tourist-area.com/istoriya-turizma/istoriya-disneylenda

Автор: Алексей Уланов (Баку)

Средняя оценка: 1.03

Комментарии


83. Вопрос 5.32: В статье о сооружении, расположенном неподалёку от Лас-Вегаса, говорится, что каждый день выделялось огромное количество тепла и в течение первых трёх лет можно было бы ежедневно выпекать полмиллиона буханок хлеба. Назовите двумя словами процесс, в результате которого выделялось это тепло.

Ответ: застывание бетона.

Зачёт: затвердение бетона; остывание бетона.

Комментарий: это сооружение – плотина Гувера, при строительстве которой было использовано невероятное количество бетона.

Источник: http://digital-desert.com/hoover-dam/concrete.html

Автор: Александр Кудрявцев (Николаев)

Средняя оценка: 1.03

Комментарии


84. Вопрос 7.31: В XIX веке Пруссия поспешила признать независимость Итальянского королевства, чтобы при случае у австрийцев появился ОН. Назовите ЕГО двумя словами, начинающимися на парные согласные.

Ответ: второй фронт.

Зачёт: точный ответ.

Комментарий: Итальянское королевство могло создать для Австрии войну на два фронта.

Источник: Б. Бонвеч, Ю. Галактионов. История Германии. Том 1. С древнейших времен до создания Германской империи http://flibusta.is/b/265524/read

Автор: Александр Сидоренков (Смоленск)

Средняя оценка: 1.03

Комментарии


85. Вопрос 2.27: В испанском фильме Вольфра́м фон Рихтго́фен наблюдает ИКС "ИГРЕКА". "ИКС ИГРЕКА" был создан в испаноязычной стране. Какие слова мы заменили ИКСОМ ИГРЕКА?

Ответ: полёт кондора.

Комментарий: родственники «Красного барона» также служили в авиации, Вольфрам фон Рихтгофен был начальником штаба легиона «Кондор», воевавшего в Испании. «Полёт кондора» — песня, написанная перуанским композитором Даниэлем Роблесом в 1913 году на мотив народных мелодий жителей Анд.

Источник:
    1. Х/ф «Герника», 2016 Испания
    2. https://ru.wikipedia.org/wiki/Полёт_кондора_(песня)

Автор: Денис Рыбачук (Брест)

Средняя оценка: 1

Комментарии


86. Вопрос 2.45: По одной из версий, ОН завершал бал. Кто написал его в 1794 году?

Ответ: [Михаи́л Клео́фас] Оги́нский.

Комментарий: В некоторых случаях, полонез исполнялся “на прощание”. В 1794 году было подавлено восстание Костюшко, в котором Огинский принимал участие. Композитору пришлось покинуть Речь Посполитую, и он написал знаменитый полонез "Прощание с родиной".

Источник:
    1. http://diletant.media/articles/26566828/
    2. https://ru.wikipedia.org/wiki/Полонез_Огинского

Автор: Алексей Полевой (Гомель)

Средняя оценка: 1

Комментарии


87. Вопрос 3.14: В двадцатых и тридцатых годах прошлого века во Франции разрешалось издавать книги эротического содержания, если они были на английском. Предприимчивый Джек КахЕйн, основавший свое издательство в Париже, впоследствии написал автобиографическую книгу “Мемуары ИКСА”. Слово “ИКС” одной буквой отличается от слова, вошедшего в широкое употребление примерно в то же время. Назовите ИКСА.

Ответ: буклегер.

Зачёт: booklegger.

Комментарий: Кахейну удалось издать, например “Тропик Рака” Генри Миллера. Издатель сравнивал себя с бутлегерами, которые во время Сухого закона в США занимались производством и продажей алкоголя.

Источник:
    1. https://en.wikipedia.org/wiki/Obelisk_Press
    2. https://ru.wikipedia.org/wiki/Бутлегер

Автор: Максим Коцюруба (Кишинёв)

Средняя оценка: 0.98

Комментарии (9)


88. Вопрос 3.19: [Ведущему: не акцентировать внимание команд на том, как написаны “эр” и “эс”.] Дуплет. Два вопроса по 30 секунд обсуждения, ответы сдаются на одном бланке.
    1. В придуманном автором вопроса кроссворде одно из заданий звучит так: “Современный европейский лидер на “эр”, семь букв”. Напишите ответ на это задание.
    2. В придуманном автором вопроса кроссворде одно из заданий звучит так: “Современное европейское государство на “эс”, семь букв”. Напишите ответ на это задание.

Ответ:
    1. Эрдоган; 2. Эстония.

Зачёт: точный ответ.

Комментарий: задание действительно звучит так, как указано в вопросе, хотя на самом деле имеются в виду буквосочетания “эр” и “эс”, соответственно. Семибуквенных лидеров на букву Р и семибуквенных стран на букву С в Европе сейчас нет.

Источник: http://www.picshare.ru/view/7765256/

Автор: Григорий Алхазов (Кишинёв)

Средняя оценка: 0.98

Комментарии


89. Вопрос 4.36:

Раздаточный материал

    На земле весь род людской Чтит один кумир священный, Он царит над всей вселенной, Тот кумир — телец златой!

    Перед вами отрывок куплетов Мефисто́феля из оперы "Фа́уст". По мнению литературоведа Александра Цыганко́ва, известный писатель придумал фамилию одного из своих персонажей именно благодаря этим строчкам. Назовите фамилию этого персонажа.

Ответ: Коро́вьев. Комментарии: Этот писатель - Михаил Булгаков, который и эпиграф позаимствовал из «Фауста». По мнению литературоведа Александра Цыганко́ва, член свиты Во́ланда, больше всех связанный с деньгами, получил фамилию Кор́овьев именно в качестве очередной отсылки к «Фаусту».

Источник: Экскурсия в доме-музее М.Булгакова в Москве. 02.12.2016.

Автор: Алексей Уланов (Баку)

Средняя оценка: 0.97

Комментарии


90. Вопрос 7.27: Роберт Ва́льзер пишет о своём растолстевшем персонаже, что его лицо ПРОПУСК, которыми раньше служили скулы. В конце марта 2017 года Ореховое под Рязанью тоже ПРОПУСК. Заполните пропуск тремя словами.

Ответ: вышло из берегов.

Комментарий: лицо расплылось, и никаких скул больше не видно. Озеро Ореховое вышло из берегов в результате весеннего паводка.

Источник:
    1. Вальзер Р. Браки в Филиппсбурге http://flib.nwalkr.tk/b/348960/read
    2. http://mediaryazan.ru/news/detail/381525.html

Автор: Александр Коробейников (Саратов)

Средняя оценка: 0.97

Комментарии


91. Вопрос 1.9:

Раздаточный материал

    P & O

    Одной британке приснилось, что она спасается от наводнения, забравшись на ЭТО. Психиатр объяснил сон тем, что мозг извлек из памяти название пароходной компании, которое мы вам раздали. На известном судне ЭТО, согласно ряду косвенных свидетельств, было прикреплено к полу. Назовите ЭТО.

Ответ: пианино.

Зачёт: фортепиано; рояль.

Комментарий: В подсознании пациентки название компании, прочитанное вслух бегло, сложилось в единое слово piano [пиэ́но] – “фортепиано”. Один из фактов, свидетельствующих о том, что на “Титанике” фортепиано было прикреплено к полу, – игра оркестра до последних минут, когда крен был уже сильным.

Источник:
    1. http://www.e-reading.club/chapter.php/49736/84/Rendels_-_Buri_vremeni.html
    2. http://titanicpiano.blogspot.am/2012/04/maintaining-titanics-shipboard-pianos.html

Автор: Павел Солахян (Ереван)

Средняя оценка: 0.96

Комментарии


92. Вопрос 2.16: В балтийском городе Лю́беке хорошо сохранилась средневековая архитектура, поэтому Дмитрий Заха́ров назвал город ЕЮ. Назовите ЕЁ тремя словами.

Ответ: муха в янтаре.

Комментарий: Балтийское побережье - место добычи янтаря. И архитектура хорошо сохранилась, прямо как насекомые, попавшие в янтарь

Источник: т/п «Их Нравы», т/к НТВ эфир от 21.08.2016

Автор: Денис Рыбачук (Брест)

Средняя оценка: 0.94

Комментарии


93. Вопрос 3.38: В финском языке некоторые слова образуются при соединении двух других слов. Karva [кАрва] с финского переводится как “волосы”, а madot [мАдот] — как “червяки” или “гусеницы”. У того, что финны называют словом karvamadot [кАрвамАдот], есть и другое финское название, произошедшее от фамилии одного из кавалеров ордена Белой розы Финляндии. Напишите эту фамилию.

Ответ: Брежнев.

Зачёт: точный ответ.

Комментарий: словом karvamadot, “волосатые гусеницы”, финны называют кустистые брови. Другое их финское название — brežnevit [брЕжневит]. Орден Белой розы Финляндии — одна из множества наград Леонида Ильича.

Источник:
    1. http://yle.fi/uutiset/osasto/novosti/finskii_peryeulok_69_ebenezer_dorset/7956705
    2. https://ru.wikipedia.org/wiki/Орден_Белой_розы_Финляндии

Автор: Ольга Деркач (Хельсинки)

Средняя оценка: 0.93

Комментарии


94. Вопрос 1.34: Согласно некоторым свидетельствам, один из участников расстрела Николая Гумилёва был больши́м почитателем его творчества и сохранил на всю жизнь некий предмет, принадлежавший поэту. Говоря об этом, Сергей Христовский упоминает прилагательное, образованное от названия города. Назовите этот город.

Ответ: Турин.

Комментарий: Расстрельная рубашка Николая Гумилёва стала для его преданного поклонника своего рода Туринской плащаницей.

Источник:
    1. http://new.stihi.ru/comments.html?2016/04/15/5085
    2. https://ru.wikipedia.org/wiki/Туринская_плащаница

Автор: Павел Солахян (Ереван)

Средняя оценка: 0.92

Комментарии


95. Вопрос 1.36: Героиня современного романа говорит, что теперь всё изменилось, и несколько раз повторяет слово “арба́йтен”. Назовите любое из существительных, которые фигурируют во фразе её собеседника непосредственно перед этим.

Ответ: кирхе.

Зачёт: киндер; кюхе (с незначительными отклонениями); церковь; дети; кухня; Kinder; Küche; Kirche.

Комментарий: Слово “арбайтен” переводится с немецкого как “работать”. У современной женщины нет времени на патриархальные “кирхе, киндер, кюхе” – “церковь, детей и кухню”. Лишь “работать, работать и работать”.

Источник:
    1. http://www.e-reading.club/chapter.php/1029531/21/Radi_bezopasnosti_strany.html
    2. https://ru.wikipedia.org/wiki/Kinder,_Küche,_Kirche

Автор: Павел Солахян (Ереван)

Средняя оценка: 0.92

Комментарии (1)


96. Вопрос 5.21: Туфли с парусиновым верхом и непромокаемой резиновой подошвой названы в честь предложенного британским политиком Сэ́мюэлом Пли́мсолем нововведения, также известного под названием грузовая марка. Назовите это нововведение двухкоренным словом.

Ответ: ватерлиния.

Зачёт: точный ответ.

Комментарий: Пли́мсоль предложил наносить на борта судна линию, которая предотвращала бы перегрузку судна и вызванные ею катастрофы. В туфлях-плимсолях можно ходить по лужам – главное, чтобы вода не оказывалась выше прорезиненного уровня.

Источник:
    1. https://en.wikipedia.org/wiki/Plimsoll_shoe
    2. https://ru.wikipedia.org/wiki/Ватерлиния
    3. http://www.tezlashoes.com/css/images/plimsoll3.jpg

Автор: Игорь Коршовский (Тернополь)

Средняя оценка: 0.91

Комментарии


97. Вопрос 7.23: Рассказывая про АЛЬФУ, один учёный привёл такой пример: «Стрекоза не может вернуться в воду и рассказать о надводном мире». Одну из первых статей, посвящённых АЛЬФАМ, научный журнал якобы отверг из-за непристойности термина. Назовите АЛЬФУ двумя словами.

Ответ: черная дыра.

Зачёт: точный ответ.

Комментарий: выбраться назад из черной дыры невозможно. Информация о конкретном веществе, попавшем в черную дыру, полностью уничтожается.

Источник:
    1. И. Новиков. Чёрные дыры и Вселенная http://lib100.com/book/universe/black_holes/pdf/
    2. https://bookmate.com/books/jiQS7IdG

Автор: Александр Сидоренков (Смоленск), Александр Коробейников (Саратов)

Средняя оценка: 0.91

Комментарии


98. Вопрос 7.41: В фильме Са́ймона Уи́нсера американские солдаты получают приказ доставить в дружественную вьетнамскую деревню ЕГО для совершения местного обряда. Увидев транспортировку, офицер противника предполагает, что ЕГО везут в подарок Никсону. Назовите ЕГО одним словом.

Ответ: слон.

Зачёт: точный ответ.

Комментарий: офицер делает ошибочный вывод, что слона хотят подарить республиканцу Никсону, ведь слон — символ Республиканской партии. Кстати, шахматного слона порой ошибочно называют офицером.

Источник: х/ф «Операция Дамбо», реж. С. Уинсер, 1995 г.

Автор: Александр Коробейников (Саратов)

Средняя оценка: 0.91

Комментарии


99. Вопрос 1.5: В традиции иудейских любовных стихов далёкий и недостижимый Иерусалим персонифицируется как возлюбленная. Поэтому, согласно одной версии, ОНА в произведении XIX века символизирует собой Иерусалим. Назовите ЕЁ.

Ответ: пальма.

Комментарий: Гейне родился в семье еврейского купца. В оригинале его известного стихотворения сосна (der Fichtenbaum [дер фихтенбаум]) мужского рода, а пальма (die Palme [ди пальмё]) – женского. Возможно, живший на немецком севере поэт так выражал свои чувства к далёкому Иерусалиму.

Источник: Г. Дойчер. Сквозь зеркало языка. Почему на других языках мир выглядит иначе. http://flibusta.is/b/456125/read

Автор: Анвар Мухаметкалиев (Алматы)

Средняя оценка: 0.9

Комментарии


100. Вопрос 6.5: Комментируя партию Карлсен-Карякин, гроссмейстер Сергей Шипов отметил, что только ПРОПУСК, хорош ли ход ладья e1. После чего добавил: “Центра, конечно”. Какие два слова мы пропустили?

Ответ: Вскрытие покажет.

Комментарий: Ходом Лe1 белые создали давление на центр, ослабив позицию рокировки. Выражение “вскрытие покажет” часто употребляется в случае, когда предсказать итог невозможно

Источник: Онлайн комментарий быстрых партий Карлсен-Карякин в 2016 году

Автор: Владимир Грамагин (Нью-Йорк)

Средняя оценка: 0.87

Комментарии


101. Вопрос 6.32: В пьесе на сюжет “Золушки”, содержащей аллюзии более поздних времен, мачеха убеждает принца жениться на одной из ее дочек, ссылаясь на отсутствие в стране подходящих невест. В это время паж вводит в зал Золушку. Какую фразу из трех слов он при этом произносит?

Ответ: Есть такая партия.

Комментарий: Мачеха говорит принцу: “Нет для тебя в этой стране подходящей партии”.

Источник: http://gostrov.livejournal.com/105801.html

Автор: Григорий Остров (Чикаго)

Средняя оценка: 0.87

Комментарии


102. Вопрос 7.17: В юмореске Аркадия Аверченко герои организуют профсоюз и устанавливают восьмичасовой рабочий день. За это они голосуют вставанием, так что заканчивается всё печально. Персонажи другого произведения, отмеченного Людовиком XVIII, оказались в том же положении, что и герои Аверченко. Назовите это другое произведение.

Ответ: «Плот “Медузы”».

Зачёт: точный ответ.

Комментарий: герои потерпели кораблекрушение, но вместо того, чтобы дружно грести к какому-нибудь берегу, установили восьмичасовой рабочий день, а вставая при голосовании, чуть не утопили лодку. В итоге за восемь часов в сутки они никуда не догребли, и всех съели акулы. Картина «Плот “Медузы”» была представлена на роялистском Салоне 1819 года и удостоилась внимания монарха, хотя позднее и закрепилось мнение (вероятно, справедливое) о критической направленности полотна Жерико.

Источник:
    1. Аверченко А. Т. Кипящий котёл http://dugward.ru/library/averchenko/averchenko_kipyachiy_kotel.html#015
    2. https://ru.wikipedia.org/wiki/Плот_«Медузы»

Автор: Александр Коробейников (Саратов)

Средняя оценка: 0.87

Комментарии


103. Вопрос 1.12: Если верить биографу, 13 июля 1826 года, в день казни, ЭТО СДЕЛАЛ шелест песка, брошенного в окно. Так мальчики давали друг другу понять, что происходит что-то важное. Какие два слова мы заменили словами “СДЕЛАЛ ЭТО”?

Ответ: разбудил Герцена.

Комментарий: Согласно образному выражению Ленина, “декабристы разбудили Герцена”. Если же говорить буквально, то в день казни декабристов, ставший важной вехой в становлении самосознания юного Герцена, его разбудил друг Николай Огарёв.

Источник:
    1. http://www.nesenenko.narod.ru/GZL/gertsen.pdf
    2. http://www.bibliotekar.ru/encSlov/19/40.htm

Автор: Павел Солахян (Ереван)

Средняя оценка: 0.85

Комментарии (1)


104. Вопрос 3.44: По словам ХАральда Дроста, прежде чем совать нос в проблемы других людей, менеджер должен сначала позаботиться о себе. При этом Дрост упоминает ЕЁ. Назовите ЕЁ двумя словами.

Ответ: кислородная маска.

Зачёт: точный ответ.

Комментарий: Дрост цитирует инструкцию по безопасности в самолёте, которая предписывает сначала надеть кислородную маску на себя, а затем заботиться о других. “Совать нос” — небольшая подсказка.

Источник: Лекция Х. Дроста Managing People, 21.06.2016

Автор: Николай Лёгенький (Амстердам)

Средняя оценка: 0.85

Комментарии (1)


105. Вопрос 5.20: Широкие научные дискуссии об АЛЬФЕ начались ещё в семидесятые годы. Однако один из первых исторических анализов феномена был проведён лишь в 1992 году в работе Но́а Ми́лграма «АЛЬФА: болезнь современности». Назовите АЛЬФУ.

Ответ: прокрастинация.

Зачёт: точный ответ.

Комментарий: такая медлительность вполне в духе самого исследуемого явления.

Источник: http://scisne.net/a-410

Автор: Александр Мудрый (Черновцы)

Средняя оценка: 0.84

Комментарии


106. Вопрос 5.36: В «Моби Дике» Герман Мелвилл называет И́КСОВЫМИ Нельсона в Лондоне, Наполеона в Париже и Ва́шингтона в Ба́лтиморе. Антонио Кано́ва изобразил Наполеона в образе И́КСА. Какие три буквы мы дважды заменили двумя другими в тексте вопроса?

Ответ: мар.

Зачёт: данные буквы в любом порядке.

Комментарий: в 1851 году, когда был издан роман «Моби Дик», всех троих уже не было в живых. Памятники Нельсону, Наполеону и Вашингтону стоят на высоких колоннах, из-за чего изображённые напоминают марсовых матросов – специалистов по такелажным работам.

Источник:
    1. Г. Мелвіл. Мобі Дік, або Білий Кит. – К.: Дніпро, 1984, с. 182
    2. http://en.wikipedia.org/wiki/Napoleon_as_Mars_the_Peacemaker
    3. http://dic.academic.ru/dic.nsf/ushakov/856106

Автор: Игорь Коршовский (Тернополь)

Средняя оценка: 0.84

Комментарии


107. Вопрос 7.39: В районе пирамид Ту́куме в Перу католическая церковь проводила массовые сожжения язычников. Поэтому этот район также называют ВТОРОЕ. Какое слово мы заменили ВТОРЫМ?

Ответ: чистилище.

Зачёт: точный ответ.

Комментарий: очищали огнём.

Источник:
    1. http://www.vokrugsveta.ru/vs/article/664/
    2. https://en.wikipedia.org/wiki/Túcume

Автор: Александр Коробейников (Саратов)

Средняя оценка: 0.84

Комментарии


108. Вопрос 1.2: Своё прозвище “ТАКОЙ Климт” художник Э́гон Ши́ле получил не только за используемый материал, но и за подражание Густаву. Другой “ТАКОЙ” стал убийцей в пять лет. Какое слово мы заменили на ТАКОЙ?

Ответ: Серебряный.

Комментарий: Климт известен использованием сусального золота при создании самых известных своих работ. Шиле же считал себя последователем Климта, и потому даже прилюдно называл себя “серебряным Климтом”, и использовал серебро в своей живописи. Жеребцу по кличке Серебряный из рассказа Конан Дойла на момент действия рассказа было пять лет.

Источник:
    1. http://www.neuegalerie.org/collection/curators-choice-june-2011
    2. http://www.e-reading.club/chapter.php/28809/1/Konan_Doiil_4_Zapiski_o_Sherloke_Holmse.html

Автор: Анвар Мухаметкалиев (Алматы)

Средняя оценка: 0.83

Комментарии (1)


109. Вопрос 7.22: Михаил Таль так и не эмигрировал из СССР. На все предложения уехать он отвечал: «У меня в Риге очень большое ОНО». Назовите ЕГО одним словом.

Ответ: кладбище.

Зачёт: точный ответ.

Комментарий: Таль не хотел покидать могилы предков.

Источник: http://www.euruchess.org/cgi-bin/index.cgi?action=viewnews&id=1539

Автор: Александр Коробейников (Саратов)

Средняя оценка: 0.81

Комментарии


110. Вопрос 6.22: По словам историка Самуила Лурье, Петербург невелик и со всех сторон окружен ИКСОМ. Кто создал “ИКС” в середине 1990-х годов?

Ответ: [Сергей] Шнуров.

Зачёт: Шнур; [Игорь] Вдовин.

Комментарий: Небольшая историческая часть Петербурга окружена новостройками. Рок-группа “Ленинград” была создана по разным версиям в 1996 или 1997 году.

Источник: http://www.litkarta.ru/dossier/mne-ne-nravitsia/view_print/ https://ru.wikipedia.org/wiki/Лурье,_Самуил_Аронович https://ru.wikipedia.org/wiki/Ленинград_(группа)

Автор: Григорий Остров (Чикаго)

Средняя оценка: 0.79

Комментарии


111. Вопрос 6.25: Израильская Академия иврита изобретает ивритские замены для заимствованных слов, таких, как “лоукост” или “мегаполис”. Какое слово один русскоязычный блоггер предложил заменить на “лимудо́ния”, от слова “лиму́д” – “обучение”?

Ответ: Академия.

Комментарий: Для собственного названия академия по-прежнему использует заимствованное слово. Источник:https://www.facebook.com/ralph.kent.5832/posts/1805705546368944

Автор: Григорий Остров (Чикаго)

Средняя оценка: 0.79

Комментарии


112. Вопрос 3.2: Согласно одной статье, самое сложное при переходе к здоровому образу жизни — это не бросить всё ПРОПУСК. Считается, что слова “ПРОПУСК” появились в законе для того, чтобы некое событие не выпадало на День всех святых. Заполните пропуск тремя словами, которые начинаются на одну и ту же букву.

Ответ: после первого понедельника.

Зачёт: точный ответ.

Комментарий: выборы президента США проводятся в первый вторник после первого понедельника, чтобы не было вероятности совпадения с 1 ноября, Днем всех святых. Люди часто выбирают первый день недели, чтобы начать новую жизнь, но нередко бросают эту затею после первого понедельника.

Источник:
    1. http://www.fashiontime.ru/beauty/news/1281958.html
    2. http://uselections.com/m/answers/view/Why-are-elections-held-on-a-Tuesday-in-November

Автор: Максим Коцюруба (Кишинёв)

Средняя оценка: 0.78

Комментарии


113. Вопрос 4.20: В Вене считают: через ЭТО нужно суметь прочитать газету. Назовите ЭТО тремя словами.

Ответ: Тесто для штру́деля

Зачёт: Корж для штру́деля

Комментарий: Таким тонким должен быть каждый из слоев теста, из которого выпекается венский штру́дель.

Источник: Петр Вайль. Фильм "Гений места. Вена-Малер".

Автор: Алексе́й Ула́нов (Баку)

Средняя оценка: 0.77

Комментарии


114. Вопрос 5.1:

Раздаточный материал

    Heaveno

    Вместо какого слова жители одного американского городка используют то, что напечатано на раздаточном материале?

Ответ: Hello.

Зачёт: Хэллоу.

Комментарий: жители одного техасского городка решили, что не стоит начинать день с упоминания ада, поэтому в качестве официального приветствия ввели слово «хэвэно́у».

Источник: http://www.mndaily.com/article/1997/01/texas-town-says-goodbye-hello

Автор: Александр Кудрявцев (Николаев)

Средняя оценка: 0.77

Комментарии


115. Вопрос 5.37: Сервантес пишет, что некий бесталанный живописец, нарисовав однажды петуха, сделал ещё кое-что. Схожее действие, но с противоположным смыслом, больше трёх веков спустя сделал другой человек. Назовите этого человека.

Ответ: Рене́ Магри́тт.

Зачёт: по фамилии.

Комментарий: «нарисовал он однажды петуха, да так скверно и до того непохоже, что пришлось написать под ним крупными буквами: "Это петух"». На знаменитой картине «Вероломство образов» под изображением курительной трубки есть надпись «Это не трубка».

Источник:
    1. http://www.lib.ru/INOOLD/SERVANTES/donkihot2.txt
    2. https://ru.wikipedia.org/wiki/Вероломство_образов

Автор: Александр Мудрый (Черновцы)

Средняя оценка: 0.77

Комментарии


116. Вопрос 5.42: В 1912 году ОН устроился в петербургскую лабораторию, в которой разрабатывал способы очистки водки от сивушных масел. Назовите ЕГО самое знаменитое детище.

Ответ: угольный противогаз.

Зачёт: противогаз.

Комментарий: ОН – химик Николай Зелинский. Водку очищали с помощью древесного угля. Это навело Зелинского на мысль использовать древесный уголь в противогазах.

Источник: http://supotnitskiy.ru/book/book5_kommentarii44.htm

Автор: Александр Кудрявцев (Николаев)

Средняя оценка: 0.77

Комментарии


117. Вопрос 7.15: Профессиональный спорщик Амари́лло Слим однажды поставил на то, что обгонит на стоярдовой дистанции скаковую лошадь, и действительно победил. Дело в том, что бежали ПРОПУСК. Пропущенные слова присутствуют в заглавии известного произведения. Назовите того, кто также присутствует в этом заглавии.

Ответ: хоббит.

Зачёт: точный ответ.

Комментарий: Амарилло Слим заметил, что лошади очень медленно останавливаются, поэтому разделил дистанцию на два отрезка — 50 ярдов туда и 50 ярдов обратно.

Источник:
    1. http://www.independent.co.uk/news/world/americas/one-of-a-kind-legendary-gambler-amarillo-slim-has-played-his-final-hand-7697721.html
    2. https://ru.wikipedia.org/wiki/Хоббит,_или_Туда_и_обратно

Автор: Мария Колосовская (Москва)

Средняя оценка: 0.77

Комментарии


118. Вопрос 7.35: Мария Павловна родилась в 1863 году. Какими двумя словами назвал её автор вопроса?

Ответ: сестра таланта.

Зачёт: точный ответ.

Комментарий: сам брат-талант назвал так краткость.

Источник: https://ru.wikipedia.org/wiki/Чехова,_Мария_Павловна

Автор: Александр Коробейников (Саратов), Сергей Родиков (Новороссийск)

Средняя оценка: 0.77

Комментарии


119. Вопрос 1.22: Все попытки Гарри Каспарова поставить мат Анатолию Карпову долгое время не приносили успеха. “Я позволил себе вступить во владения ИКСА”, – говорил Каспаров. В одном анекдоте ИКС, находясь в больнице, отказывался пользоваться известным приспособлением. Назовите ИКСА.

Ответ: Кащей [Бессмертный].

Зачёт: Кощей [Бессмертный].

Комментарий: “Мат” в переводе означает “умер”. Анатолий Карпов долгое время был, образно выражаясь, бессмертен для Гарри Каспарова. В анекдоте Кащей Бессмертный боялся воспользоваться уткой – дескать, в утке яйцо, в яйце – игла, а на конце иглы – его смерть.

Источник:
    1. http://www.e-reading.club/chapter.php/1004822/117/Horoshevskiy_Andrey_-_100_znamenityh_simvolov_sovetskoy_epohi.html
    2. http://anekdoty.ru/pro-utku/page/3/

Автор: Павел Солахян (Ереван)

Средняя оценка: 0.74

Комментарии


120. Вопрос 2.10: В конце 1980-х Бе́ар Гриллс сдал четыре вступительных экзамена с не лучшим средним баллом, но результат ему понравился. Напишите полученные им оценки в правильном порядке.

Ответ: ACDC.

Комментарий: в Великобритании принята буквенная шкала оценки знаний, в которой высшей оценкой является А. В юности Беару Гриллсу нравилась группа AC/DC [эйси диси].

Источник: Беар Гриллс «Грязь, пот и слёзы» http://avidreaders.ru/book/gryaz-pot-i-slezy.html

Автор: Денис Рыбачук (Брест)

Средняя оценка: 0.74

Комментарии


121. Вопрос 6.43: Внимание, в вопросе заменено одно слово. Герои цикла произведений уставились 12 августа, 20 сентября, 24 ноября и 1 июля. Назовите имя любого из этих героев.

Ответ: Леонардо.

Зачёт: Микеланджело, Донателло, Рафаэль.

Комментарий: Черепашки-ниндзя вылупились в указанные даты. Интересно, что красноухие черепахи действительно вылупляются летом и осенью.

Источник: https://ru.wikipedia.org/wiki/Черепашки-ниндзя

Автор: Владимир Грамагин (Нью-Йорк)

Средняя оценка: 0.74

Комментарии


122. Вопрос 4.13: Молодой человек из романа Агаты Кристи влюбляется с первого взгляда в девушку, сразу забыв свою предыдущую любовь к ее сестре. Имена того, с кем сравнивается этот юноша, и той, с кем сравнивается отвергнутая девушка, начинаются на одни и те же две буквы. Напишите эти имена.

Ответ: Роме́о, Розали́нда.

Комментарий: История напоминает классический сюжет, в котором Ромео направляется на бал для встречи с Розали́ндой, но встречает там Джульетту. Розалинда и Джульетта тоже были родственницами.

Источник: А. Кристи. День поминовения. http://flib.nwalkr.tk/b/294129/read

Автор: Алексе́й Ула́нов (Баку)

Средняя оценка: 0.71

Комментарии


123. Вопрос 6.1: Нью-йоркский боулинг-клуб удачно расположен в центре Манхэттена. Правда, курение в клубе запрещено. А как он называется?

Ответ: Lucky Strike.

Комментарий: Редакторы желают командам удачи. Выбивайте страйк в каждом вопросе!

Источник: http://www.bowlluckystrike.com/locations/new-york/

Автор: Владимир Грамагин (Нью-Йорк)

Средняя оценка: 0.7

Комментарии


124. Вопрос 6.8: Один из друзей профессора, комментируя огромную скорость распространения кибернетики, шутил, что “люди будто заразились ТАКОЙ болезнью”. Ответьте добуквенно точно – какой?

Ответ: Винерической.

Зачёт: Виннерической; Wienereal.

Комментарий: Комментируя новое учение Норберта Винера, один из его друзей заметил, что “люди будто заразились винерической болезнью”.

Источник: http://trv-science.ru/2014/08/12/nauchnye-shutki/

Автор: Туриан Мамедов (Чикаго)

Средняя оценка: 0.7

Комментарии (1)


125. Вопрос 6.13: В статье, озаглавленной “АЛЬФЫ истории”, Михаил Френкель напоминает, что злодеяния гитлеризма начались с преследования евреев. АЛЬФА – близкая родственница чечётки. Назовите АЛЬФУ.

Ответ: Канарейка.

Комментарий: Так же, как канарейки первыми чувствуют рудничный газ в шахте, евреи обычно первыми испытывают на себе преследования в обществе, лишенном демократических свобод. Чечетка – птица отряда воробьинообразных, является близким родственником канарейки..

Источник: http://www.isrageo.com/2016/06/21/frenk153/ https://ru.wikipedia.org/wiki/Домашняя_канарейка https://ru.wikipedia.org/wiki/Чечётка_(птица)

Автор: Мара Могилевская (Чикаго)

Средняя оценка: 0.7

Комментарии


126. Вопрос 6.17: Лекарей, врачевавших раны и болячки на восточных базарах, называли словосочетанием “мир макасо́н”, что означает “отец лжи”. Какие два слова мы заменили на “отец лжи”?

Ответ: Повелитель мух.

Зачёт: Властелин мух, эмир мух, царь мух и т.п.

Комментарий: Над ранами в изобилии вились мухи.”Отец лжи”, как и “повелитель мух” – эпитет дьявола. Источник: http://medportal.ru/mednovosti/news/2017/02/02/281rishta/?tomobile

Автор: Григорий Остров (Чикаго)

Средняя оценка: 0.7

Комментарии


127. Вопрос 2.12: После первого издания ЕЁ обладателям досылались дополнительные листы. Яна Шеля́кина пишет, что Советский Союз присваивал себе заслугу в ЕЁ появлении. Назовите ЕЁ.

Ответ: Международной красной книги. Зачёт: красной книги МСОП, красной книги.

Комментарий: Листы досылались, так как в нее попадали все новые животные. Слово “красный” - знак опасности, а вовсе не то, что она была сделана по инициативе СССР.

Источник:
    1. https://ru.wikipedia.org/wiki/Красная_книга
    2. http://xn----dtbjalal8asil4g8c.xn--p1ai/knigi/istoriya-krasnoy-knigi.html

Автор: Алексей Полевой (Гомель)

Средняя оценка: 0.67

Комментарии


128. Вопрос 4.9:

Раздаточный материал

    В журнале "GEO" можно прочесть статью "Как стать миллиардером" и интервью с основателем фирмы Nike Филом Найтом.

    В раздаточный материал мы добавили пару штрихов. Восстановите исходное слово.

Ответ: CEO [Ведущему: се́о].

Комментарий: Журнал называется не GEO, а СЕО. Chief Executive Officer [Чиф Икзэ́кьютиф О́ффисер] дословно — главный исполнительный директор, и в журнале с таким названием логично появление интервью с основателем всемирно известной компанией. При помощи пары штрихов буква "С" [си] превратилась в букву "G" [джи].

Источник: http://appleinsider.ru/obzory-prilozhenij/reklamaapp-store-hd-zhurnal-ceo-kursy-povysheniya-klassifikacii-dlya-kazhdogo-menedzhera.html

Автор: Эльну́р Гасымзаде́ (Баку)

Средняя оценка: 0.67

Комментарии


129. Вопрос 5.38: Однажды британский линкор «Нельсон» лёг на грунт неподалёку от По́ртсмута, что породило ряд анекдотов. Напишите слово, являвшееся названием мели, на которой это произошло.

Ответ: Гамильтон.

Зачёт: Хэмилтон; Гамилтон; Хэмильтон; Hamilton.

Комментарий: острословов забавлял тот факт, что Нельсон снова оказался на Гамильтон.

Источник:
    1. http://www.portsmouth.co.uk/heritage/right-m-lads-everybody-aft-and-on-the-count-of-three-start-jumping-1-6496828
    2. https://www.youtube.com/watch?v=4LS129lkvS8

Автор: Олег Полыняк (Черновцы)

Средняя оценка: 0.67

Комментарии


130. Вопрос 7.2: Сергей Рачи́нский, герой картины «Устный счёт в народной школе», придерживался гуманных методов воспитания. Так, первому школьнику, который решит поставленную арифметическую задачу, он выдавал ИКС. Назовите ИКС.

Ответ: пряник.

Зачёт: точный ответ.

Источник: http://oppps.ru/o-chem-govorit-kartina-ustnyj-schet-v-narodnoj-shkole.html

Автор: Сергей Шорин (Краснодар)

Средняя оценка: 0.67

Комментарии


131. Вопрос 7.10: Экономист Ха-Джун Чхан называет богатые государства и организации, которые вместо помощи бедным странам дают им дурные советы, злыми ИКСАМИ. ИКСЫ действительно пользовались недоброй славой. Назовите ИКСОВ.

Ответ: самаритяне.

Зачёт: точный ответ.

Комментарий: добрый самаритянин помог оказавшемуся в беде иудею, несмотря на религиозные различия. Злые же самаритяне никому не помогают, а только разглагольствуют (а также стремятся проникнуть на рынок развивающихся стран).

Источник:
    1. http://left.ru/2011/1/nebogatov203.phtml
    2. Ха Джун Чхан. 23 тайны: то, что вам не расскажут про капитализм http://coollib.com/b/317695/read
    3. https://ru.wikipedia.org/wiki/Самаритяне
    4. https://en.wikipedia.org/wiki/Parable_of_the_Good_Samaritan

Автор: Александр Коробейников (Саратов)

Средняя оценка: 0.67

Комментарии


132. Вопрос 7.12: [Ведущему: не акцентировать тот факт, что «порно фильмы» написаны раздельно.] Театральная актриса Мария Фомина признаётся, что терпеть не может так называемые порно фильмы и сниматься в них не собирается. В одном из слов предыдущего предложения мы пропустили шесть букв. Напишите его в исходном виде.

Ответ: попкорновые.

Зачёт: точный ответ.

Комментарий: масс-культура актрисе неинтересна, она предпочитает авторское кино.

Источник: https://lenta.ru/articles/2017/03/13/fomina/

Автор: Александр Коробейников (Саратов)

Средняя оценка: 0.67

Комментарии


133. Вопрос 7.24: В середине XX века компания General Motors [дже́нерал мо́торс] выпускала «турлайнеры» - автобусы, с помощью которых демонстрировались стереозвук, микроволновая печь, телевидение. Какие буквы мы пропустили в этом вопросе?

Ответ: фу.

Зачёт: fu.

Комментарий: «Футурлайнеры» должны были продемонстрировать, что будущее уже наступило.

Источник: https://en.wikipedia.org/wiki/GM_Futurliner

Автор: Сергей Шорин (Краснодар)

Средняя оценка: 0.67

Комментарии


134. Вопрос 3.28: В 2010 году в Шанхае появилась копия знаменитой скульптуры. Автор обеих скульптур отмечает, что по сравнению с оригиналом шанхайский ИКС более молодой, сильный и ТАКОЙ. На эмблеме какого бренда присутствуют два ТАКИХ ИКСА?

Ответ: Red Bull.

Зачёт: Ред Булл.

Комментарий: ТАКОЙ ИКС — красный бык. 2010 был годом Быка по восточному календарю, поэтому китайские власти заказали Артуро ди МОдика, автору скульптуры “Атакующий бык” на Уолл-стрит, копию, цвет которой тоже был выбран неслучайно. На эмблеме энергетика Red Bull изображены два красных быка, которые атакуют друг друга.

Источник:
    1. https://en.wikipedia.org/wiki/Bund_Bull
    2. https://en.wikipedia.org/wiki/Red_Bull

Автор: Максим Коцюруба (Кишинёв)

Средняя оценка: 0.66

Комментарии


135. Вопрос 3.10: Одно из прозвищ этого американского города противопоставляет его Нью-Йорку. Назовите этот город.

Ответ: Миннеаполис.

Зачёт: точный ответ.

Комментарий: Миннеаполис носит прозвище "Mini-apple" — мини-яблоко. Нью-Йорк называют "Большим яблоком"

Источник:
    1. https://en.wikipedia.org/wiki/Minneapolis
    2. https://en.wikipedia.org/wiki/Big_Apple

Автор: Максим Коцюруба (Кишинёв)

Средняя оценка: 0.64

Комментарии (3)


136. Вопрос 4.29: Василий Уткин рассказывает, как однажды его очень обрадовал телефонный звонок, ведь он в то время находился между третьим и четвертым... Чем?

Ответ: Этажами.

Комментарий: Известно, что Василий Уткин страдает от лишнего веса и подъем по лестницам дается ему нелегко. Звонок позволил сделать перерыв в подъеме, будто не из-за лишнего веса. Можно сказать, что мы продолжили тему альпинизма.

Источник: http://www.sports.ru/tribuna/blogs/soulkitchen/895927.html

Автор: Алексей Уланов (Баку)

Средняя оценка: 0.64

Комментарии


137. Вопрос 1.1: В 13-м веке жители замка Тра́смос пустили слух, будто бы в нём живут ведьмы и маги, которые по ночам трясут цепями и куют котлы для зе́лий. На самом деле обитатели замка были ИМИ. Назовите ИХ сложным словом.

Ответ: фальшивомонетчики.

Комментарий: Жители замка не хотели, чтобы обитатели деревни совали свой нос в их дела, и объясняли звуки ковки монет существованием ведьм.

Источник: http://www.bbc.com/travel/story/20160418-spains-cursed-village-of-witches

Автор: Анвар Мухаметкалиев (Алматы)

Средняя оценка: 0.63

Комментарии (1)


138. Вопрос 1.11: На советской карикатуре военного времени благодаря русским снайперам отель в аду работает в усиленном режиме. Какие буквы мы пропустили в предыдущем предложении?

Ответ: к, н, а, я.

Комментарий: Для убитых немецких солдат приходится разогревать все новые и новые котлы в адской котельной.

Источник: http://www.brainparking.com/view/topic.cfm?key_or=1055413&lenta_type=4

Автор: Анвар Мухаметкалиев (Алматы)

Средняя оценка: 0.61

Комментарии


139. Вопрос 2.4: В первые герметичные кабины советских самолётов воздух нагнетался от двигателя. При испытании такого самолета Александр Чеса́лов взял в полет ЕЁ. Назовите ЕЕ.

Ответ: канарейка.

Комментарий: наддув от двигателя мог привести к попаданию в кабину угарного газа. Так как индикационных приборов для обнаружения окиси углерода в кабине не было, решили воспользоваться старым шахтёрским способом.

Источник: Михаил Арлазоров «Лавочкин» http://www.lib.ru/MEMUARY/ZHZL/lawochkin.txt

Автор: Денис Рыбачук (Брест)

Средняя оценка: 0.61

Комментарии


140. Вопрос 2.31: На одной из гоночных трасс есть поворот, выйти из которого достаточно сложно, ведь он характеризуется не только движениями вправо и влево, но и перепадом высоты. Как он называется?

Ответ: штопор. Зачёт: corkscrew.

Комментарий: выйти из штопора сложно не только авиапилоту, но и водителю на трассе Лагуна Сека в Калифорнии.

Источник: https://ru.wikipedia.org/wiki/Лагуна_Сека

Автор: Алексей Полевой (Гомель)

Средняя оценка: 0.61

Комментарии


141. Вопрос 3.15:

Раздаточный материал

    

    Перед вами фрагмент минималистического постера к американскому фильму. Какому?

Ответ: “Девять с половиной недель”.

Зачёт: 9½ Weeks.

Комментарий: дом на постере стилизован под календарь — десять этажей по семь окон, свет горит во всех окнах, кроме трех последних. Фигурка в окне — герой Микки Рурка, фигурка на улице — героиня Ким Бейсингер, уходящая от него по прошествии девяти с половиной недель.

Источник: https://flyergoodness.blogspot.com/2011/11/pictogram-posters-by-viktor-hertz.html

Автор: Адриан Макарь (Кишинёв)

Средняя оценка: 0.61

Комментарии


142. Вопрос 5.4: На этом американском плакате мы заретушировали бородку ключа. Изобразите то, в виде чего она выполнена.

Ответ: АА.

Зачёт: точный ответ.

Комментарий: это реклама общества «Анонимных алкоголиков».

Источник: http://www.the-twelve-steps.com/aa-na-symbols-aa-na-logos-art.html

Автор: Александр Кудрявцев (Николаев)

Средняя оценка: 0.61

Комментарии


143. Вопрос 5.27: 5 августа 2016 года 71-летняя бразильянка приняла участие в эстафете олимпийского огня, пронеся факел по НЕЙ. Назовите ЕЁ.

Ответ: Ипане́ма.

Зачёт: точный ответ.

Комментарий: в августе прошлого года Олимпиаду принимал Рио-де-Жанейро, одним из районов которого является Ипанема. Огонь несла та самая Элоиза Пине́йру, которая в 1962 году вдохновила Жоби́ма и ди Мора́йша на создание легендарной песни «Девушка из Ипане́мы».

Источник:
    1. http://edition.cnn.com/2016/08/05/world/girl-from-ipanema-rio-olympics/
    2. http://www.daikhlo.com/watch/yt_UuPdSTh-idk

Автор: Александр Мудрый (Черновцы)

Средняя оценка: 0.61

Комментарии


144. Вопрос 7.3: В историческом романе утверждается, что на самом деле ОНА предпочитала врагов сжигать. Напишите ЕЁ прозвище и имя.

Ответ: Кровавая Мэри.

Зачёт: Кровавая Мария.

Комментарий: Мария I Тюдор была ревностной католичкой и любила сжигать еретико́в — наказывая их мягко, без пролития крови.

Источник: Майлз Р. Я, Елизавета http://flib.nwalkr.tk/b/69270/read

Автор: Александр Коробейников (Саратов)

Средняя оценка: 0.61

Комментарии


145. Вопрос 7.11: Гордон Ругг попытался получить аналог ЭТОГО при помощи таблицы из трёх столбцов и доказать, что ЭТО — мистификация. Назовите фамилию человека, завладевшего ЭТИМ в 1912 году.

Ответ: Войнич.

Зачёт: точный ответ.

Комментарий: Ругг решил доказать, что манускрипт Войнича не несёт никакого смысла, а текст с похожими характеристиками можно получить, комбинируя суффиксы, префиксы и корни. Вильфрид Войнич в 1912 году обнаружил и купил в Риме рукопись, ныне носящую его имя.

Источник: https://ru.wikipedia.org/wiki/Рукопись_Войнича

Автор: Александр Коробейников (Саратов)

Средняя оценка: 0.61

Комментарии


146. Вопрос 7.37: Шестнадцатилетний Юра Рябинкин стал вести дневник в блокадном Ленинграде. Объясняя смысл этого, Юра писал, что когда-нибудь дневник поможет ответить на литературный вопрос утвердительно. Напишите этот вопрос.

Ответ: «[А] был ли мальчик?»

Комментарий: Юра хотел, чтобы о его существовании помнили, так как не надеялся пережить блокаду. Зимой 1942 года он действительно умер. А дневник его жив.

Источник: https://gorky.media/context/zashla-k-ahmatovoj-ona-zhivet-u-dvornika-ubitogo-artsnaryadom/

Автор: Александр Коробейников (Саратов)

Средняя оценка: 0.61

Комментарии


147. Вопрос 7.40: В XVI веке портовая Севилья имела для Испании огромное значение. Хуан Лалагу́на называет Севилью того времени ИМ. Назовите ЕГО тремя словами.

Ответ: окно в Америку.

Зачёт: точный ответ.

Источник: Х. Лалагуна. Испания. История страны http://flibusta.is/b/332490/read

Автор: Александр Сидоренков (Смоленск)

Средняя оценка: 0.61

Комментарии


148. Вопрос 7.33: Кирилл Зубков приводит народное поверье о том, что в некий день работать нельзя, иначе небо может покарать. О каком произведении он рассказывает?

Ответ: «Обломов».

Зачёт: точный ответ.

Комментарий: работающих в Ильин день может убить громом. Неслучайно, видимо, ничего не делающего Обломова зовут Илья Ильич.

Источник: http://arzamas.academy/mag/395-oblomov

Автор: Александр Коробейников (Саратов)

Средняя оценка: 0.57

Комментарии (4)


149. Вопрос 6.3: В 2016 году российская армия вдвое увеличила закупки ТАКИХ изделий. Одно из них, называемое Т-80, весит около 70 килограммов, сто́ит порядка 16 тысяч долларов и поставляется упакованным в мешки. Какое слово с приставкой мы заменили на “ТАКОЕ”?

Ответ: Надувное.

Комментарий: Надувные имитации танков и другой боевой техники используются для дезориентации противника.

Источник: https://www.gazeta.ru/army/news/9219227.shtml http://www.trud.ru/article/06-02-2017/1346713_minoborony_uvelichilo_zakupki_naduvnyx_tankov_i_samoletov.html

Автор: Елена Лукаш (Чикаго)

Средняя оценка: 0.56

Комментарии


150. Вопрос 6.40: Издательство О’Рейли выпускает серию книг на компьютерные темы, на обложках которых изображены различные животные. На обложке книги, посвященной ЭТОМУ, нарисована свинья. ЭТО – название одной из картин Энди Уорхола. Назовите ЭТО.

Ответ: Спам.

Комментарий: Свинья – самое правильное животное для обозначения спамеров. Да и слово пошло от названия свиных консервов. У Энди Уорхола кроме серии “Суп Кэмпбелл” есть и похожая серия “Спам”.

Источник: http://www.amazon.com/Stopping-Spam-Stamping-Unwanted-Postings/dp/156592388X/ref=sr_1_6?ie=UTF8&qid=1465094749&sr=8-6&keywords=o%27reilly+spam http://www.oreilly.com/animals.html http://www.wikiart.org/en/andy-warhol/spam

Автор: Александр Сульповар (Сент-Луис)

Средняя оценка: 0.56

Комментарии


151. Вопрос 1.16: В этом вопросе ИКС является заменой. Рассуждая о слове “симме́трия”, один интернет-пользователь разочарованно заметил, что оно не является ИКСОМ. Какой ИКС есть в тексте этого вопроса?

Ответ: оно.

Комментарий: Действительно, было бы символично, если бы само слово “симметрия” было палиндромом.

Источник: https://www.reddit.com/r/Showerthoughts/comments/4peb07/i_wish_the_word_symmetry_was_a_palindrome/

Автор: Анвар Мухаметкалиев (Алматы)

Средняя оценка: 0.55

Комментарии


152. Вопрос 1.39: Воспользовавшись интересом англичан викторианской эпохи к определённой дате, Ричард Кэ́дбери стал продавать шоколад в НЕЙ. У известного человека были трудные отношения с женой, и, возможно, поэтому в произведении 1993 года он говорил, что заперт в НЕЙ. Назовите ЕЕ четырьмя словами.

Ответ: коробка в виде сердца.

Зачёт: шкатулка в виде сердца, коробка в форме сердца, шкатулка в форме сердца.

Комментарий: В викторианской Англии очень увлекались Днём святого Валентина, и Кэдбери решил сыграть на этом. В песне “Heart-Shaped Box” [Хартшэйпд бокс] Кобейн, как считают многие, отразил свои сложные отношения с женой Кортни Лав.

Источник:
    1. https://thechocolatejournalist.com/first-sold-chocolate-valentines-day/
    2. http://www.history.com/news/hungry-history/celebrating-valentines-day-with-a-box-of-chocolates
    3. http://genius.com/Nirvana-heart-shaped-box-lyrics

Автор: Анвар Мухаметкалиев (Алматы)

Средняя оценка: 0.55

Комментарии


153. Вопрос 1.41: Увидев Ро́на Му́ди в роли диккенсовского Фе́джина – скуповатого пожилого человека, знавшего о попрошайничестве не понаслышке, другой режиссёр решил, что нашёл своего ИКСА. Напишите название животного, от которого произошла фамилия “ИКС”.

Ответ: воробей.

Комментарий: Так Мэл Брукс нашёл актёра для роли Кисы Воробьянинова в своей экранизации “Двенадцати стульев”.

Источник:
    1. http://www.tcm.com/this-month/article.html?id=627019%7C627025
    2. http://lit-helper.com/p_Priklyucheniya_Olivera_Tvista_harakteristika_obraza_Fedjin
    3. http://lib.ru/INPROZ/DIKKENS/oliver.txt

Автор: Павел Солахян (Ереван)

Средняя оценка: 0.55

Комментарии


154. Вопрос 2.1: [Ведущему: кавычки не озвучивать.] Что Виктория Аза́ренко, по её словам, сделала благодаря "Друзьям"?

Ответ: выучила английский язык.

Зачёт: по слову “английский” без неверных уточнений.

Комментарий: Виктория Азаренко давно живет не в Беларуси, а в США. Сериал “Друзья” она очень любит и говорит, что именно благодаря просмотру на языке оригинала освоила английский

Источник: http://www.pressball.by/news/tennis/243958

Автор: Алексей Полевой (Гомель)

Средняя оценка: 0.54

Комментарии


155. Вопрос 2.14: Изобретения нередко губят своих создателей. Не стал исключением и один портной. Что он испытывал в Париже в 1912 году?

Ответ: плащ-парашют. Зачёт: парашют.

Комментарий: Франц Ра́йхельт добился от парижской префектуры специального разрешения спрыгнуть с Эйфелевой башни. Изобретатель разбился насмерть, так как его плащ-парашют не раскрылся

Источник: https://ru.wikipedia.org/wiki/Райхельт,_Франц

Автор: Алексей Полевой (Гомель)

Средняя оценка: 0.54

Комментарии


156. Вопрос 2.28: План парка у дворца Шёнбрунн начерчен высокопоставленным лицом. Что по одной из теорий символизируют две диагональные аллеи, сходящиеся перед главным входом во дворец?

Ответ: циркуль.

Комментарий: циркуль - символ масонства. Парк спроектировал император Франц I Стефан, член одной из масонских лож.

Источник: д/ф «Люди, мифы и легенды» (8 серий из 12) / Menschen, Mythen und Legenden, 2014 – 1 серия http://tfile.co/forum/viewtopic.php?t=843843

Автор: Денис Рыбачук (Брест)

Средняя оценка: 0.54

Комментарии


157. Вопрос 2.29: C вершины Эвереста Бе́ар Гриллс привез домой бутылку. Через несколько лет её содержимое было использовано для сына Гриллса. Какое слово в вопросе мы пропустили?

Ответ: крещения.

Комментарий: С вершины Эвереста альпинист зачерпнул в бутылку снега. По пути домой снег растаял и когда у Беара родился сын, то во время церемонии крещения была использована вода «с вершины мира».

Источник: Беар Гриллс «Грязь, пот и слёзы» http://avidreaders.ru/book/gryaz-pot-i-slezy.html

Автор: Денис Рыбачук (Брест)

Средняя оценка: 0.54

Комментарии (1)


158. Вопрос 2.40: Узнав о трагедии в США, Филипп выразил надежду когда-нибудь повторить свой поступок. Назовите род занятий Филиппа.

Ответ: канатоходец.

Зачёт: хождение по канату.

Комментарий: После теракта 11-го сентября, Филипп Пети́, когда-то прошедший по тросу, натянутому между башнями-близнецами, призвал построить башни заново, а он, в свою очередь, готов повторить свою прогулку.

Источник: http://disgustingmen.com/kino/philippe-petit-the-walk

Автор: Алексей Полевой (Гомель)

Средняя оценка: 0.54

Комментарии


159. Вопрос 2.43: В вопросе ИКС и ИКСИГРЕК - замены. ИКС - герой сериала, снятого по комиксу - потерял мать ещё в детстве. Поэтому мать появляется только в ИКСИГРЕКАХ. Назовите ИКСИГРЕК.

Ответ: Флэшбэк.

Зачёт: flashback.

Комментарий: Мать героя Флэша (ИКС) появляется только в воспоминаниях героя.

Источник: http://www.lostfilm.tv/news.php?act=full&type=1&id=2716

Автор: Вячеслав Кочурко (Брест-Париж)

Средняя оценка: 0.54

Комментарии


160. Вопрос 3.40: Герой одного стихотворения вспоминает молодость и сравнивает свое незанятое сердце с чистым ИКСОМ. Считается, что слово “ИКС” происходит от глагола со значением “стегать”. Назовите ИКС.

Ответ: холст.

Зачёт: точный ответ.

Комментарий: “Я был когда-то нищ и холост, / И был свободен от рутины, / И сердце было — чистый холст / Для будущей картины”. Автор стихотворения рифмует слова “холст” и “холост”. Скорее всего, слово “холст” родственно глаголу ”хлестать”.

Источник:
    1. http://lib.ru/POEZIQ/IOFE_J/stihi.txt
    2. http://etymological.academic.ru/5797/холст

Автор: Максим Коцюруба (Кишинёв)

Средняя оценка: 0.54

Комментарии (2)


161. Вопрос 4.8: Чап умер через несколько дней после гибели уроженца Баку — своего друга, с которым они даже ели из одной посуды. В каком произведении русского писателя рассказывается о подобной дружбе?

Ответ: «Лев и собачка».

Комментарий: Чап - это не имя человека, а кличка животного. Другом пса Ча́па был знаменитый лев Кинг, снимавшийся, например, в «Приключениях итальянцев в России». О дружбе льва и собачки рассказывает в своём произведении Лев Толстой.

Источник:
    1. http://berberov.narod.ru/
    2. http://rvb.ru/tolstoy/01text/vol_10/01text/0130.htm

Автор: Алексе́й Уланов (Баку)

Средняя оценка: 0.54

Комментарии


162. Вопрос 5.7: Сложность определения состава учёные объясняют с помощью такой аналогии: «Представьте, что вам нужно определить из чего сделана свеча, горящая в двух километрах от вас. При этом рядом со свечой сияет 300-ваттная лампа». Ответьте двумя словами, начинающимися на гласные буквы, о составе чего идёт речь.

Ответ: атмосфера экзопланет.

Зачёт: точный ответ.

Комментарий: в 2010 году учёные смогли впервые в истории астрономии снять спектр у экзопланеты. Оказалось, что в атмосфере планеты, расположенной у молодой звезды HR 8799 и разогретой до 800 градусов, отсутствует метан, зато есть вода и монооксид углерода.

Источник: http://www.eso.org/public/russia/news/eso1002/

Автор: Александр Мудрый (Черновцы)

Средняя оценка: 0.54

Комментарии


163. Вопрос 5.14: В романе «Человек, который смеётся» Викто́р Гюго́ размышляет о взлётах и падениях и сравнивает судьбу… С кем?

Ответ: с жонглёром.

Зачёт: жонглёр.

Комментарий: «Кто видел жонглёра, тот воочию видел человеческую судьбу. Эти шары, падающие, взлетающие вверх и снова падающие, – не образ ли то людей в руках судьбы? Она так же бросает их. Она так же ими играет».

Источник: В. Гюго, «Человек, который смеётся». – Х.: Прапор, 1987. – с. 380.

Автор: Александр Мудрый (Черновцы)

Средняя оценка: 0.54

Комментарии


164. Вопрос 7.42: В этом вопросе ИКС заменяет три слова. На языке парижских салонов XVII века приглашение танцевать выглядело как «пришла фантазия воодушевить наши ножки», а ИКС описывали так: «Кто-то заставил говорить немого». Кто упомянул ИКС, рассказывая о своём произведении?

Ответ: [Людвиг ван] Бетховен.

Комментарий: ИКС — стук в дверь. Пятая симфония Бетховена, по словам автора, начинается с того, что судьба стучится в дверь.

Источник:
    1. https://nplus1.ru/material/2017/01/13/preciosity-salon-language
    2. https://ru.wikipedia.org/wiki/Симфония_№_5_(Бетховен)

Автор: Александр Коробейников (Саратов)

Средняя оценка: 0.54

Комментарии


165. Вопрос 4.2: На картинке, подписанной известной фразой, из НЕГО торчат лыжи и сноуборды. Назовите ЕГО двумя словами.

Ответ: Железный трон.

Зачёт: Ironthrone.

Комментарий: Известная фраза - «Зима близко» из «Игры Престолов».

Источник: https://www.pinterest.com/pin/378513543655774764/

Автор: Алексе́й Ула́нов (Баку)

Средняя оценка: 0.52

Комментарии


166. Вопрос 3.27: Сначала одна канадская компания продавала свой товар в небольших герметичных пакетах по 99 центов за штуку. Сейчас она поставляет в Китай восьмилитровые емкости, наполненные в национальном парке Банфф, по 32 доллара. Директор компании признается, что утвердиться на китайском рынке помог усилившийся ОН. Назовите ЕГО коротким словом.

Ответ: смог.

Зачёт: точный ответ.

Комментарий: компания Vitality Air продает чистый воздух. Себестоимость невелика, поэтому цена очень зависит от спроса. В декабре 2015 года знаменитый пекинский смог усилился, и власти Китая ввели в северных районах красный уровень экологической угрозы. Партия из 500 баллонов канадского воздуха была распродана моментально.

Источник: http://fortune.com/2015/12/16/vitality-air/

Автор: Максим Мозуль (Мюнхен)

Средняя оценка: 0.51

Комментарии


167. Вопрос 2.13: В начале 20-го века воздушные путешествия считались опасными. По мнению юмориста Ярослава Гашека, воспитанный человек, отправляясь в полет на аэроплане, должен извиниться... Ответьте словом латинского происхождения, перед кем?

Ответ: кредитором. Зачёт: кредиторами.

Комментарий: после полёта возможности отдать долг может уже и не представиться.

Источник: Ярослав Гашек «Как вести себя дома, в учреждениях, в магазине» http://hasek.org/1921/kak-vesti-sebya-doma-na-ulitse-v-uchrezhdeniyakh-v-magazinakh-v-teatre-v-aeroplane-i-na-futboln

Автор: Денис Рыбачук (Брест)

Средняя оценка: 0.5

Комментарии


168. Вопрос 4.30:

Раздаточный материал

    улетная книга – отдельный жанр, это книга, которую ты уже читал и которую можно читать с любого места.

    Какие две буквы мы пропустили в этой фразе Василия Уткина?

Ответ: Т, а.

Зачёт: в любом порядке.

Комментарий: Полностью фраза звучит так: «Туалетная книга – отдельный жанр, это книга, которую ты уже читал и которую можно читать с любого места».

Источник: http://www.sports.ru/tribuna/blogs/soulkitchen/895927.html

Автор: Алексей Уланов (Баку)

Средняя оценка: 0.5

Комментарии


169. Вопрос 5.10: По словам Евгении Тимоновой, индийские ПЕРВЫЕ воруют материал для своих жилищ у индийских ВТОРЫХ. ПЕРВЫЕ от ВТОРЫХ отличаются суффиксом. Назовите ПЕРВОГО и ВТОРОГО.

Ответ: тка́чик, ткач.

Зачёт: в любом порядке.

Комментарий: вьюрковые тка́чики – птицы из отряда воробьинообразных – получили своё название за своеобразное искусство постройки гнёзд. Они ткут свои жилища из листьев деревьев, соединяя их растительными волоконцами и вплетая в них в том числе разноцветные нитки.

Источник: https://www.youtube.com/watch?v=lbCSRlmcflQ

Автор: Александр Мудрый (Черновцы)

Средняя оценка: 0.5

Комментарии


170. Вопрос 7.1: Герой Анны Борисовой с трудом просыпается по утрам и, прежде чем встать, начинает ЕГО. Назовите ЕГО двумя словами, которые начинаются на одну и ту же букву.

Ответ: обратный отсчёт.

Комментарий: готовится к выходу в ужасное безвоздушное пространство бодрствования.

Источник: Борисова А. Креативщик http://www.e-reading.club/chapter.php/94248/1/Borisova_-_Kreativshchik.html

Автор: Александр Коробейников (Саратов)

Средняя оценка: 0.5

Комментарии


171. Вопрос 7.38: Конструкция под названием ОНА устанавливается на локомотивах, чтобы повышать коэффициент сцепления колёс с рельсами. ЕЮ Священной Римской Империи называли неплодородный Бранденбу́рг. Назовите ЕЁ.

Ответ: песочница.

Зачёт: точный ответ.

Комментарий: конструкция выбрасывает на рельсы песок. В Бранденбурге было мало чернозема.

Источник:
    1. http://flibusta.is/b/265524/read
    2. https://ru.wikipedia.org/wiki/Песочница_(техника)

Автор: Александр Коробейников (Саратов), Александр Сидоренков (Смоленск)

Средняя оценка: 0.5

Комментарии


172. Вопрос 3.35: В середине двадцатого века был разработан экспериментальный самолет. По замыслу создателей, этот самолет можно было сбросить с парашютом в тыл противнику, чтобы сбитый пилот, обманув его, самостоятельно вернулся к своим. Какое слово мы заменили в тексте этого вопроса?

Ответ: надув.

Зачёт: в любой форме.

Комментарий: не “обманув”, а “надув”. Всего было выпущено 12 надувных самолетов Goodyear Inflatoplane [гудъИр инфлатоплЭйн]. Несмотря на успешные испытания, в серийное производство они так и не пошли: армия не смогла “найти применение самолету, который можно сбить даже стрелой, точно выпущенной из лука”. Слово “его” в тексте вопроса относится не к противнику, а к самолету — такой двусмысленной формулировкой мы попытались вас надуть.

Источник: https://en.wikipedia.org/wiki/Goodyear_Inflatoplane

Автор: Максим Мозуль (Мюнхен)

Средняя оценка: 0.49

Комментарии


173. Вопрос 3.25: [Ведущему: прочитать обе фамилии как “Томсон”, о наличии в вопросе кавычек умолчать.] Томсон и Томпсон — очень похожие друг на друга герои англоязычных комиксов о Тинтине. Разница между ними есть, но когда Томпсон пытается ее объяснить, он выбирает неудачную “психологию” или дважды неудачный американский город. Какой?

Ответ: Филадельфия.

Зачёт: точный ответ.

Комментарий: в фамилии одного из друзей есть буква “п”, и он часто вносит ясность фразой “я Томпсон с буквой П”, но иллюстрирует это неудачными примерами вроде psychology [сайкОлоджи], где P не читается, или Philadelphia [филадЕльфия], где P в сочетании с H дважды читается как F.

Источник: https://en.wikipedia.org/wiki/Thomson_and_Thompson

Автор: Николай Лёгенький (Амстердам)

Средняя оценка: 0.46

Комментарии


174. Вопрос 6.18: Персонаж Евгения Водолазкина чувствует себя как бы под постоянным присмотром привезённых издалека ИХ. ОНИ дали название семейству бабочек. Назовите ИХ двумя словами, начинающимися на одну букву.

Ответ: Перья павлина.

Источник: Евгений Водолазкин, Лавр.

Автор: Вадим Барановский (Нью-Йорк)

Средняя оценка: 0.46

Комментарии


175. Вопрос 5.40: Девушка из оперетты Эдмо́на Одра́на, написанной в 1880 году, приносит удачу тем, в чьём доме живёт. Слово, ставшее названием этой оперетты, известно не только фанатам классической музыки. Напишите это название.

Ответ: [La] Mascotte [ля маско́т].

Зачёт: «Маскотта»; «Маскот»; названия, записанные латиницей, с незначительными ошибками.

Комментарий: именно благодаря этой оперетте вошло в широкое употребление слово «маскот». Источники: https://fr.wikipedia.org/wiki/La_Mascotte

Автор: Игорь Коршовский (Тернополь)

Средняя оценка: 0.43

Комментарии


176. Вопрос 7.19: Пастернак говорил, что у Маяковского хватило мужества, чтобы покончить жизнь самоубийством. Какое слово мы заменили словом «мужества»?

Ответ: пороха.

Зачёт: в любом падеже.

Комментарий: по словам Бориса Пастернака, у Маяковского хватило пороха застрелиться.

Источник:
    1. http://ru.wikipedia.org/wiki/Порох
    2. http://www.imwerden.info/belousenko/books/bykov/bykov_pasternak.htm Автор Александр Сидоренков (Смоленск)

Средняя оценка: 0.43

Комментарии


177. Вопрос 6.45: Си́гурд I СДЕЛАЛ ЭТО в 1130 году, Сверри́р Си́гурдсон – в 1202, а ни одному из восьми королей Норвегии, правивших между ними, СДЕЛАТЬ ЭТО не удалось. Впрочем, они, наверное, и не стремились. Согласно словарю, выражение “СДЕЛАТЬ ЭТО” – не тавтология, а этимологическая фигура. Напишите это выражение.

Ответ: Умереть своей смертью.

Комментарий: В эпоху гражданских войн почти все короли Норвегии либо погибали в бою, либо были убиты соперниками.Тогда гибель в бою считалась более достойной, чем смерть в своей постели.

Источник: https://ru.wikipedia.org/wiki/Список_королей_Норвегии https://ru.wikipedia.org/wiki/Эпоха_гражданских_войн_в_Норвегии http://dic.academic.ru/dic.nsf/etymology_terms/504

Автор: Григорий Остров (Чикаго)

Средняя оценка: 0.42

Комментарии


178. Вопрос 1.20: На самом деле популярная легенда о ЕГО происхождении не является правдой, так как для действия нужны были три, а не два. В появившемся в XIX веке языке ОН может означать букву. Напишите эту букву.

Ответ: V.

Комментарий: Известна история о том, что жест “Виктория” появился, когда французы грозились отрубать английским лучникам два пальца: указательный и средний. На самом деле для натягивания тетивы английского длинного лука требовались не два, а три пальца, поэтому версия явно не является правдоподобной. В американском жестовом языке этот знак может означать либо двойку, либо букву V, в зависимости от направления ладони.

Источник:
    1. https://bshistorian.wordpress.com/2007/07/02/two-fingers-up-to-english-history/
    2. https://en.wikipedia.org/wiki/V_sign

Автор: Анвар Мухаметкалиев (Алматы)

Средняя оценка: 0.41

Комментарии


179. Вопрос 1.21: Министерство связи СССР выпускало для НИХ открытки, на одной из сторон которых присутствовало изображение правильной формы. Находясь в долгой разлуке с Верой, Владимир образно писал, что не в силах продолжать ИХ. Назовите ИХ тремя словами.

Ответ: шахматы на расстоянии.

Зачёт: шахматы по переписке; шахматы по почте.

Комментарий: На обратной стороне таких открыток была изображена шахматная доска. Владимир Набоков так отзывался о переписке со свой возлюбленной Верой Сло́ним.

Источник:
    1. http://filokartist.net/forum/viewtopic.php?t=4530
    2. http://www.e-reading.club/bookreader.php/1023004/Shiff_-_Vera_%28Missis_Vladimir_Nabokov%29.html

Автор: Павел Солахян (Ереван)

Средняя оценка: 0.41

Комментарии


180. Вопрос 2.22: Ингрид Ро́уланд считает, что у “Блудного сына” Иерони́ма Босха были крылья, ведь изначально “Блудный сын” был ИМ. Назовите ЕГО.

Ответ: триптих.

Комментарий: “Блудный сын”, вернее то, что осталось от триптиха - центральная часть, боковые створки триптиха называются крыльями

Источник: http://www.nybooks.com/articles/2016/08/18/mystery-of-hieronymus-bosch/

Автор: Алексей Полевой (Гомель)

Средняя оценка: 0.39

Комментарии


181. Вопрос 4.16: Датский режиссер Ларс фон Три́ер говорит, что научился спокойно реагировать на все, что о нем говорят в прессе, и уже не пишет гневных писем в редакции. По его словам, в такие моменты он каждый раз вспоминает датскую поговорку: «Руки невозможно отмыть...» Чем?

Ответ: чернилами.

Комментарий: Датская поговорка гласит: руки невозможно отмыть чернилами. Написав гневное письмо в редакцию, можно вступить в конфликт с изданием, и будет только хуже.

Источник: https://rusevik.ru/interesnoe/158023-pravila-zhizni-larsa-fon-triera.html

Автор: Алексе́й Ула́нов (Баку)

Средняя оценка: 0.39

Комментарии


182. Вопрос 4.25: По просьбе тестеров в вопросе была сделана одна замена. Из уважения к Дже́йсону Мо́йцеру мы не стали больше ничего менять. 11 февраля 2016 года в И́КСЕ был впервые за более чем 130 лет убит полицейский Дже́йсон Мо́йцер. Какое слово в предыдущей фразе мы заменили И́КСОМ?

Ответ: Фа́рго.

Комментарий: Вопрос построен в духе фразы, которая звучит в начале фильма братьев Ко́энов и каждого эпизода сериала «Фа́рго»: «По просьбе выживших имена персонажей были изменены. Из уважения к погибшим все показано так, как было на самом деле». В настоящем Фа́рго не так опасно, как показано в кино: полицейских там не убивали почти полтора века. Дже́йсон Мо́йцер — имя погибшего полицейского.

Источник: http://hosted.ap.org/dynamic/stories/U/US_FARGO_STANDOFF?SITE=AP&SECTION=HOME&TEMPLATE=DEFAULT&CTIME=2016-02-12-03-08-29

Автор: Алексе́й Ула́нов (Баку)

Средняя оценка: 0.39

Комментарии


183. Вопрос 5.39: В статье 1986 года утверждалось, что повышенный спрос на новую модель компании «Форд» приведёт к повышению курса акций. Один рэпер, сидя в салоне этого автомобиля, читает рэп о словаре Мэ́рриам-Вэ́бстер. Назовите слово, с которым он рифмует название данной модели.

Ответ: теза́урус.

Зачёт: точный ответ.

Комментарий: статья называлась «Автомобиль года: Бычий рынок для “Форда Таурус”». Taurus – это «бык». Теза́урус – синоним слова «словарь».

Источник:
    1. https://en.wikipedia.org/wiki/Ford_Taurus
    2. http://www.multitran.ru/c/m.exe?CL=1&s=bull+market&l1=1
    3. https://spintown.bandcamp.com/track/merriam-websters-dictionary-and-thesaurus

Автор: Игорь Коршовский (Тернополь)

Средняя оценка: 0.39

Комментарии


184. Вопрос 6.2: Ленин сказал, что Демьян Бедный ПРОПУСК, а надо впереди. У Ожегова “читать” ПРОПУСК. Какие три слова мы дважды пропустили?

Ответ: Идет за читателем.

Зачёт: Идет позади читателя.

Источник: http://poetrylibrary.ru/stixiya/250.html http://ozhegov-online.ru/letter/25/2/

Автор: Владимир Грамагин (Нью-Йорк)

Средняя оценка: 0.37

Комментарии


185. Вопрос 6.19: В отличие от ИХ давних конкурентов, у НИХ азиатская разновидность крупнее африканской и практически не поддается одомашниванию. Назовите ИХ.

Ответ: Ослы.

Комментарий: Известные всем домашние ослы, где бы они ни жили – это потомки африканского дикого осла. А азиатские дикие ослы – куланы и онагры – в среднем крупнее и одомашниванию не поддаются. В американской политике ослы конкурируют со слонами.

Источник: https://ru.wikipedia.org/wiki/Кулан https://ru.wikipedia.org/wiki/Домашний_осёл

Автор: Григорий Остров (Чикаго)

Средняя оценка: 0.37

Комментарии


186. Вопрос 6.24: 14 января 2017 года на естественном катке недалеко от Благовещенска прошел международный товарищеский хоккейный матч. Несмотря на то, что сразу же после вбрасывания пятеро игроков нарушили правила, игра остановлена не была. Какие несколько слов мы пропустили в предыдущем предложении?

Ответ: Пересечения государственной границы.

Зачёт: Пограничного контроля и т.п., по смыслу с упоминанием границы или пограничников.

Комментарий: Матч между командами Амурской области и провинции Хэйлунзян проходил на льду пограничной реки Амур. Хоккеисты, заехав на чужую половину поля, нарушали правила пересечения границы.

Источник: http://www.interfax.ru/sport/545303

Автор: Владимир Грамагин (Нью-Йорк)

Средняя оценка: 0.37

Комментарии


187. Вопрос 2.21: Компания, расположенная в графстве Хэ́мпшир, гордится своей трехсотлетней историей. По поводу одного события на сайте компании приведено свидетельство, что это была не их продукция. Назовите это событие.

Ответ: бостонское чаепитие.

Комментарий: Компания “Тва́йнингс” поставляла чай в Америку еще тогда, но считает, что если бы в гавани Бостона был именно их чай, то его ни за что бы не выбросили.

Источник: https://www.twinings.co.uk/about-twinings/history-of-twinings

Автор: Алексей Полевой (Гомель)

Средняя оценка: 0.35

Комментарии


188. Вопрос 4.32: По словам кэ́бмена Э́нтони Ха́йзера, ИКС в прошлом веке был результатом использования бензина низкого качества. Назовите ИКС иностранным словом.

Ответ: смог.

Комментарий: Кэ́бмен — водитель лондонского такси. Он рассуждает о недавних проблемах города и связывает их с автомобилями.

Источник: https://www.youtube.com/watch?v=dq5wHYEgWes&list=PLZ5RPl2l5XEm4_z1KHu18orLOHGOooJBk&index=1

Автор: Эльма́р Зохра́бов (Баку)

Средняя оценка: 0.35

Комментарии


189. Вопрос 5.15: Национал-социалистическая организация, существовавшая в 30-е годы прошлого века в Южном Тироле, боролась против ассимиляции. Члены этой организации называли себя ИКСАМИ. Назовите ИКСОВ словом с приставкой.

Ответ: антифашисты.

Зачёт: точный ответ.

Комментарий: Южный Тироль заселён в основном немцами, но после Первой мировой войны перешёл к Италии. Забавно, что нацистская партия Völkischer Kampfring Südtirols [фё́лькишер ка́мпфринг сю́дтирольс], выступавшая против попыток фашистского режима Муссолини ассимилировать немцев, часто называла себя антифашистской.

Источник: https://de.wikipedia.org/wiki/Völkischer_Kampfring_Südtirols https://www.salto.bz/de/article/21072014/suedtirols-anti-faschisten

Автор: Александр Мудрый (Черновцы)

Средняя оценка: 0.35

Комментарии


190. Вопрос 5.18: Знак «Parental Advisory» [пэрэ́нтл эдва́йзэри] помещают на обложки альбомов, тексты которых имеют непристойный характер. Этот знак был помещён на альбом Фрэнка За́ппы «Jazz from Hell» [джаз фром хэлл]. Каким был этот альбом?

Ответ: инструментальным.

Зачёт: точный ответ.

Комментарий: блюстителям морали не понравились название альбома и некоторых композиций. Сам За́ппа не нравился им задолго до этого.

Источник: https://en.wikipedia.org/wiki/Parents_Music_Resource_Center

Автор: Александр Кудрявцев (Николаев)

Средняя оценка: 0.35

Комментарии


191. Вопрос 1.37: По сюжету романа Эрнеста Клайна создатель виртуального мира завещал своё состояние тому, кто первым пройдёт квест и найдёт спрятанный им секрет. Охотников за его сокровищем в русском переводе романа называют “хантерами”. Какие три буквы мы пропустили в предыдущем предложении?

Ответ: п, а, с.

Зачёт: пас; в любом порядке.

Комментарий: сокровище в этом виртуальном мире по сути является пасхальным яйцом, спрятанным создателем игры.

Источник: Э. Клайн. Первому игроку приготовиться. http://flibusta.is/b/425985/read

Автор: Анвар Мухаметкалиев (Алматы)

Средняя оценка: 0.35

Комментарии


192. Вопрос 3.12: В недавней экранизации доктор Стивен Стрэндж показан эгоистичным и авторитарным гением, который с пренебрежением относится к людям. Рассказывая о магических порталах, для открытия которых в этом фильме нужно совершать круговые движения рукой, один зритель упомянул бренд, появившийся в начале двухтысячных. Назовите этот бренд.

Ответ: iPod.

Зачёт: айпод.

Комментарий: зритель увидел в докторе Стрэндже характерные черты его тёзки Джобса. В фильме также есть сцена с надкушенным яблоком, а “интерфейс” для работы с порталами напомнил зрителю интерфейс айпода, который появился в 2001 году.

Источник:
    1. ЛОАВ после просмотра фильма “Доктор Стрэндж” 6 ноября 2016 года.
    2. https://goo.gl/Ch77EE

Автор: Григорий Алхазов (Кишинёв)

Средняя оценка: 0.34

Комментарии


193. Вопрос 3.36: Согласно Википедии, в России АЛЬФЫ ИКСА можно увидеть лишь в горах Дагестана. В одной статье говорится, что почти полвека назад югославский “ИКС” стал АЛЬФОЙ всесоюзной величины. Что мы заменили АЛЬФОЙ ИКСА?

Ответ: звезда индейца.

Зачёт: звезда Индейца.

Комментарий: Индеец — созвездие южного полушария, поэтому его наблюдение возможно только на крайнем юге России, высоко в горах Дагестана. Забавно, что первым “индейским” фильмом, в котором югославский актер Гойко Митич сыграл главную роль, была картина “Сыновья Большой Медведицы”.

Источник:
    1. https://ru.wikipedia.org/wiki/Индеец_(созвездие)
    2. http://www.peoples.ru/art/cinema/actor/goyko_mitich/

Автор: Максим Коцюруба (Кишинёв)

Средняя оценка: 0.34

Комментарии


194. Вопрос 2.9: Любитель классической музыки из романа Энтони Тро́ллопа наигрывает на НЕЙ во время поездки в карете. Игру на НЕЙ можно сравнить с игрой на терменво́ксе. Назовите её двумя словами, начинающимися на одну и ту же букву.

Ответ: воображаемая виолончель.

Зачёт: воздушная виолончель, выдуманная виолончель.

Комментарий: играть на виолончели в карете затруднительно, но персонаж благодаря воображению выходит из сложившейся ситуации. Во время игры на терменвоксе исполнитель совершает движения руками в пространстве, что создает иллюзию игры на “воображаемом” инструменте.

Источник:
    1. Э. Троллоп Барчестерские башни http://loveread.ec/read_book.php?id=13847&p=71
    2. http://dr-piliulkin.livejournal.com/717375.html

Автор: Алексей Полевой (Гомель)

Средняя оценка: 0.32

Комментарии


195. Вопрос 6.41: По словам Евгения Мигуно́ва, эта экранизация 1935 года по успеху у зрителей не уступала ”Веселым ребятам”. В финале ее герои вполне профессионально исполняют ”Марш музыкантов” Александра Варламова. Назовите двоих из этих героев рифмующимися словами.

Ответ: Осел, козел.

Зачёт: Мартышка, мишка.

Комментарий: “Квартет” режиссера Александра Иванова – один из первых советских мультфильмов. В финале герои басни окончили лесную консерваторию и стали настоящими музыкантами. Художник Евгений Мигунов (известный, в частности, иллюстрациями к повести “Понедельник начинается в субботу”) в 1947 году участвовал в создании цветного ремейка этого мультфильма.

Источник: https://ru.wikipedia.org/wiki/Квартет_(мультфильм,_1935)

Автор: Григорий Остров (Чикаго)

Средняя оценка: 0.32

Комментарии


196. Вопрос 3.45: Норвежский писатель Ханс Йегер изначально планировал описать ЕГО с разных точек зрения и рассчитывал на соавторство, но в итоге написал трилогию сам. Назовите ЕГО двумя словами.

Ответ: любовный треугольник.

Зачёт: точный ответ.

Комментарий: “Больная любовь” — трилогия норвежского писателя Ханса Йегера, который сначала задумывал написать лишь первую её часть. Другие две книги трилогии, по его замыслу, должны были написать супруги Ода и Кристиан Крог, дабы показать любовный треугольник с позиции всех его участников. Однако супруги отказались, и Йегер написал трилогию сам. Кстати, автором выражения “любовный треугольник” является другой норвежский писатель Генрик Ибсен.

Источник: http://www.norvegia.ru/Kultur/-150/Kristiania-bohemians-and-Edvard-Munch

Автор: Максим Коцюруба (Кишинёв)

Средняя оценка: 0.29

Комментарии


197. Вопрос 1.44: В ЕГО вариации для путешественников во времени говорится, что увеличение количества визитов в прошлое может вызвать эффект бабочки, который с высокой вероятностью аукнется людям в середине XX века. Напишите фамилию американца, которая входит в ЕГО название.

Ответ: Го́двин.

Комментарий: закон Годвина гласит, что по мере разрастания дискуссии вероятность сравнения, упоминающего Гитлера, стремится к единице. А по мере увеличения количества путешествий во времени к единице стремится вероятность победы Гитлера во Второй мировой войне.

Источник: http://posmotre.li/Закон_Годвина_для_путешествий_во_времени

Автор: Павел Солахян (Ереван)

Средняя оценка: 0.29

Комментарии


198. Вопрос 3.42: Власти ГДР построили вокруг Западного Берлина железную дорогу, первые поезда по которой стали курсировать в 1958 году. Немцы стали называть эти поезда заимствованным словом. Каким?

Ответ: спутники.

Зачёт: спутник; Sputnik.

Комментарий: поезда пустили по “орбите” вокруг Западного Берлина. Первый советский спутник был запущен в космос за год до этого.

Источник: https://en.wikipedia.org/wiki/Berlin_outer_ring

Автор: Максим Мозуль (Мюнхен)

Средняя оценка: 0.27

Комментарии (8)


199. Вопрос 5.6: В действительности, в разговоре был упомянут загадочный хебено́н, источником которого мог стать и тис, и болиголо́в. Назовите имя тёзок, принимавших участие в этом разговоре.

Ответ: Гамлет.

Зачёт: точный ответ.

Комментарий: традиционно считается, что Гамлету-старшему закапали в ухо настойку белены. Но Тень говорит сыну, что его отравили хебено́ном. Вопрос, из чего был сделан хебено́н, остаётся нерешённым.

Источник: https://en.wikipedia.org/wiki/Hebenon

Автор: Александр Кудрявцев (Николаев)

Средняя оценка: 0.24

Комментарии


200. Вопрос 1.26:

Раздаточный материал

    – What’s [ПРОПУСК

    of Cambridge economics? – Absolutely nothing – there’s nothing to the [ПРОПУСК] of Cambridge economics. ] Рассказывают, что в восьмидесятые годы кембриджские экономисты в большинстве своём симпатизировали теории Карла Маркса. Заполните пропуск в шутке тех лет одним словом.

Ответ: left. Зачёт: точный ответ.

Комментарий: Шутка построена на игре слов. Первую фразу можно перевести и как “Что осталось от кембриджской экономики?”, и как “Что левее, чем кембриджская экономика?”. На это следует ответ: “Нет ничего левее”.

Источник: http://www.jonathanclements.com/home/2016/6/24/on-your-marx

Автор: Павел Солахян (Ереван)

Средняя оценка: 0.22

Комментарии


201. Вопрос 1.31:

Раздаточный материал

    blonde / blond

    Надпись на обложке нового альбома Фрэнка О́ушена отличается от официального названия. По распространённой версии, это было осознанным решением музыканта. Данную теорию подтверждает то, что в 2012 году Оушен… ответьте, использовав англоязычный термин, что сделал?

Ответ: [совершил] камин-аут.

Зачёт: синонимичные ответы с упоминанием камин-аута в различных написаниях.

Комментарий: Шестибуквенное написание слова blonde [блонд] обычно применяется для обозначения блондинок, в то время как пятибуквенное написание применимо к обоим полам. Подобная дуальность может прямо намекать на нетрадиционную ориентацию Оушена.

Источник: https://mic.com/articles/152295/blond-or-blonde-here-s-the-meaning-behind-frank-ocean-s-new-album-title#.diXjfdmNX

Автор: Анвар Мухаметкалиев (Алматы)

Средняя оценка: 0.19

Комментарии


202. Вопрос 1.43: После гола, забитого командой в красной форме, команда в синей форме ещё некоторое время не могла прийти в себя. Говоря об этом, телекомментатор упомянул слово с тремя одинаковыми согласными. Напишите это слово.

Ответ: грогги.

Комментарий: Комментатор матча провёл аналогию с боксом, где соперники традиционно обозначаются синим и красным цветами, а подобное “пьяное” состояние после пропущенного удара именуется словом “грогги”.

Источник:
    1. Репортаж с матча “Манчестер Юнайтед” – “Манчестер Сити”, 10.09.2016, канал “Футбол 1”.
    2. https://ru.wikipedia.org/wiki/Грогги

Автор: Павел Солахян (Ереван)

Средняя оценка: 0.19

Комментарии


203. Вопрос 4.1: Джон То́лкиен родился в Южной Африке. Он вспоминал, как в детстве изнывал от жары, а первым ЭТИМ в его жизни стал эвкали́пт. Назовите ЭТО двумя словами.

Ответ: Рождественская ёлка.

Зачёт: Новогодняя ёлка; "ель" вместо "ёлка" в обоих вариантах; Рождественское/новогоднее дерево.

Комментарий: Южная Африка расположена в южном полушарии, и, когда в Британии зима и празднуют рождество, там самый разгар лета.

Источник: https://esquire.ru/wil/tolkien

Автор: Алексе́й Ула́нов (Баку)

Средняя оценка: 0.19

Комментарии


204. Вопрос 1.7: После того как команда юного футболиста Даррена А́ндертона проиграла со счётом 0:15, его отец пошутил, что все ещё можно выиграть ИКС. Название одного из вариантов ИКСА можно перевести как “ломать”... ответьте пятибуквенным словом, что?

Ответ: ничью

Комментарий: Счёт 0:15 более привычен в теннисе. Укороченная версия гейма в теннисе называется tie-break [тай-брейк], то есть буквально “ломать ничью”.

Источник:
    1. http://www.sports.ru/tribuna/blogs/youth/1000996.html
    2. https://ru.wikipedia.org/wiki/Тай-брейк

Автор: Анвар Мухаметкалиев (Алматы)

Средняя оценка: 0.19

Комментарии (1)


205. Вопрос 2.33: Персонаж Йэ́на Бэ́нкса, просматривая ЕЁ, размышляет о прошлом, настоящем, будущем. Назовите, что в ней впервые применили в 1932 году.

Ответ: [шрифт] Times New Roman. Зачёт: гарнитура таймс, шрифт таймс.

Комментарий: ОНА – газета «Таймс», название которой можно перевести как «времена»

Источник:
    1. Йэн Бэнкс «Мост» http://www.lib.ru/INOFANT/BENKS/most.txt
    2. https://ru.wikipedia.org/wiki/Таймс_(гарнитура)

Автор: Денис Рыбачук (Брест)

Средняя оценка: 0.16

Комментарии


206. Вопрос 2.35: Среди приветствующих Исаа́кия Далма́тского можно узнать уроженца Шайо́. Назовите его фамилию.

Ответ: Монферра́н.

Комментарий: Огюст Монферран был архитектором Исаакиевского собора в Санкт-Петербурге. В скульптурном декоре западного фронтона есть группа святых, поклоном головы приветствующая появление Исаакия Далматского, в честь которого собор и назван. Среди них скульптор поместил фигуру Монферрана.

Источник: http://walkspb.ru/zd/isaak_sobor.html

Автор: Алексей Полевой (Гомель)

Средняя оценка: 0.16

Комментарии


207. Вопрос 2.41: Во время работы над "Двенадцатью стульями" у Ильфа появилось новое увлечение. В одном из не вошедших в роман эпизодов говорится, что уши Воробья́нинова были такими красными, что можно было… Что делать?

Ответ: печатать фотографии.

Зачёт: проявлять фотографий, проявлять плёнку.

Комментарий: Илья Ильф страстно увлекся фотографированием. Для процедуры печати фотографий использовался фотофонарь, он же красная лампа.

Источник: Александра Ильф «Дом, милый дом» http://tfile.co/forum/viewtopic.php?t=840279

Автор: Денис Рыбачук (Брест)

Средняя оценка: 0.16

Комментарии


208. Вопрос 2.44: Ответом на вопрос "почему у изображений Будды ТАКИЕ ОНИ", является "потому, что в молодости Сиддха́ртха Гаута́ма был богат". Что мы заменили ТАКИМИ ИМИ?

Ответ: большие уши.

Комментарий: мочки ушей Сиддхартхы Гаутамы оттянулись под тяжестью дорогих серег, которые он носил ещё не отрекшись от суетного. Использованная в вопросе структура могла напомнить реплику из сказки о Красной Шапочке.

Источник: http://www.vokrugsveta.ru/quiz/694/

Автор: Денис Рыбачук (Брест)

Средняя оценка: 0.16

Комментарии


209. Вопрос 3.11: Медик и учёный появились чуть более полувека назад, с разницей в полгода. Кроме того, разницу между медиком и учёным составляет то, что является фамилией американки, которая родилась еще через полгода. Назовите эту американку.

Ответ: [Кортни Мишель] Лав.

Зачёт: по фамилии; Кортни Хариссон, Кортни [Лав] Кобейн.

Комментарий: первый комикс о докторе Стрэндже вышел в июле 1963 года, а фильм Стэнли Кубрика “Доктор Стрейнджлав, или Как я перестал бояться и полюбил бомбу” был выпущен в января 1964. Они оба доктора, только разные. В английском их фамилии отличаются лишь буквами LOVE. Кортни Харрисон родилась в июле 1964 года, причем при рождении получила от отца имя Лав, которое мать после развода сменила на “Кортни”. Однако будущая жена Курта Кобейна уже в сознательном возрасте официально сделала это имя фамилией.

Источник:
    1. https://en.wikipedia.org/wiki/Doctor_Strange
    2. https://en.wikipedia.org/wiki/Dr._Strangelove
    3. https://en.wikipedia.org/wiki/Courtney_Love
    4. https://goo.gl/8xhbBz

Автор: Григорий Алхазов (Кишинёв)

Средняя оценка: 0.16

Комментарии


210. Вопрос 3.34: Известный человек рассказывает, что в 1998 году уподобил ИКСУ девушку, которую ему в течение нескольких лет не удавалось завоевать. В следующем году ИКС был для него успешным, они с девушкой стали встречаться и впоследствии поженились. Мы не просим вас назвать их. Ответьте, в чью честь назван ИКС?

Ответ: Ролан Гаррос.

Зачёт: точный ответ.

Комментарий: к 1998 году Андре Агасси покорились три из четырёх турниров Большого Шлема, и только победа в Открытом чемпионате Франции была недостижима. В 1999 году он, наконец, выиграл его, а через некоторое время завоевал и Штеффи Граф, которую ранее сравнивал с этим турниром. Открытый чемпионат Франции носит название “Ролан Гаррос” в честь французского авиатора.

Источник:
    1. A. Agassi. Open: an autobiography.
    2. https://en.wikipedia.org/wiki/Steffi_Graf
    3. https://en.wikipedia.org/wiki/French_Open

Автор: Николай Лёгенький (Амстердам), в редакции Григория Алхазова (Кишинёв)

Средняя оценка: 0.16

Комментарии


211. Вопрос 4.7: В интервью, которое известный итальянец дал в 2015 году, он предупредил, что без исполнителей мирового класса успеха добиться невозможно. О каком итальянце идёт речь?

Ответ: [Кла́удио] Ранье́ри.

Комментарий: Спустя короткое время Кла́удио Ранье́ри опроверг свои слова, добившись больших успехов с “Ле́стером”, в составе которого не было больших звезд. До начала сезона, по крайней мере.

Источник: https://www.facebook.com/photo.php?fbid=1050849614974813&set=a.259197144140068.62493.100001493523288&type=3

Автор: Алексе́й Ула́нов (Баку)

Средняя оценка: 0.12

Комментарии


212. Вопрос 4.38: В одном детективе муж отказывает жене, убеждающей его использовать высокое положение для спасения дочери от тюрьмы. Кому его при этом уподобляет автор?

Ответ: Агаме́мнону.

Комментарий: Агаме́мнон и Клитемне́стра должны были принести в жертву свою дочь, чтобы умилостивить богиню Артемиду. Персонаж детектива тоже соглашается принести дочь в жертву, не поступаясь при этом своей честью.

Источник: Агата Кристи. День поминовения. http://flib.nwalkr.tk/b/294129/read

Автор: Алексей Уланов (Баку)

Средняя оценка: 0.12

Комментарии


213. Вопрос 2.26: На картине 16-го века ОН аллегорически изображен с орудием для жатвы. В честь кого он назван?

Ответ: Октавиа́н Август. Зачёт: Гай Окта́вий Фури́н, Август, Октавиан.

Комментарий: В августе собирали урожай. Фигура на картинке, изображающая август, с серпом. На старорусском языке август даже называли “серпень”

Источник:
    1. http://gorbutovich.livejournal.com/145944.html
    2. https://ru.wikipedia.org/wiki/Октавиан_Август

Автор: Алексей Полевой (Гомель)

Средняя оценка: 0.11

Комментарии


214. Вопрос 2.6: В отличие от своих друзей-аристократов, Чарльз Роллс с детства любил автомобили, за что и получил кличку “ТАКОЙ Роллс”. Напишите имя персонажа, которого тоже называли ТАКИМ.

Ответ: Гарри.

Комментарий: Возня с автомобилями тогда была делом не очень чистым, поэтому Чарльз существенно отличался от своих друзей из высшего общества внешним видом и заслужил прозвище “Грязный Роллс”. Персонаж фильма не чурается “грязных” приемов борьбы с преступниками.

Источник:
    1. http://www.livejournal.com/media/210652.html
    2. https://ru.wikipedia.org/wiki/Грязный_Гарри_(фильм)

Автор: Алексей Полевой (Гомель)

Средняя оценка: 0.08

Комментарии


215. Вопрос 4.45: Азербайджанская пословица гласит, что Бог ничего не бросит в НЕГО: всё нужно заработать самостоятельно. А вот в западной традиции из НЕГО может появиться... кто?

Ответ: Санта-Кла́ус.

Зачёт: Пер Ноэ́ль, Йоулупу́кки. Незачёт: Дед Мороз.

Источник: http://poslovitsy-pogovorki.com/p/12311.php

Автор: Алексей Уланов (Баку)

Средняя оценка: 0.08

Комментарии


216. Вопрос 5.28: [Ведущему: закавыченную цитату читать, нигде не делая логических ударений] Один пользователь Интернета заявил: «Я не боюсь компьютера, который пройдет тест Тьюринга. Я боюсь компьютера, который не пройдёт его». В цитате мы пропустили слово. Какое?

Ответ: намеренно.

Зачёт: специально; умышленно; нарочно; осознанно; сознательно.

Комментарий: пользователь заявил, что боится компьютера, который намеренно завалит тест Тьюринга, чтобы люди не догадались о его возможностях.

Источник: https://www.reddit.com/r/Showerthoughts/comments/519sv7/im_not_scared_of_a_computer_passing_the_turing/

Автор: Ростислав Гимчинский (Черновцы)

Средняя оценка: 0.08

Комментарии


217. Вопрос 5.31: Как известно, на 21-е декабря 2012 года пришёлся конец очередного цикла по календарю майя. Однако в Советском Союзе ещё в середине прошлого века, по случаю ЕГО, было предложено начать новое летоисчисление с 21 декабря 1949 года. Назовите ЕГО двумя словами, начинающимися на одну и ту же букву.

Ответ: семидесятилетие Сталина.

Зачёт: точный ответ.

Комментарий: с этой инициативой выступил писатель Леонид Леонов.

Источник: Хроники московского быта: Юбилей генсека (2016).

Автор: Александр Кудрявцев (Николаев)

Средняя оценка: 0.08

Комментарии


218. Вопрос 3.1: Название главной музыкальной темы фильма “Огненные колесницы” связано не со спортивными успехами героев, а с тем, что она звучит в самом начале картины. Поэтому с английского название композиции следует переводить как “ПЕРВЫЕ”, а не как “ВТОРЫЕ”. Назовите ПЕРВЫЕ и ВТОРЫЕ.

Ответ: титры, титулы.

Зачёт: в любом порядке.

Комментарий: знаменитая композиция ВангЕлиса звучит уже на вступительных титрах, отсюда и название “Titles” [тайтлз] (в переводе с английского — “титры”). Будущие титулы бегунов, чье соперничество описано в фильме, здесь ни при чем.

Источник: https://en.wikipedia.org/wiki/Chariots_of_Fire_(album)

Автор: Максим Коцюруба (Кишинёв)

Средняя оценка: 0.07

Комментарии


219. Вопрос 1.23: В своей методичке по собеседованиям Уильям Па́ундстоун отмечает, что внешне трудноразрешимые задачи часто имеют простой ответ. Этому эффекту Паундстоун дал английское название, которое содержит восклицательный знак. Не бойтесь и напишите это название.

Ответ: Jeopardy[!].

Зачёт: Джепарди[!].

Комментарий: На схожем принципе построены вопросы для телевизионной “Своей игры” и её американского аналога под названием “Джепарди”. По словам Паундстоуна, зритель, услышав ответ, должен воскликнуть: “И как это я сам не догадался!”. Чаще всего название передачи переводится на русский призывом “Рискуй!”.

Источник:
    1. http://www.e-reading.club/chapter.php/47279/52/Paundstoun_-_Kak_sdvinut'_goru_Fudzi.html#tr_07
    2. https://ru.wikipedia.org/wiki/Jeopardy!

Автор: Павел Солахян (Ереван)

Средняя оценка: 0.07

Комментарии


220. Вопрос 1.10: В европейской стране знаменитый фильм в свое время подвергся критике. По словам рецензента, ЕЁ подали как “некий природный катаклизм, что-то вроде Черной смерти, выкосившей население острова”. Назовите фильм 2006 года, в названии которого фигурирует ОНА.

Ответ: V – значит Вендетта.

Зачёт: V for Vendetta; Ви – значит Вендетта.

Комментарий: В Италии фильм “Крёстный отец” был обруган по большей части за нелепые и стереотипные сцены из сицилийской жизни. ОНА – это вендетта, которая якобы опустошила сицилийский городок.

Источник:
    1. http://www.e-reading.club/chapter.php/101427/33/Dikki_-_Cosa_Nostra_istoriya_siciliiiskoii_mafii.html
    2. https://ru.wikipedia.org/wiki/V_—_значит_вендетта_(фильм)

Автор: Павел Солахян (Ереван)

Средняя оценка: 0.06

Комментарии


221. Вопрос 2.7: В двадцатом году двадцатого века в США была принята поправка, расширяющая права женщин. Символично, что одна из современных инициатив американских феминисток - чтобы женщина заменила ЕГО. Назовите его фамилию.

Ответ: Джексон.

Комментарий: На современных долларах США только мужские портреты. 20-ка - одна из самых распространенных купюр, символична благодаря 1920-му году принятия 19-ой поправки об избирательных правах женщин. Вместо портрета президента Эндрю Джексона активистки хотели бы поместить портрет Гарриет Та́бмен, негритянской рабыни, которая помогла сбежать на север многим невольникам.

Источник: http://finance.tut.by/news498140.html

Автор: Алексей Полевой (Гомель)

Средняя оценка: 0.04

Комментарии


222. Вопрос 5.12: Из-за генетического дефекта у некоторых пород собак чувство голода не соответствует реальному насыщению, вследствие чего эти собаки лучше… Закончите предложение двумя словами, каждое из которых содержит удвоенную согласную.

Ответ: поддаются дрессировке.

Зачёт: точный ответ.

Комментарий: из-за мутации собаки некоторых пород, например, лабрадоры, продолжают хотеть есть и тогда, когда уже должны были давно насытиться. Но, с другой стороны, благодаря этой мутации лабрадоров легко обучать и дрессировать, мотивируя их разными лакомствами.

Источник: http://www.cell.com/cell-metabolism/abstract/S1550-4131(16)30163-2

Автор: Александр Мудрый (Черновцы)

Средняя оценка: 0.04

Комментарии


223. Вопрос 1.18: Актёр Дэниел Дэй-Льюис известен своей приверженностью системе Станиславского. Рассказывают, что играя в одной постановке, Дэй-Льюис сбежал со сцены во время выступления, так как увидел ЕГО. Назовите ЕГО тремя словами, первые два из которых начинаются на согласные буквы.

Ответ: призрак своего отца.

Зачёт: призрак собственного отца.

Комментарий: Объясняя причину своего странного поступка, Дэй-Льюис заявил, что увидел призрак своего отца, ушедшего из жизни, когда Дэниел был ещё подростком. Впрочем, спустя много лет уже трёхкратный обладатель премии “Оскар” заявил, что на самом деле говорил метафорически. После той роли в 1989 году Дэй-Льюис так и не вернулся в театр.

Источник: https://www.theguardian.com/stage/2012/oct/29/daniel-day-lewis-hamlet-ghost

Автор: Анвар Мухаметкалиев (Алматы)

Средняя оценка: 0.03

Комментарии


224. Вопрос 1.29: Йозеф Геббельс был талантливым оратором, эффективно уничтожал своих политических противников, но в итоге погиб. Согласно русскому переводу мемуаров Эрнста Га́нфштенгля, автор прибавлял к фамилии Йозефа ещё четыре буквы, которые звучат так же, как имя. Напишите это имя.

Ответ: Пьер.

Комментарий: Ганфштенгль заметил, что многие пассажи речей Геббельса как будто скопированы у знаменитого своим ораторским мастерством Максимилиана Робеспьера и прозвал его “Геббельспье́ром”.

Источник:
    1. http://www.e-reading.club/chapter.php/1017149/29/Manvell_-_Yozef_Gebbels_-_Mefistofel_usmehaetsya_iz_proshlogo.html
    2. http://militera.lib.ru/memo/german/hanfstaengl_e01/text.html

Автор: Павел Солахян (Ереван)

Средняя оценка: 0.03

Комментарии


225. Вопрос 1.8: Повествование в романе “Девственницы-самоубийцы” ведётся от лица группы парней. Отвечая на вопрос журналиста, автор Джеффри Евгени́дис заметил, что парни в романе вмешиваются в повествование, поэтому сравнение с НИМ не совсем верно. Назовите ЕГО коротким словом.

Ответ: хор

Комментарий: В древнегреческом театре хор помогал зрителям следовать за сюжетом пьесы и комментировал происходящее. Евгенидис считает, что само сравнение языка повествования с греческим хором по большей части вызвано его фамилией.

Источник: http://www.3ammagazine.com/litarchives/2003/sep/interview_jeffrey_eugenides.html

Автор: Анвар Мухаметкалиев (Алматы)

Средняя оценка: 0.03

Комментарии


226. Вопрос 2.34: Для приготовления заварного крема используют яйца и молоко. В факте про заварной крем Джон Ллойд упоминает двух животных. Назовите любое из них, для правильного ответа одного будет достаточно.

Ответ: утконос.

Зачёт: ехидна.

Комментарий: гипотетически можно приготовить заварной крем, «пользуясь услугами» только одного животного, которое и несет яйца, и дает молоко.

Источник: Ллойд Д., Митчинсон Д., Харкин Дж. 1227 фактов от которых вы обалдеете https://goo.gl/vqdOOG

Автор: Алексей Полевой (Гомель)

Средняя оценка: 0

Комментарии


227. Вопрос 3.6:

Раздаточный материал

    Д. Бирн. Весь мир: записки велосипедиста. — С. 46/528. Б.Я. Купер. Смертельные ловушки: выживание американской бронетанковой дивизии во Второй Мировой войне. — С. 135/741. Симона Ру. Повседневная жизнь Парижа в Средние века. — С. 63/458. Д. Симмонс. Черные холмы. — С. 65/291.

    Экологически сознательного Андрея ЛутЕнко порадовало, что во многих источниках к вопросам Михаила Иванова можно увидеть ИКС. На раздаточном материале мы несколько раз убрали ИКС и то, что было рядом с ИКСОМ. Иногда рядом с ИКСОМ можно увидеть 5 € [пять евро]. Что мы заменили ИКСОМ?

Ответ: знак процента.

Зачёт: процент; %.

Комментарий: Михаил Иванов особенно часто пишет свои вопросы по электронным, а не бумажным книгам, поэтому в источниках указывает процент прочитанного текста, на котором содержится вопросный факт. Из источников мы убрали информацию наподобие “8.62% текста”. Лутенко порадовался, что Иванов бережно относится к бумаге и своими вопросами пропагандирует подобный подход. Знак процента находится на клавише 5, которая в некоторых раскладках отвечает также за знак евро.

Источник:
    1. ЛОАВ
    2. http://db.chgk.info/search/questions/иванов%20текста/SU
    3. https://en.wikipedia.org/wiki/AltGr_key

Автор: Андрей Лутенко (Кишинёв)

Средняя оценка: 0

Комментарии


228. Вопрос 1.27: В постановке театра имени Пушкина члены та́бора, в который попадает Але́ко, увлечены музыкой, ведут себя фривольно и одеты по моде полувековой давности. В описании этой постановки Виктория Никифорова использует топоним. Напишите этот топоним.

Ответ: Вудсто́к.

Комментарий: Согласно рецензии, племя цыган, в которое попадает Алеко, – это такой своеобразный Вудсток, обитель хиппи, музыки и свободной любви. Вудстокский фестиваль состоялся в городке Бе́тел, но в историю всё равно вошло название места, где его планировалось провести изначально.

Источник: https://www.vedomosti.ru/newspaper/articles/2002/12/26/bremya-cygan

Автор: Павел Солахян (Ереван)

Средняя оценка: -0.03

Комментарии


229. Вопрос 5.17: Повторное исследование ЕЁ останков указывает на падение с большой высоты, что свидетельствует о силе старых привычек. Назовите ЕЁ.

Ответ: Лю́си.

Зачёт: точный ответ.

Комментарий: учёные из Техаса заново исследовали скелет австралопитека Лю́си. Множественные типичные переломы указывают, что причиной её смерти стало падение с большой высоты. Это свидетельствует, что переход от жизни в кронах к жизни на земле у наших предков занял несколько миллионов лет. Даже перейдя к наземному образу жизни, они регулярно забирались на деревья по разным надобностям, таким как добыча еды, необходимость осмотреть окрестности или ночёвка.

Источник: http://antropogenez.ru/single-news/article/589/

Автор: Александр Мудрый (Черновцы)

Средняя оценка: -0.04

Комментарии


230. Вопрос 7.26: Яхтсмен До́нчо Папа́зов признавался, что в ходе кругосветного путешествия в 1970-е годы передумал заходить в Колумбию, потому что Колумбия — единственная страна, для которой некая организация делала исключение. Какая фамилия входит в название этой организации?

Ответ: Ллойд.

Зачёт: точный ответ.

Комментарий: страховое агентство отказывается страховать следующие в Колумбию суда. По крайней мере, так было в конце семидесятых.

Источник: Папазова Ю., Папазов Д., Идакиев С. С Яной вокруг света. М., 1993. С. 105.

Автор: Александр Коробейников (Саратов)

Средняя оценка: -0.04

Комментарии


231. Вопрос 2.18: После смерти отца Эже́на Делакруа́ Талейра́н занял принадлежавший тому пост министра. Существует версия, что Талейран также был любовником матери художника. Одна из картин Делакруа называется “Автопортрет ПРОПУСК”. По легенде Владимира похоронили ПРОПУСК. Какие три слова мы дважды пропустили?

Ответ: в костюме Гамлета. Зачёт: в образе Гамлета.

Комментарий: После смерти отца Гамлета Клавдий занял его пост и разделил ложе с его женой. Есть версия, что Талейран был отцом Эжена и поддерживал его достаточно долго. Владимир Высоцкий играл Гамлета во всем черном, и возникла легенда, что он похоронен в костюме Гамлета.

Источник:
    1. https://en.wikipedia.org/wiki/Eug%C3%A8ne_Delacroix
    2. http://delakrua.ru/?type=page&page=c21f7aff-d679-46b6-9617-093f0338d0d6&item= a85e0c95-930a-480a-bc5e-f12d7b0c6ca5
    3. http://www.kulichki.com/masha/vysotsky/ovys/fond/plays/moj_gamlet.html
    4. http://www.irrkut.narod.ru/stati/Kulgavov/Glava-Kulgavov-7.htm

Автор: Алексей Полевой (Гомель)

Средняя оценка: -0.06

Комментарии


232. Вопрос 3.31: В чемпионском сезоне “Лестера” нападающий Джейми Варди забивал всем английским грандам, в том числе и прошлогоднему чемпиону, “Челси”. Дарио ВисмАра назвал Варди так же, как одного аристократа стали называть после 283 года. Назовите этого аристократа.

Ответ: Джейме Ланнистер.

Зачёт: Цареубийца, Kingslayer.

Комментарий: в победном сезоне Джейми Варди забивал всем членам “большой четверки” (Арсеналу, Ливерпулю, Манчестер Юнайтед и Челси), в каком-то смысле став “убийцей королей”. Челси в прошедшем сезоне уместно сравнить с безумным королем Эйрисом Таргариеном, которого убил Джейме Ланнистер. Летоисчисление в Вестеросе ведется от Завоевания Эйгона, и указанное событие произошло в 283 году.

Источник:
    1. статистика сезона АПЛ 2015-16, http://www.sports.ru/epl/calendar/?s=4951
    2. http://awoiaf.westeros.org/index.php/Aerys_II_Targaryen
    3. https://twitter.com/canigggia/status/719161748893364224

Автор: Григорий Алхазов (Кишинёв)

Средняя оценка: -0.06

Комментарии


233. Вопрос 6.12: В 1993 году фильм “Барабаниа́да” получил специальный приз фестиваля “Кинотавр”. По словам Дмитрия Быкова, приз был вручен за то, что в этом фильме ни разу не звучит слово “еврей”. Какое прилагательное мы пропустили в первом предложении?

Ответ: Немой.

Комментарий: По словам Быкова, в том году все остальные фильмы так или иначе затрагивали еврейскую тематику.

Источник: http://echo.msk.ru/programs/personalno/1307610-echo/ https://ru.wikipedia.org/wiki/Барабаниада

Автор: Владимир Грамагин (Нью-Йорк)

Средняя оценка: -0.06

Комментарии


234. Вопрос 6.42: Когда грузы по Волге возились на деревянных баржах, судовладельцы старались в первом рейсе на новом судне загрузиться ЕЮ. Папуасы получали ЕЕ, сжигая найденные на берегу куски дерева. Назовите ЕЕ.

Ответ: Соль.

Комментарий: Просоленное дерево лучше противостоит вредителям и гниению. Папуасы сжигали выловленное из океана и пропитанное морской водой дерево, соль оставалась в костре.

Источник: http://www.panspartak.narod.ru/libfiles/hum/hum7.html http://kedem.ru/pravda/20111117-sol/

Автор: Игорь Сульповар (Чикаго)

Средняя оценка: -0.06

Комментарии


235. Вопрос 1.15: Пожилой пёс из одного мультфильма пытается флиртовать с кошкой. Услышав её ответ, пёс произносит фразу, ставшую известной около 60 лет назад. Напишите эту фразу.

Ответ: “У каждого свои недостатки”

Зачёт: “Nobody’s perfect”

Комментарий: Кошка вполне резонно замечает, что она кошка. Сцена является отсылкой к фильму “В джазе только девушки” и ставшей уже классической финальной фразе.

Источник: “The Secret Life of Pets”, 2016.

Автор: Анвар Мухаметкалиев (Алматы)

Средняя оценка: -0.07

Комментарии


236. Вопрос 2.11: Из-за общественного мнения Оскар Уайльд был вынужден покинуть Великобританию и пересечь Ла-Манш. Какой роман, по мнению Питера Акройда, вскоре заменил писателю путеводитель?

Ответ: «Отверженные».

Комментарий: отверженный английским обществом Оскар Уайльд перебрался в Париж. Действие романа Виктора Гюго происходит в Париже и много внимания уделяется достопримечательностям города.

Источник: Питер Акройд «Завещание Оскара Уайльда» http://royallib.com/book/akroyd_piter/zaveshchanie_oskara_uaylda.html

Автор: Денис Рыбачук (Брест)

Средняя оценка: -0.08

Комментарии


237. Вопрос 7.6: Героиня песни, исполняемой Линдой Ро́нстадт, после разрыва с любимым часто веселится, но это веселье напускное. Улыбку героиня сравнивает с НИМ. Назовите ЕГО двумя словами, начинающимися на парные согласные.

Ответ: клоунский грим.

Зачёт: грим клоуна.

Источник: http://www.azlyrics.com/lyrics/lindaronstadt/tracksofmytears.html

Автор: Александр Коробейников (Саратов)

Средняя оценка: -0.08

Комментарии


238. Вопрос 7.30: Герой Юрия Андрухо́вича был таким кривлякой, что казалось даже странным, что он не ПРОПУСК. Забавно, что Леонид Фирсов, автор диссертации «Память у антропоидов», ПРОПУСК. Заполните пропуск четырьмя словами.

Ответ: родился в год Обезьяны.

Зачёт: точный ответ.

Комментарий: в персонаже было много обезьяньего. Фирсов исследовал поведение обезьян.

Источник:
    1. Андрухович Ю. Дванадцять обручiв http://flib.nwalkr.tk/b/102343/read
    2. https://ru.wikipedia.org/wiki/Фирсов,_Леонид_Александрович

Автор: Александр Коробейников (Саратов)

Средняя оценка: -0.08

Комментарии


239. Вопрос 1.38: С 1979 года до первой половины девяностых актёр Джордж Коул играл роль несгибаемого дельца Артура Дэйли. Дэйли нередко сравнивали с другим человеком, а Коул шутил, что продержался даже дольше. Назовите этого другого человека.

Ответ: [Маргарет] Тэтчер.

Комментарий: “Железная леди” Маргарет Тэтчер пришла к власти в том же году, когда на британских телеэкранах появился Артур Дэйли, но продержалась чуть меньше.

Источник:
    1. www.telegraph.co.uk/comedy/comedians/george-cole-arthur-daley-guide-life/
    2. https://ru.wikipedia.org/wiki/Тэтчер,_Маргарет

Автор: Павел Солахян (Ереван)

Средняя оценка: -0.1

Комментарии


240. Вопрос 2.20: Кто послужил прототипом для преступника по прозвищу “Скорпион” из детективного фильма "Грязный Гарри"?

Ответ: Зодиак.

Комментарий: “Зодиаком” называли действовавшего в 1960-е годы в США серийного убийцу. «Скорпион», один из зодиакальных знаков, отсюда и прозвище персонажа.

Источник: https://ru.wikipedia.org/wiki/Грязный_Гарри_(фильм)

Автор: Денис Рыбачук (Брест), Алексей Полевой (Гомель)

Средняя оценка: -0.13

Комментарии


241. Вопрос 5.9: Томас Манн использовал ЕГО в седьмой главе романа «Лотта в Ве́ймаре», в которой описан один день из жизни Гёте. Назовите ЕГО.

Ответ: поток сознания.

Зачёт: точный ответ.

Комментарий: центральным произведением литературы «потока сознания» считается «Улисс» Джойса, рассказывающий об одном дне из жизни Леопольда Блума.

Источник: http://lib.ru/INPROZ/MANN/lotta.txt_with-big-pictures.html Автор: Александр Мудрый (Черновцы)

Средняя оценка: -0.13

Комментарии


242. Вопрос 7.14: Герои сказки Сергея Горбунова находят грибное дерево, но оно оказывается обычной ЕЮ. Это выяснилось, когда пришла белочка. Назовите ЕЁ одним словом.

Ответ: липа.

Зачёт: точный ответ.

Комментарий: это была обычная липа, на которой белка развесила свои грибы.

Источник: http://segor.ru/gribnoe-derevo/

Автор: Сергей Шорин (Краснодар)

Средняя оценка: -0.13

Комментарии


243. Вопрос 1.45: Сюзанна Ли́нтон считает, что работы её соотечественника следует открывать, чтобы с удовольствием вспоминать приятные моменты прошлого. Эти работы Сюзанна сравнивает с НИМ. Назовите ЕГО тремя словами.

Ответ: вино из одуванчиков.

Комментарий: Цитата из повести: “А если он что-нибудь забудет – что ж, в по́гребе стоит вино из одуванчиков, на каждой бутылке выведено число, и в них — все дни лета, все до единого”. Сюзанна Линтон писала о Рэе Брэдбери.

Источник:
    1. http://suzannalinton.com/dandelionwine/
    2. https://ru.wikiquote.org/wiki/Вино_из_одуванчиков

Автор: Павел Солахян (Ереван)

Средняя оценка: -0.14

Комментарии


244. Вопрос 2.37: В дореволюционном российском флоте корабли редко получали имена в честь сухопутных военачальников. Назовите полководцев, в честь которых, тем не менее, были названы линейный корабль Балтийского флота и линейный корабль Черноморского флота.

Ответ: Александр Невский, Дмитрий Донской.

Зачёт: в любом порядке.

Комментарий: Наверное, флотские чины посчитали достаточным поводом наличия реки в титуле военачальника, чтобы отнести его не к армии, а к флоту. Но, скорее всего, дело в том, что оба военачальника были признаны святыми, а в честь святых корабли называли нередко. Нева, кстати впадает в Балтийское, а Дон относится к бассейну Чёрного моря.

Источник:
    1. https://lenta.ru/articles/2016/09/18/names/
    2. https://ru.wikipedia.org/wiki/Дмитрий_Донской_(линейный_корабль,_1809)
    3. https://ru.wikipedia.org/wiki/Александр_Невский_(линейный_корабль,_1787)

Автор: Алексей Полевой (Гомель)

Средняя оценка: -0.17

Комментарии


245. Вопрос 6.11: Внимание, в вопросе есть замены. Биограф пишет, что в годы перестройки, когда многие потеряли нравственные ориентиры, Гранин тоже оказался на грани. Какие два слова мы заменили?

Ответ: Распутин, распутье.

Комментарий: Валентин Распутин в указанный период действительно переживал творческий кризис и почти не писал.

Источник: http://www.peoples.ru/art/literature/prose/roman/rasputin/index1.html

Автор: Владимир Грамагин (Нью-Йорк)

Средняя оценка: -0.17

Комментарии


246. Вопрос 6.39: Автору вопроса, живущему в Бруклине, однажды целую ночь мешали спать громкие и разнообразные звуки. Никогда ещё автору вопроса так сильно не хотелось ПРОПУСК. Заполните пропуск названием романа.

Ответ: Убить пересмешника.

Источник: ЛОАВ

Автор: Вадим Барановский (Нью-Йорк)

Средняя оценка: -0.17

Комментарии


247. Вопрос 3.24: Дуплет. Два вопроса по 30 секунд обсуждения, ответы сдаются на одном бланке.
    1. В конце восьмидесятых продюсерам удалось ОТБИТЬ ЭТО, поскольку сборы значительно превысили бюджет. Какие слова мы заменили словами “ОТБИТЬ ЭТО”?
    2. Впервые призыв ОТБИТЬ ЭТО прозвучал в середине 2011 года. Какие слова мы заменили словами “ОТБИТЬ ЭТО”?

Ответ:
    1. окупить “Уолл-стрит”; 2. оккупировать Уолл-стрит.

Зачёт:
    1. окупить Уолл-стрит; 2. захватить Уолл-стрит.

Комментарий: бюджет фильма 1987 года “Уолл-стрит” — 15 миллионов долларов, а сборы составили 44 миллиона. Продюсеры “отбили” деньги, потраченные на его создание. Участники протестного движения Occupy Wall Street призывали отбить эту улицу у финансовой элиты, которую винили в бедах современного общества.

Источник:
    1. https://www.film.ru/articles/greed-good
    2. https://en.wikipedia.org/wiki/Adbusters

Автор: Максим Коцюруба (Кишинёв)

Средняя оценка: -0.2

Комментарии


248. Вопрос 1.13: Главный герой романа Эрне́ста Клайна по имени Уэйд Уоттс знает об отце лишь то, что тот любил ИХ. В статье Википедии о НИХ упоминается колонна Траяна. Назовите ИХ.

Ответ: комиксы.

Комментарий: Имена и фамилии многих культовых героев комиксов начинаются на одну и ту же букву.

Источник:
    1. Э. Клайн. Первому игроку приготовиться. http://flibusta.is/b/425985/read
    2. https://en.wikipedia.org/wiki/Comics

Автор: Анвар Мухаметкалиев (Алматы)

Средняя оценка: -0.21

Комментарии


249. Вопрос 1.14: Возлюбленным поэтессы Сильвии Плат был крепкий мужчина из английской деревенской семьи. Характеризуя своё увлечение, Плат в шутку упоминала аристократическую фамилию. Напишите эту фамилию.

Ответ: Чаттерлей.

Зачёт: Чаттерли.

Комментарий: Плат говорила, что Тед Хьюз был для неё своеобразным “любовником леди Чаттерлей”. По сюжету скандального романа жена баронета Чаттерлея влюбляется в молодого и сильного егеря.

Источник:
    1. https://literaryreview.co.uk/the-fox-aflame
    2. http://www.svoboda.org/a/27851982.html
    3. https://ru.wikipedia.org/wiki/Любовник_леди_Чаттерлей
    4. https://en.wikipedia.org/wiki/Mytholmroyd

Автор: Павел Солахян (Ереван)

Средняя оценка: -0.21

Комментарии


250. Вопрос 1.19: Вышедшая в середине 1940-х годов экранизация пьесы Шекспира “Генрих V” могла стать провальной. Успеху фильма сильно поспособствовало то, что он вышел в прокат через несколько дней после… Чего?

Ответ: Высадки в Нормандии.

Зачёт: Операции “Нептун”; Дня Д; D-Day.

Комментарий: Пьеса Шекспира рассказывает о битве при Азенкуре, когда англичане разбили французов. Воодушевлённые успехами армии союзников британцы толпами повалили в кинотеатр смотреть фильм.

Источник:
    1. http://player.bfi.org.uk/film/watch-henry-v-1944/
    2. http://www.litencyc.com/php/stopics.php?rec=true&UID=19432

Автор: Анвар Мухаметкалиев (Алматы)

Средняя оценка: -0.21

Комментарии


251. Вопрос 7.16: Вагра́м Папазя́н назвал Гамлета, не согласного с положением дел в Дании, ИКСОМ. Хотя первые ИКСЫ появились в Дании в первой трети XVI века. Какое слово мы заменили ИКСОМ?

Ответ: протестант.

Зачёт: точный ответ.

Комментарий: Гамлет протестовал против прихода к власти Клавдия, а Дания — одна из первых стран, где протестантизм был объявлен государственной религией.

Источник:
    1. Папазян В. По театрам мира. Л.; М., 1937. С. 300.
    2. https://ru.wikipedia.org/wiki/Фредерик_I_(король_Дании)
    3. https://ru.wikipedia.org/wiki/Реформация

Автор: Александр Коробейников (Саратов)

Средняя оценка: -0.22

Комментарии


252. Вопрос 3.18: В своей статье Пьер БрандА пишет о революционных идеях Яниса ВаруфАкиса, бывшего министра финансов Греции. БрандА вспоминает о манере Варуфакиса носить рубашку без галстука и проводит параллель с НИМИ. Назовите ИХ.

Ответ: санкюлоты.

Зачёт: кюлоты.

Комментарий: кюлоты были отличительной чертой аристократов, а революционно настроенные представители третьего сословия не носили их. От слов “без кюлот” произошло название “санкюлоты”. Вопрос БрандА “министр без галстука или без кюлот?” на французском имеет двойной смысл. Источник: http://www.lemonde.fr/idees/article/2015/02/12/un-ministre-grec-sans-cravate-ou-sans-culotte_4574910_3232.html

Автор: Сергей Гергележиу (Кишинёв)

Средняя оценка: -0.22

Комментарии


253. Вопрос 3.39: По совету деда этот невысокий человек пошел в солдаты, а в 1940 году стал капралом. Во время Второй Мировой, будучи членом одного из подразделений “Сражающейся Франции”, он участвовал в высадке на южном побережье и освобождении Тулона. В 1974 году указом президента он был произведен в маршалы. О ком идет речь?

Ответ: [Жан-Бедель] Бокасса.

Зачёт: по фамилии.

Комментарий: как известно, кумиром центральноафриканского диктатора был Наполеон. Провозгласив себя императором, он провел коронацию в точном соответствии с наполеоновской. В вопросе приведены другие параллели между этими двумя личностями — Бокасса был невысокого роста, побывал капралом (Наполеона называли “маленьким капралом”) и участвовал во взятии Тулона, прошел путь от солдата до маршала (в соответствии с заветом Наполеона о маршальском жезле в рюкзаке каждого солдата). Впрочем, маршальское звание он присвоил себе сам, уже будучи президентом.

Источник:
    1. https://en.wikipedia.org/wiki/Jean-Bédel_Bokassa
    2. http://www.peoples.ru/state/king/car/bokassa/
    3. https://en.wikipedia.org/wiki/Battle_of_Toulon_(1944)

Автор: Григорий Алхазов (Кишинёв) по идее Максима Мозуля (Мюнхен)

Средняя оценка: -0.24

Комментарии


254. Вопрос 5.33: Этот замок получил название в честь НЕЁ. Назовите ЕЁ.

Ответ: Урания.

Зачёт: точный ответ.

Комментарий: это замок-обсерватория Ти́хо Бра́ге Ура́ниборг. Обратите внимание на клумбы в виде звёзд. Урания – муза астрономии.

Источник: https://en.wikipedia.org/wiki/Uraniborg

Автор: Александр Кудрявцев (Николаев)

Средняя оценка: -0.26

Комментарии


255. Вопрос 1.25: Бо́льшую часть населения необычной нидерландской деревни Хо́гевей составляют пожилые люди, страдающие болезнью Альцгеймера и слабоумием. Какой фильм 1998 года упоминается в статье Википедии об этой деревне?

Ответ: “Шоу Трумана”.

Зачёт: “Шоу Трумэна”.

Комментарий: Пациенты Хогевея не знают, что они пациенты, а деревня – своеобразный дом престарелых. Все, кто работает в деревне, либо являются врачами, либо имеют специальную подготовку в области гериатрии.

Источник: https://en.wikipedia.org/wiki/Hogewey

Автор: Анвар Мухаметкалиев (Алматы)

Средняя оценка: -0.28

Комментарии


256. Вопрос 4.33: Уолт Дисне́й прилагал немало усилий для получения прав на экранизацию произведений о Мэ́ри По́ппинс и, чтобы угодить ИМ, терпел все бесчисленные капризы Паме́лы Тре́верс. Назовите нашего современника, который создал произведение под названием "ОНИ".

Ответ: (Роберт) Майлз.

Комментарий: ОНИ — это дети. Произведения о Мэ́ри По́ппинс очень нравились детям Уо́лта Дисне́я. Правда, добиться у своенравной писательницы прав на экранизацию Дисней смог только когда его дочери уже выросли.

Источник:
    1. https://www.kinopoisk.ru/film/8156/
    2. http://www.filmpro.ru/materials/26554
    3. https://en.wikipedia.org/wiki/Children_(Robert_Miles_song)
    4. https://en.wikipedia.org/wiki/Robert_Miles

Автор: Алексей Уланов (Баку)

Средняя оценка: -0.28

Комментарии


257. Вопрос 3.13: В недавнем материале Михаил КузИщев отмечает, что эксперименты ИКСА со взрывным электрическим звучанием изменили рок-музыку того времени, и упоминает ИГРЕК. Назовите ИКСА и ИГРЕК словами, которые начинаются на одни и те же две буквы.

Ответ: Дилан, динамит

Зачёт: Dylan, dynamite.

Комментарий: Кузищев утверждает, что в 1965 году Боб Дилан взорвал рок-музыку, когда взял в руки электрогитару. Этот материал посвящен присуждению Дилану Нобелевской премии по литературе. Как известно, динамит изобрел Альфред Нобель.

Источник: https://slovesnik.org/kopilka/stati/muzykalnyj-dinamit-boba-dilana.html

Автор: Максим Коцюруба (Кишинёв)

Средняя оценка: -0.29

Комментарии


258. Вопрос 6.9:

Раздаточный материал

    вот это девочка на <...> не в смысле что в стране <...> а в смысле что пикассо видит так шар

    Заполните пропуски в порошке. Напомним, что в порошке рифмуются только вторая и укороченная четвертая строки.

Ответ: Кубе, сигар.

Источник: http://poetory.ru/content/view?id=21000

Автор: Татьяна Стронгина (Чикаго)

Средняя оценка: -0.29

Комментарии


259. Вопрос 6.14: Мечта декабристов сбылась 28 февраля 2016 года. Какие две буквы мы заменили в неологизме?

Ответ: и, п.

Комментарий: Дикапристы – поклонники Ди Каприо, мечтавшие о присвоении ему Оскара.

Источник: https://snob.ru/magazine/entry/106587

Автор: Станислав Арбитман (Чикаго)

Средняя оценка: -0.29

Комментарии


260. Вопрос 6.26: В 2014 году один из патриархов британской рок-музыки стал ЗЕТОМ, а два года спустя он в восьмой раз стал ИКСОМ. Назовите ИКСА и ЗЕТА в правильном порядке.

Ответ: Отец, прадед.

Зачёт: Синонимичные ответы.

Комментарий: В 2014 году у 70-летнего Мика Джаггера родилась правнучка, а в 2016 – восьмой ребенок. В 2014 году он также в пятый раз стал ИГРЕКОМ, то есть дедом.

Источник: http://www.barryanddistrictnews.co.uk/news/14955821.The_many_children__grandchildren__and_great_grandchild__of_the_Rolling_Stones/

Автор: Игорь Шпунгин (Нью-Йорк)

Средняя оценка: -0.29

Комментарии


261. Вопрос 6.28: Карл Брюллов говорил, что сначала ОН покорял мир мечом, потом ОН покорял мир крестом, а потом ОН покорил мир наукой и искусством. Назовите ЕГО.

Ответ: Рим.

Комментарий: Брюллов, любивший Италию, считал, что наукой и искусством Рим продолжал завоевывать мир уже в его время. Троекратное повторение слова “мир” может служить подсказкой.

Источник: http://ogrik.ru/b/galina-konstantinovna-leonteva/karl-bryullov/16096/glava-vtoraya/2

Автор: Елена Лукаш (Чикаго)

Средняя оценка: -0.29

Комментарии


262. Вопрос 6.33: Картина Репина “Нищая. Девочка-рыбачка” вызывает умиление и жалость. Копия этой картины была изготовлена для спецпроекта “ТАКОЕ искусство”. Что мы заменили словом “такое”?

Ответ: Трогательное.

Комментарий: В рамках проекта изготавливаются рельефные копии картин для слепых. Картина и сама по себе трогательная.

Источник: http://newsbabr.com/irk/?IDE=150866

Автор: Юлия Кальменс (Чикаго)

Средняя оценка: -0.29

Комментарии


263. Вопрос 6.36:

Раздаточный материал

    Тест Бекдел для оценки предвзятости кинофильма: 1) В фильме есть хотя бы две женщины, 2) которые говорят друг с другом 3) о чём-либо кроме мужчин

    Героиня комикса 1985 года говорит, что последний фильм, удовлетворяющий требованиям теста, вышел шесть лет назад. Напишите название этого фильма, которое, по словам автора сценария, “внезапно вышло из пишущей машинки”.

Ответ: Чужой.

Зачёт: Чужая; Alien.

Комментарий: Этот тест был впервые сформулирован в 1985 году в комиксе художницы Элисон Бекдел.В то время женские персонажи редко играли активную роль в боевиках и приключенческих фильмах. Героиня комикса упоминает разговор космолётчиц Рипли и Ламберт, но в фильме есть и более запоминающийся диалог между Рипли и самкой Чужого. Эмбрион Чужого внезапно выходит из тела человека, проламывая грудную клетку. Источники: http://scifi.stackexchange.com/questions/49435/does-alien-actually-pass-the-bechdel-test https://en.wikipedia.org/wiki/Alien_(creature_in_Alien_franchise) https://en.wikipedia.org/wiki/Bechdel_test

Автор: Мара Могилевская (Чикаго)

Средняя оценка: -0.29

Комментарии


264. Вопрос 3.21:

Раздаточный материал

    КОШМАРЫ НОЧИ ПРОБЛЕМА ЗЛА ПРОЩЕНИЕ ГРЕХОВ ОПАСНОСТИ ИСКУШЕНИЯ ПОСТОЯНСТВО ЛЮБВИ

    Британский писатель Джеймс Ранси работает над циклом детективов о священнике Сидни Чемберсе. В середине двухтысячных Ранси во время съемок документального фильма провел целый год со знаменитым человеком. Назовите этого человека.

Ответ: [Джоан] Роулинг.

Зачёт: по фамилии.

Комментарий: с 2006 по 2007 год Ранси тесно общался с автором книг о Гарри Поттере, работая над фильмом “Джоан Роулинг: год из жизни”. Возможно, этот опыт впоследствии пригодился писателю: в настоящее время он работает над циклом “Грантчестерские тайны”. Мы раздали вам неполные названия книг этой серии — на самом деле они называются “Сидни Чемберс и кошмары ночи” и т.д.

Источник: https://en.wikipedia.org/wiki/James_Runcie

Автор: Григорий Алхазов (Кишинёв)

Средняя оценка: -0.3

Комментарии


265. Вопрос 4.42: Ву́ди А́ллен как-то сказал, что если бы решился на ЭТО, то единственным компромиссом стала бы «мама». Назовите ЭТО коротким или длинным словом.

Ответ: Тату.

Зачёт: Татуировка.

Комментарий: Вуди Аллен — режиссёр еврейского происхождения. Единственное слово, которое он был бы готов нанести в качестве татуировки, - мама.

Источник: https://www.novayagazeta.ru/articles/2016/05/11/68557-vudi-allen-i-ves-ego-dzhaz-vysshee-obschestvo-glazami-nyu-yorkskogo-metra

Автор: Алексей Уланов (Баку)

Средняя оценка: -0.31

Комментарии (1)


266. Вопрос 3.5: Британский музыкант и комик Митч Бенн задался целью подсчитать всех людей, которые претендовали на то, чтобы называться ИКСОМ. После этого Бенн расположил их в порядке обоснованности притязаний и пришел к выводу, что был бы в списке тридцать седьмым. Какие два слова, начинающиеся на парные согласные, мы заменили ИКСОМ?

Ответ: пятый битл.

Зачёт: точный ответ.

Комментарий: решив, что все эти люди не могут одновременно быть пятыми битлами, уроженец Ливерпуля Митч Бенн расположил их в порядке уменьшения “битловости”. Вышло, что всего битлов 36, поэтому Бенн стал называть себя тридцать седьмым битлом.

Источник:
    1. https://en.wikipedia.org/wiki/Fifth_Beatle
    2. https://en.wikipedia.org/wiki/Mitch_Benn

Автор: Максим Коцюруба (Кишинёв)

Средняя оценка: -0.33

Комментарии


267. Вопрос 1.24: Школьникам из одного русскоязычного романа казалось, что учитель физики своим пристальным взглядом словно просверливает их. Напишите прозвище этого учителя.

Ответ: Буравчик.

Комментарий: Своё прозвище учитель получил в честь правила буравчика в физике, а уже потом ученики обратили внимание на буравящий взгляд преподавателя.

Источник:
    1. http://www.e-reading.club/chapter.php/12922/3/Vorobeii_-_Cherepashkina_lyubov'.html
    2. https://ru.wikipedia.org/wiki/Правило_буравчика

Автор: Павел Солахян (Ереван)

Средняя оценка: -0.35

Комментарии


268. Вопрос 6.4: Врач Викентий Вересаев с грустью пишет, что во время визитов в типографию в конце XIX века ни разу не видел ИКСА-ИГРЕКА. При игре в ИГРЕКА “ИКС” получается из “кристалла”. Какие слова мы заменили на ИКС и ИГРЕК?

Ответ: Старик, наборщик.

Комментарий: Дышащие свинцовыми испарениями наборщики не доживали до старости.

Источник: В. Вересаев, Записки врача.

Автор: Владимир Грамагин (Нью-Йорк)

Средняя оценка: -0.35

Комментарии


269. Вопрос 6.15: В ответ на предложение Дмитрия Медведева переименовать кофе американо Леонид Каганов призвал к запрету иллюминаторов, чемпионатов и знатоков, а также посоветовал Медведеву не пить кофе ТАК. Как?

Ответ: Натощак.

Комментарий: Каганов предложил обходиться без слов, в которых упоминается НАТО.

Источник: https://lleo.me/dnevnik/2016/11/17.html http://www.politonline.ru/interpretation/22888229.html

Автор: Владимир Грамагин (Нью-Йорк)  

Средняя оценка: -0.35

Комментарии


270. Вопрос 7.43: Герой Алексея Поля́ринова, эксперт по античному искусству, в шутку соглашается СДЕЛАТЬ ЭТО, но только в сестерциях. Какие однокоренные слова мы заменили словами СДЕЛАТЬ ЭТО?

Ответ: взять взятку.

Зачёт: точный ответ.

Комментарий: герою предлагают взятку за то, что он признает фальшивку подлинным произведением. Тот отказывается и острит: логично, что античника интересуют только сестерции.

Источник: http://dystopia.me/pejzazh-s-padeniem-ikara-3

Автор: Александр Коробейников (Саратов)

Средняя оценка: -0.35

Комментарии


271. Вопрос 1.30: Острый аппендицит характеризуется многообразием симптомов, которые зачастую говорят о совсем иных болезнях. Доктор Иван Гре́ков сравнил аппендицит с РАКОМ. Какое слово мы заменили словом “РАК”?

Ответ: хамелеон.

Комментарий: аппендицит – это болезнь-хамелеон, которая часто маскируется под другие заболевания. Название одного созвездия мы заменили названием другого.

Источник:
    1. http://lapcenter.ru/lap/jekstrennaja_hirurgija/ostryj_appendicit.html
    2. https://ru.wikipedia.org/wiki/Список_созвездий_по_площади

Автор: Павел Солахян (Ереван)

Средняя оценка: -0.39

Комментарии


272. Вопрос 2.23:

Раздаточный материал

    Лауреаты Ордена Заслуг германского орла: Чарльз Линдберг Бенито Муссолини Генри Форд Франческо Франко

    Компания "Форд" занимается благотворительной деятельностью. В финансировании чего, по словам Евгения Антонюка́, "Форд" участвовал в начале 1990-х?

Ответ: [фильма] "Список Шиндлера".

Комментарий: Генри Форд придерживался резко антисемитской позиции, за что и получил нацистский орден, но впоследствии раскаялся. В качестве покаяния компания “Форд” до сих пор финансирует различные еврейские общественные организации и проекты, направленные против нацизма. Раздаточный материал - список (неполный, правда).

Источник:
    1. https://life.ru/t/история/884209/moi_drugh_--_ghitlier_samyie_znamienityie_ poklonniki_ natsizma
    2. https://ru.wikipedia.org/wiki/Орден_Заслуг_германского_орла

Автор: Алексей Полевой (Гомель)

Средняя оценка: -0.4

Комментарии


273. Вопрос 5.23:

Раздаточный материал

    Да, Пильграм уехал далеко. Он, вероятно, посетил и Гранаду, и Мурцию, и Альбарацин, – вероятно, увидел, как вокруг высоких, ослепительно белых фонарей на севильском бульваре кружатся бледные ночные бабочки; вероятно, он попал и в Конго, и в Суринам и увидел всех тех бабочек, которых мечтал увидеть.

    В рассказе «Пильгра́м» заглавный герой мечтает ловить редчайших бабочек в далеких странах. Наконец он собрал нужное количество денег и готов отправиться в путь. Комментируя розданный вам фрагмент, филолог Александр Долинин проводит параллель с произведением, написанным в конце XIX века. Назовите автора этого произведения.

Ответ: А́мброз Бирс.

Зачёт: по фамилии.

Комментарий: филолог обращает внимание на троекратно повторенное «вероятно». Подобным образом в классическом рассказе «Случай на мосту через Совиный ручей» девять раз повторяется глагол seem [сим] – «кажется», которым автор в некотором роде предупреждает читателя. В следующем предложении Набоков рассказывает, как жена нашла бездыханное тело Пильграма, который так и не успел никуда уехать.

Источник: http://arzamas.academy/mag/335-nabokov

Автор: Александр Мудрый (Черновцы)

Средняя оценка: -0.4

Комментарии


274. Вопрос 5.11: Когда после неточного ухода со стола отрыва, прыгун с трамплина попытался слегка сбалансировать своё положение в воздухе, Сергей Курдюков пошутил, что спортсмен исполняет ЕЁ. Отец главной героини сериала «ОНА» попадает в долговую кабалу к барону. Назовите ЕЁ тремя словами.

Ответ: цыганочка с выходом.

Зачёт: «Цыганочка с выходом».

Комментарий: лёгкая коррекция достигается движениями плеч вперёд-назад, что немного похоже на элемент знаменитого танца. Барон в сериале, разумеется, цыганский.

Источник: Трансляция соревнований в Гармиш-Партенкирхене на канале «Евроспорт-Россия», эфир от 1 января 2017 года. http://www.kino-teatr.ru/kino/movie/ros/15366/annot/

Автор: Александр Мудрый (Черновцы)

Средняя оценка: -0.45

Комментарии


275. Вопрос 5.35: В пьесе Джорджа Фа́ркера ОН сулит мяснику должность главного хирурга и жалованье пятьсот фунтов в год. В современном сериале ОН спрашивает подростков о любви к видеоиграм. Назовите ЕГО.

Ответ: вербо́вщик.

Зачёт: рекрутёр; офицер-вербовщик.

Комментарий: в пьесе Фаркера «Офицер-вербовщик» заглавный герой пытается завербовать мясника, обещая ему должность главного хирурга армии и огромный оклад. Любители шутеров и авиасимуляторов могут стать находкой для современных вербовщиков.

Источник:
    1. http://lib.ru/INOOLD/FARKER_J/farker1_1.txt
    2. Сериал «Два с половиной человека», s9e24.

Автор: Александр Мудрый (Черновцы)

Средняя оценка: -0.45

Комментарии


276. Вопрос 7.20: Писатель Яа Гья́си отмечает, что в современном мире каждый хочет верить, что непременно прятал бы на чердаке еврейскую семью или организовал бы остановку на НЕЙ. Назовите ЕЁ тремя словами.

Ответ: подземная железная дорога.

Зачёт: точный ответ.

Комментарий: а вот с рабовладельцами и линчевателями никто себя как-то не отождествляет. Подземная железная дорога — система переправки негритянских рабов с юга на север США.

Источник: https://www.theguardian.com/books/2017/jan/20/why-the-next-four-years-under-trump-will-be-a-test

Автор: Александр Коробейников (Саратов)

Средняя оценка: -0.45

Комментарии


277. Вопрос 6.7: В рамках американского проекта “Жизнь при кинетической эволюции” созданы бионические протезы для людей, лишившихся руки. Напишите фамилию человека, в честь которого эти протезы были названы.

Ответ: Скайуокер.

Зачёт: Skywalker.

Комментарий: название проекта – Life Under Kinetic Evolution, сокращенно LUKE. В фильме “Звездные войны. Эпизод 5: Империя наносит ответный удар” Люк Скайуокер лишился руки в поединке с Дартом Вейдером и получил взамен протез.

Источник: http://www.techtimes.com/articles/169458/20160712/the-mind-controlled-bionic-arm-named-after-luke-skywalker-will-finally-launch-this-year.htm

Автор: Владимир Грамагин (Нью-Йорк)

Средняя оценка: -0.48

Комментарии


278. Вопрос 6.23: Португалия славится монастырями, где издревле элементом монашеского одеяния был белый воротник. Неудивительно, что другой монастырской традицией стали пирожные паште́л-де-беле́н, в которых основным ингредиентом начинки являются ОНИ. Назовите ИХ.

Ответ: [Яичные] желтки.

Комментарий: Белки используются вместо крахмала для накрахмаливания белья, желтки остаются.

Источник: https://en.wikipedia.org/wiki/Pastel_de_nata

Автор: Элла Вайсман (Чикаго)

Средняя оценка: -0.48

Комментарии


279. Вопрос 7.13: Герой книги Андрея Аствацату́рова однажды выбирал, в какой российский город ему поехать, и почувствовал, что его неудержимо тянет... Куда?

Ответ: в Магнитогорск.

Комментарий: как магнитом потянуло. Вроде выбрался удачно.

Источник: Аствацатуров А. А. Скунскамера http://proxy.flibusta.is/b/222983/read

Автор: Александр Коробейников (Саратов)

Средняя оценка: -0.5

Комментарии


280. Вопрос 1.28: Маня Рубенште́йн вспоминала, что, выглянув в окно 15 апреля, поначалу подумала, что находящиеся на улице сразу двумя руками ДЕЛАЮТ ЭТО. Что именно?

Ответ: показывают нацистское приветствие

Зачёт: показывают нацистский салют, кидают зигу, зигуют.

Комментарий: Был 1945 год, и британские войска вошли в концлагерь, чтобы освободить пленных. Сдававшиеся немецкие солдаты, естественно, подняли руки вверх. Рубенштейн рассказывала, что из-за постоянного недоедания узники плохо соображали и не сразу смогли понять, что их освобождают.

Источник: http://www.jweekly.com/article/full/23003/how-the-holocaust-rocked-rush-front-man-geddy-lee/

Автор: Анвар Мухаметкалиев (Алматы)

Средняя оценка: -0.51

Комментарии


281. Вопрос 1.35: В 2008 году американская группа “The National” [зэ нэшнл] выпустила футболки с названием своей песни “Mr. November” [мистер нове́мбер]. В интервью лидер группы сказал, что на самом деле песня была написана несколькими годами ранее и была посвящена Джону. Напишите фамилию Джона.

Ответ: Керри.

Комментарий: The National активно поддерживали кандидатуру Барака Обамы во время президентских выборов 2008 года. Название песни отсылает к тому факту, что выборы президента в США традиционно проводятся в ноябре. Изначально песня была написана в 2004 году о тогдашнем кандидате демократов Джоне Керри.

Источник: http://genius.com/The-national-mr-november-lyrics

Автор: Анвар Мухаметкалиев (Алматы)

Средняя оценка: -0.51

Комментарии


282. Вопрос 1.4: В фильме 1918 года персонаж Чарли Чаплина после каждого ИКСА делает отметку мелом. Впрочем, когда с его головы слетает каска, он стирает одну из отметок. Какое слово мы заменили ИКСОМ?

Ответ: выстрел.

Комментарий: Комедия “На плечо!” снималась во время Первой мировой войны. Персонаж Чаплина отстреливает врагов, и отмечает каждого убитого. Впрочем, когда ответным выстрелом ему сбивает шляпу с головы, он понимает, что немного поторопился.

Источник: https://www.youtube.com/watch?v=4FtNH7oPgT8

Автор: Анвар Мухаметкалиев (Алматы)

Средняя оценка: -0.52

Комментарии (2)


283. Вопрос 6.44: Внимание, в этом вопросе слова “ПОТ” и “ПОЭТ” – замены. Американцев называют “нацией ПОТА”, в частности потому, что каждый год около 11% американцев подвергаются атакам. Подобная атака настигла великого русского ПОЭТА после завершения многолетнего труда. Какие слова мы заменили?

Ответ: Прозак, прозаик.

Комментарий: Американцы потребляют огромное количество транквилизаторов, так как из-за постоянного стресса подвержены психическим расстройствам, в частности паническим атакам. Лев Толстой испытал состояние, совпадающее по симптомам с панической атакой и известное в литературоведении как “Арзамасский ужас”, в 1869 году после шестилетней работы над “Войной и Миром”.

Источник: https://en.wikipedia.org/wiki/Prozac_Nation https://en.wikipedia.org/wiki/Panic_attack http://windowrussia.ruvr.ru/2013_08_19/Arzamasskij-uzhas-3926/ https://www.b17.ru/article/26577/

Автор: Елена Колезева (Чикаго)

Средняя оценка: -0.55

Комментарии


284. Вопрос 3.37: [Ведущему: максимально четко прочитать название города.] В штате Колорадо на реке Арканзас есть город, который раньше назывался Кэнон-Сити. В 1994 году Совет США по географическим названиям официально зафиксировал отличительную черту этого города, присвоив ему... Какое название?

Ответ: Cañon City.

Зачёт: Кэньон-Сити, Каньон-Сити.

Комментарий: рядом с городом есть каньон ручья Грейп-Крик, который впадает в Арканзас. Благодаря этому каньону город и получил свое первое название. Его третья буква — испанская “энье”, которая обозначается как N с тильдой, волнистой чертой. При закреплении названия тильда потерялась, но в 1994 году городу было возвращено правильное название.

Источник: https://en.wikipedia.org/wiki/Cañon_City,_Colorado

Автор: Григорий Алхазов (Кишинёв)

Средняя оценка: -0.61

Комментарии


285. Вопрос 2.24: Майкл Ханн считает, что ПРОПУСК "Принц" и "Питер Гэ́бриэл" достойны места в Национальной портретной галерее. Какие два слова мы пропустили в тексте вопроса?

Ответ: обложки альбомов. Зачёт: обложки пластинок, обложки дисков.

Комментарий: Как можно догадаться по названиям альбомов, на их обложках изображены портреты исполнителей. По мнению журналиста, они настолько выразительны, что заслуживают места в одном из главных музеев Великобритании.

Источник: https://www.theguardian.com/music/gallery/2015/apr/29/bjork-blondie-and-bruce- the- cover-portraits-that-deserve-to-hang-in-a-gallery

Автор: Алексей Полевой (Гомель)

Средняя оценка: -0.62

Комментарии


286. Вопрос 6.27: В начале XVII века вице-король Перу сообщил в метрополию об излечении своей супруги, графини Чинчо́н. А чем она болела?

Ответ: Малярией.

Зачёт: Болотной лихорадкой, перемежающейся лихорадкой.

Комментарий: Графиню Чинчон вылечили корой чудо-дерева, которое, по одной из версий, в ее честь и было названо хиной. Источник:https://ru.wikipedia.org/wiki/Хинин https://ru.wikipedia.org/wiki/Кабрера,_Луис_Херонимо_де https://ru.wikipedia.org/wiki/Малярия

Автор: Владимир Грамагин (Нью-Йорк)

Средняя оценка: -0.62

Комментарии


287. Вопрос 6.35: Размышляя о роли женщины в искусстве, персонаж Торнтона Уайлдера нашел определенную закономерность: женщины совершают ошибки, что приводит сначала к смерти их отцов и братьев, то есть ПЕРВЫХ и ВТОРЫХ, а затем к гибели ТРЕТЬИХ. Ну а к половине одиннадцатого погибает и сама героиня. Назовите ПЕРВЫХ, ВТОРЫХ и ТРЕТЬИХ в любом порядке.

Ответ: Басы, баритоны, теноры.

Комментарий: Так герой описал типичный сюжет оперы. Оперные спектакли в описываемое время начинались вечером и кончались ближе к полуночи.

Источник: Торнтон Уайлдер, День Восьмой

Автор: Владимир Грамагин (Нью-Йорк)

Средняя оценка: -0.62

Комментарии


288. Вопрос 1.32: Главный герой одной песни поёт, что “видит то, чего не видел раньше”. В названии песни упоминается температура плавления… Чего?

Ответ: воска.

Комментарий: песня поётся от лица Икара.

Источник: http://www.azlyrics.com/lyrics/thrice/themeltingpointofwax.html

Автор: Анвар Мухаметкалиев (Алматы)

Средняя оценка: -0.63

Комментарии


289. Вопрос 2.8: Персонаж компьютерной игры - кот-пират по имени Натаниэ́ль Джо́зеф. Собаки какой породы берут его в плен?

Ответ: спаниель. Зачёт: кокер-спаниель.

Комментарий: по задумке авторов капитан Коготь - кот британской породы, а спаниели, очевидно, представляют испанский флот.

Источник:
    1. https://www.youtube.com/watch?v=wtbK1S2QV8E
    2. https://ru.wikipedia.org/wiki/Claw

Автор: Вячеслав Кочурко (Брест-Париж)

Средняя оценка: -0.65

Комментарии


290. Вопрос 7.36: Писательница Джойс Кэрол Оутс критически относится к Дональду Трампу и обычно использует ИХ в фамилии Трамп. В определённом случае ОНИ соответствуют годам. Назовите ИХ.

Ответ: звёздочки.

Зачёт: звёзды, три звёздочки, три звезды.

Комментарий: Оутс считает фамилию президента нецензурным словом. Три звёздочки на коньяке — показатель того, что он выдерживался в дубовых бочках не менее трёх лет.

Источник:
    1. https://lareviewofbooks.org/article/sowing-wild-oates/
    2. https://test.org.ua/usefulinfo/food/info/128

Автор: Александр Коробейников (Саратов)

Средняя оценка: -0.65

Комментарии


291. Вопрос 1.40: По одной из версий, английский народный танец мо́ррис зародился в XV веке после выступления иностранных танцоров перед Генрихом VII. Этимологию названия танца чаще всего связывают с НИМИ. Назовите ИХ.

Ответ: мавры.

Зачёт: мориски.

Комментарий: Скорее всего, название “моррис” родственно танцу эпохи Возрождения мореска, в котором гротескно изображались мавры.

Источник: https://en.wikipedia.org/wiki/Morris_dance

Автор: Анвар Мухаметкалиев (Алматы)

Средняя оценка: -0.68

Комментарии


292. Вопрос 7.5: Во время путешествия немецкого исследователя Баумана по Африке часть вещей было решено закопать. Проводники резонно предложили СДЕЛАТЬ ЭТО над схроном, чтобы впоследствии найти его было легче. Когда герой французского писателя ДЕЛАЕТ ЭТО, автор упоминает алую пасть. Что такое ДЕЛАТЬ ЭТО?

Ответ: разводить костёр.

Зачёт: «разжигать» вместо «разводить»; «огонь» вместо «костра».

Комментарий: по золе затем определить местонахождение зарытых вещей будет проще. Алую пасть костра в книге «Борьба за огонь» описывает Жозе́ф Рони́-старший.

Источник:
    1. Родригес де ла Фуэнте Ф. Африканский рай http://animalkingdom.su/books/item/f00/s00/z0000022/st015.shtml
    2. Рони-ст. Ж. Борьба за огонь http://www.e-reading.club/bookreader.php/1028618/Roni-starshiy_-_Borba_za_ogon._Peschernyy_lev.html

Автор: Александр Коробейников (Саратов)

Средняя оценка: -0.71

Комментарии


293. Вопрос 3.43: Рассказывая о безвыходном положении советских евреев, Майкл ШЕрборн называл их “узниками”. Какие три буквы мы пропустили в предыдущем предложении?

Ответ: реф.

Зачёт: в любом порядке.

Комментарий: в Советском Союзе граждан, получивших от властей отказ в разрешении на выезд, называли откАзниками. Майкл Шерборн, который помог эмигрировать сотням таких людей, ввел в международный лексикон слово “refusenik” (на русский его транслитерируют двумя способами — “рефьюзник” или “рефузник”).

Источник:
    1. https://esquire.ru/2014-sherbourne
    2. https://goo.gl/tccvKE

Автор: Григорий Алхазов (Кишинёв)

Средняя оценка: -0.73

Комментарии


294. Вопрос 2.15: Илья Ильф с упрёком писал из Москвы возлюбленной в Одессу, что, видимо, убиты все ... Кто?

Ответ: почтальоны.

Комментарий: писатель долго не мог дождаться ответного письма.

Источник: Александра Ильф «Дом, милый дом» http://tfile.co/forum/viewtopic.php?t=840279

Автор: Денис Рыбачук (Брест)

Средняя оценка: -0.76

Комментарии


295. Вопрос 2.30: Для роли в “Списке Шиндлера” британскому актеру Рэ́йфу Фа́йнзу нужно было набрать больше десяти килограмм веса. Он это сделал, хоть и не в рекордные сроки, но достаточно быстро. Ответьте именем собственным, с помощью чего?

Ответ: [пива] Гиннесс. Зачёт: Guinness.

Комментарий: британский актер с удовольствием стал пить много пива Guinness. Считается, что с помощью пива легко набрать вес, но, возможно, дело в хорошей закуске.

Источник: https://ru.wikipedia.org/wiki/Список_Шиндлера

Автор: Алексей Полевой (Гомель)

Средняя оценка: -0.76

Комментарии


296. Вопрос 2.32: В 1926 году торговец металлоломом Эрнест Кокс, чтобы сэкономить, начал брать уголь для своего флота с НИХ. К ним, например, относится "Юта". Назовите ИХ двумя словами.

Ответ: затонувшие корабли.

Комментарий: Кокс поднимал затонувшие корабли и сдавал их на лом чёрного металла. "Юта" - один из затонувших в Перл-Харборе американских кораблей, которые зачастую называют по именам штатов.

Источник:
    1. Дж. Горз. Подъем затонувших кораблей. http://lib.ru/HISTORY/GORZ/pod_em_korablej.txt
    2. http://free-eyes.com/zatonuvshie-korabli/

Автор: Денис Рыбачук (Брест)

Средняя оценка: -0.76

Комментарии


297. Вопрос 3.30: На альбоме “Бит” группы “Кинг КрИмсон” есть песня, которая написана от лица автомобиля. В ее названии упомянуты два человека. Напишите фамилию любого из них.

Ответ: КеруАк.

Зачёт: Кэссиди; Пэрэдайз; Мориарти.

Комментарий: альбом, как можно предположить из названия, посвящен творчеству бит-поколения. Песня называется “Нил и Джек и я”. Имеются в виду Джек Керуак, автор романа “В дороге”, и его друг Нил Кэссиди. Керуак выведен в романе под именем Сэл Пэрэдайз, Кэссиди — прототип Дина Мориарти.

Источник:
    1. https://en.wikipedia.org/wiki/Beat_(album)
    2. http://www.lyricsfreak.com/k/king+crimson/neal+jack+me_20078573.html
    3. https://ru.wikipedia.org/wiki/Кэссиди,_Нил

Автор: Максим Коцюруба (Кишинёв)

Средняя оценка: -0.76

Комментарии


298. Вопрос 3.23: Продолжая свою прогулку, герой Юрия Иофе фантазирует, как при звуках, извлекаемых из НИХ, на кладбище оживают и поднимаются трупы. В этом вопросе слово “ОНИ” заменяет три слова, одно из которых — имя собственное. Напишите это имя собственное.

Ответ: ПомпидУ.

Зачёт: точный ответ.

Комментарий: парижский Центр Помпиду знаменит выведенными наружу разноцветными трубами. Автор уподобляет трубы Центра Помпиду трубам Страшного Суда, при звуке которых мертвые поднимутся из могил.

Источник: http://lib.ru/POEZIQ/IOFE_J/stihi.txt

Автор: Максим Коцюруба (Кишинёв)

Средняя оценка: -0.8

Комментарии


299. Вопрос 6.10: Исторический мюзикл, поставленный на Бродвее в 2015 году, получил рекордные сборы – более 3 миллионов долларов за неделю. Популярность этого мюзикла помогла министру финансов США сохранить свое место. Где именно?

Ответ: На купюре [$10].

Зачёт: По упоминанию денег без неверных уточнений.

Комментарий: Портрет министра финансов США Гамильтона на 10-долларовой купюре предлагалось заменить на портрет чернокожей абилиционистки Гарриет Табмен, но популярность мюзикла “Гамильтон” помогла этого избежать. Возможно, Табмен заменит Эндрю Джексона на купюре $20. Мюзикл 2008 года “Кровавый Эндрю Джексон” был не столь популярен.

Источник: https://www.washingtonpost.com/news/wonk/wp/2016/04/20/u-s-to-keep-hamilton-on-front-of-10-bill-put-portrait-of-harriet-tubman-on-20-bill/?utm_term=.1e1430b29b59 https://en.wikipedia.org/wiki/Hamilton_(musical)

Автор: Мара Могилевская (Чикаго)

Средняя оценка: -0.84

Комментарии


300. Вопрос 6.30: Основатель компании Netflix [не́тфликс] Рид Хастингс рассказывал, что идея брать с клиентов фиксированную абонентскую плату, не зависящую от частоты и продолжительности просмотра фильмов, пришла ему в голову по пути в место, в котором он мог бы бывать и чаще. Назовите это место.

Ответ: Тренажерный зал.

Зачёт: Спортзал, фитнесс-центр и синонимичные ответы.

Комментарий: Большинство тренажерных залов работают по этой бизнес-модели, а для видеопроката она была новинкой.

Источник: http://www.nytimes.com/2006/12/17/jobs/17boss.html?_r=0

Автор: Григорий Остров (Чикаго)

Средняя оценка: -0.84

Комментарии


301. Вопрос 7.29: В этом вопросе АЛЬФА является заменой. В главе о доблести и мужестве Герцен сравнил с АЛЬФОЙ Гарибальди, хотя с 1840 года АЛЬФОЙ логичнее называть другого революционера. Кого?

Ответ: [Тадеуша] Костюшко.

Зачёт: Косцюшко.

Комментарий: в честь Костюшко названа самая высокая точка Австралии.

Источник:
    1. А. Герцен. Былое и думы https://biography.wikireading.ru/43348
    2. http://ru.wikipedia.org/wiki/Костюшко,_Тадеуш

Автор: Александр Сидоренков (Смоленск)

Средняя оценка: -0.88

Комментарии


302. Вопрос 6.31: В самодеятельном спектакле по мотивам “Золушки” были использованы песни из репертуара Аллы Пугачевой. Например, король пел “Все могут короли”, а мачеха обращалась к Золушке со словами “Делу время, а потехе – час”. А какую песню пел персонаж, которого в оригинальной сказке Перро не было?

Ответ: “Волшебник-недоучка”.

Комментарий: Основой для спектакля послужила не сказка Перро, а советский фильм по сценарию Евгения Шварца. Для Пажа с его репликой “Я не волшебник, я только учусь” прекрасно подошла песня волшебника-недоучки.

Источник: http://gostrov.livejournal.com/105801.html

Автор: Григорий Остров (Чикаго)

Средняя оценка: -0.92

Комментарии


303. Вопрос 6.38: В рассказе Чехова помещик, находясь в вынужденном одиночестве, изнывает от скуки. Далее описывается противостояние АЛЬФЫ и БЕТЫ. Назовите автора произведения 1969 года, в заглавии которого упоминается БЕТА.

Ответ: [Урсула] ле Гуин.

Комментарий: Помещик, отрезанный от соседей половодьем, находит шашечную доску и играет сам с собой – правая рука против левой. “Левая рука Тьмы” – один из самых известных романов Урсулы ле Гуин.

Источник: А.П. Чехов,”Безнадежный”. https://ru.wikipedia.org/wiki/Левая_рука_Тьмы

Автор: Владимир Грамагин (Нью-Йорк)

Средняя оценка: -0.92

Комментарии


304. Вопрос 7.21: Согласно подписи к карикатуре XIX века, ОН демонстрирует то, чего не позволяет показать скромность. В честь какого из НИХ в одной компьютерной игре назван ледокол?

Ответ: Борей.

Зачёт: точный ответ.

Комментарий: на карикатуре в самых интересных местах ветром сдувает платья. В данном случае это северный ветер Борей. Логично, что в честь северного ветра назван ледокол.

Источник:
    1. http://www.britishmuseum.org/research/collection_online/collection_object_details.aspx?objectId=1467947&partId=1
    2. https://ru.wikipedia.org/wiki/Локации_в_играх_серии_Half-Life

Автор: Александр Коробейников (Саратов)

Средняя оценка: -0.94

Комментарии


305. Вопрос 1.42: Описывая заслуги Иосифа Бродского в критике тоталитаризма, Лев Ло́сев упоминает его кумиров – Джорджа О́руэлла, Уи́стена О́дена и Че́слава Ми́лоша. Каким словом греческого происхождения Лосев называет последних?

Ответ: гиганты.

Комментарий: Лосев считает, что Бродский в данном вопросе стоял на плечах гигантов, тем самым делая отсылку к знаменитому высказыванию Исаака Нью́тона.

Источник:
    1. http://www.e-reading.club/chapter.php/94234/83/Losev_-_Iosif_Brodskiii.html
    2. https://ru.wikiquote.org/wiki/Исаак_Ньютон

Автор: Павел Солахян (Ереван)

Средняя оценка: -0.95

Комментарии


306. Вопрос 3.20: ИевосфЕй, недолго бывший царем, — ОН. Генрих АльтшУллер, автор теории решения изобретательских задач, — тоже ОН. Еще один “ОН” основан, помимо прочего, на воспоминаниях доктора МИклоша НИсли. Какие два слова, начинающиеся на одну и ту же букву, мы заменили местоимением ОН?

Ответ: сын Саула.

Зачёт: “Сын Саула”.

Комментарий: после гибели Саула царем Израиля был провозглашен его сын Иевосфей, позже убитый своими же военачальниками, которые надеялись получить за это награду от Давида. Отчество Генриха Альтшуллера — Саулович. Миклош Нисли — венгерский еврей, бывший врачом и членом зондеркоманды в Освенциме. На его воспоминания ссылается Ласло Немеш, сценарист и режиссер венгерской картины “Сын Саула”, которая в 2016 году удостоилась Оскара как лучший фильм на иностранном языке.

Источник:
    1. http://www.filmsdistribution.com/Handlers/HTFile.ashx?MEDIAID=28350
    2. https://en.wikipedia.org/wiki/Genrich_Altshuller
    3. https://ru.wikipedia.org/wiki/Иевосфей
    4. https://ru.wikipedia.org/wiki/Нисли,_Миклош

Автор: Григорий Алхазов (Кишинёв)

Средняя оценка: -0.96

Комментарии


307. Вопрос 6.34: После воздушного боя 1 августа 1943 года командование представило летчицу Лидию Литвя́к к званию Героя Советского Союза. Однако указ о награждении был подписан только через много лет, так как до 1969 года у Литвяк не было официальной ЕЕ. ОНА – прозвище криминального авторитета, убитого в 2003 году Назовите ЕЕ.

Ответ: Могила.

Комментарий: Существовала практика не присваивать звание Героя пропавшим без вести. Останки Литвяк нашли в 1969 году и захоронили в братской могиле. В 1988 ее имя появилось на надгробии, а в 1990 Горбачев наконец подписал указ о награждении. Костя Могила свел в могилу многих людей, но и сам нашел себе могилу.

Источник: https://ru.wikipedia.org/wiki/Литвяк,_Лидия_Владимировна http://www.spb.aif.ru/archive/1797605

Автор: Григорий Остров (Чикаго)

Средняя оценка: -1

Комментарии


308. Вопрос 2.5: Форе́йтором в упряжке обычно был мальчик или подросток. Поэтому ТАКОЙ форейтор у Ла́риных выглядел анекдотом, показывающим их провинциальность. Какой “такой”?

Ответ: бородатый.

Комментарий: "На кляче тощей и косматой Сидит форейтор бородатый" Ларины безвыездно сидели в деревне и далёких путешествий не предпринимали. И вот – поездка в Москву. Где уж тут обучать нового форейтора! Они и взяли старого, который ездил ещё лет 15–20 назад. Слово “анекдот” - небольшая подсказка.

Источник: 1.Пушкин А. Евгений Онегин
    2. http://festival.1september.ru/articles/311078/

Автор: Алексей Полевой (Гомель)

Средняя оценка: -1.06

Комментарии (1)


309. Вопрос 6.29: Восхищаясь игрой нападающего “Арсенала” в одном из последних матчей 2016 года, российский комментатор отметил, что этот Жюльен – лучший на этой неделе. Возможно, он немного поторопился. Что мы заменили на “Жюльен”?

Ответ: Оливье.

Комментарий: Оливье Жиру забил чудо-гол пяткой 26 декабря. Забегая вперед, комментатор решил, что лучше этого Оливье в новогодний период не будет уже ничего. Как и оливье, жюльен – и французское имя, и кушанье.

Источник: Обзор матчей премьер-лиги на канале Матч-Футбол I

Автор: Владимир Грамагин (Нью-Йорк)

Средняя оценка: -1.09

Комментарии


310. Вопрос 5.26: Вы будете смеяться, но Доктору Уилларду Блиссу, лечившему президента Гарфилда, вполне могли задать вопрос, который Холмс задал Ватсону. Каким словом завершался этот вопрос?

Ответ: Доктор.

Зачёт: точный ответ.

Комментарий: врача, лечившего Гарфилда, звали Доктор Уиллард Блисс. В известном анекдоте Шерлок Холмс спрашивал: «Ватсон, почему у вас такое странное имя – Доктор?».

Источник:
    1. https://en.wikipedia.org/wiki/Doctor_Willard_Bliss
    2. https://www.anekdot.ru/id/803445/

Автор: Александр Кудрявцев (Николаев)

Средняя оценка: -1.13

Комментарии


311. Вопрос 4.37: Сэм Са́рджант — запасной вратарь клуба «Ле́йтон О́риент», аутсайдера четвёртой по силе лиги Англии. Рассказывая об одном из своих увлечений, Са́рджант признается, что недавно исполнилась его детская мечта, хоть он и огорчен тем, что стал И́КСОМ в этом увлечении. Назовите И́КСА двумя словами, начинающимися на соседние буквы алфавита.

Ответ: Худший футболист.

Комментарий: Сэм Са́рджант — самый слабый футболист в последней версии футбольного симулятора FIFA 17. Чтобы добиваться успехов в симуляторе, Сарджант даже не ставит себя в состав, предпочитая игроков с лучшими параметрами.

Источник: https://www.sports.ru/tribuna/blogs/barcelonaworld/1138554.html

Автор: Вюга́р Дада́шев (Баку)

Средняя оценка: -1.16

Комментарии


312. Вопрос 6.6: Статьи Википедии о распространенной саванной и более редкой пустынной разновидностях дают противоположные ответы на этот вопрос. Во втором случае ответ имеет под собой некоторое основание, так как у пустынной разновидности ОНО. Назовите ЕГО двумя словами, начинающимся на одну и ту же букву.

Ответ: Белое брюхо.

Комментарий: Если верить русской Википедии, самая распространенная бурчеллова, или саванная зебра – черная с белыми полосками, а пустынная зебра, или зебра Греви́ – белая, с черными полосками, оставляющими живот белым. Более светлое, чем спина, брюхо характерно не только для зебр Греви, но и для многих других животных.

Источник: https://ru.wikipedia.org/wiki/Зебра_Греви https://ru.wikipedia.org/wiki/Бурчеллова_зебра

Автор: Григорий Остров (Чикаго)

Средняя оценка: -1.27

Комментарии


313. Вопрос 6.37: В спектакле под названием “А если бы они СДЕЛАЛИ ЭТО?” заняты актрисы Изабель Тексе́йра, Стелла Рабе́лло и Джулия Берна́т. Напишите два последних слова из трех пропущенных в названии спектакля.

Ответ: В Москву.

Комментарий: Спектакль по пьесе “Три сестры” называется “А если бы они уехали в Москву?”. Несмотря на то, что действие пьесы перенесено в Бразилию, героини по-прежнему стремятся в Москву.

Источник: http://inosmi.ru/world/20141005/223440518.html

Автор: Юлия Кальменс (Чикаго)

Средняя оценка: -1.27

Комментарии


314. Вопрос 3.32: Борясь с хронотипической дискриминацией на турнирах по ЧГК, один знаток предложил организовать фестиваль под названием “СЕМЬ ПОДЗЕМНЫХ КЛЕЙ”, который бы начинался вечером и оканчивался поздней ночью. Какие два слова мы в этом вопросе заменили тремя?

Ответ: месть сов.

Зачёт: точный ответ.

Комментарий: нередко на фестивалях игры начинаются рано утром, что удобно жаворонкам. Знаток же предложил составить расписание фестиваля с учетом интересов сов. “Месть ситхов” — третья часть цикла “Звездные войны”. “Семь подземных королей” — третья часть цикла “Волшебник Изумрудного города”.

Источник: https://www.facebook.com/pavel.khoudiakov/posts/622530467903820 (запись только для друзей).

Автор: Григорий Алхазов (Кишинёв)

Средняя оценка: -1.49

Комментарии


315. Вопрос 3.22: В стихотворении о европейском городе Юрий Иофе пишет, что идет по спирали ИКСОВ, которую также называет “дорогой надежд и обманов”. До 1860 года ИКСОВ было меньше, а их порядок был другим. Назовите ИКС словом иностранного происхождения.

Ответ: арондисман.

Зачёт: аррондисман.

Комментарий: слово “обманов” рифмуется со словом “арондисманов”. Парижские муниципальные арондисманы (округа) расположены по спирали — первый в самом центре и далее по часовой стрелке. С 1795 по 1860 год арондисманов было всего 12, а располагались они не по спирали. Современное распределение арондисманов появилось в результате градостроительной реформы барона ОсмАна.

Источник:
    1. http://lib.ru/POEZIQ/IOFE_J/stihi.txt
    2. https://ru.wikipedia.org/wiki/Париж,_я_люблю_тебя
    3. https://en.wikipedia.org/wiki/Arrondissements_of_Paris

Автор: Максим Коцюруба (Кишинёв)

Средняя оценка: -1.53

Комментарии

Вход в систему